Praxis Review Questions

अब Quizwiz के साथ अपने होमवर्क और परीक्षाओं को एस करें!

Giselle vocalizes and persistently points at the apple across the room while looking back and forth between the object and her mom. She is exhibiting intentional communication by showing she wants the apple. Giselle must be at least ___. A. 9 Months B. 5 Months C. 12 Months D. 10 Months

A. 9 Months

Which of the following best distinguishes a dialect from an accent? A. Differences in language as well as in pronunciation B. Differences in pronunciation only C. Differences due to the influence of a second language D. Differences that are unique to a particular speaker

A. Differences in language as well as in pronunciation

The most serious limitation of employing imitation as an intervention strategy for children with a language impairment is that imitation: A. lacks communicative intention B. relies on semantic knowledge C. is clinician controlled D. is contextualized speech

A. lacks communicative intention

Select which of the following statements best describes a typical developing 1st grader. A. learning to read B. reading to learn C. learning to write D. learning concepts such as colors

A. learning to read

As part of a routine preschool screening, an SLP tests a 4 year old whose speech is characterized by sound omissions, hypernasality, nasal emission, and weak consonants. Which of the following would be most appropriately evaluated initially? A. Oral-motor behavior B. Velopharyngeal function C. Laryngeal function D. Phonological awareness

B. Velopharyngeal function

Of the following sentences, which represents the greatest degree of syntactic complexity? A. Is John helping Bill? B. Why isn't John helping Bill? C. John isn't helping Bill. D. Why is John helping Bill?

B. Why isn't John helping Bill?

Children who present with stuttering-like disfluencies are likely to recover completely up until about what age? A. 3 years old B. 9 years old C. 6 years old D. 2.5 years old

C. 6 years old

Over the past six weeks, the general education teacher implemented evidence-based classroom and teaching modifications for a student struggling in a language arts class. Careful observation has not shown an increase in the student's performance. The teacher informs the student's parents of plans to refer the student to the speech-language pathologist at school. The teacher then makes an official referral to the local educational agency. Which of the following choices is the maximum time allowed by IDEA from the official referral for eligibility determination to completion of the evaluation? A. 14 days B. 30 days C. 60 days D. 120 days

C. 60 days

A transfer student is referred for a language evaluation. The student has spoken English for less than a year, and English is not the primary language used at home. To provide the student with culturally and linguistically appropriate service delivery, the student should be evaluated A. using norms for standard English speakers B. in the student's native language C. with the student's parents present D. based on formal assessments

B. in the student's native language

A 6-year-old child who is a bilingual English-Spanish speaker produced the following statements during a speech-language assessment. I cutted the finger. I played with her yesterday. She eated too much candy. You like ice cream? Maria is going? Father is happy. Buy a new car. Based on the sample, the speech-language pathologist should begin remediation by focusing on: A. adjectives and imperatives B. irregular verbs C. prepositional phrases D. conjunctions and embedding

B. irregular verbs

An SLP evaluates a 5-year-old child who produces several sounds in error. The SLP wants to determine the consistency of the child's errors by conducting stimulability testing. The most appropriate next step for the SLP is to ask the child to produce each misarticulated sound in A. a novel word using real objects as stimuli B. isolation after watching and listening to the SLP produce the sounds correctly C. isolation after listening to recorded samples of correctly produced sounds D. connected speech after listening to the SLP'sS L P's production of the erroneous sounds

B. isolation after watching and listening to the SLP produce the sounds correctly

Which of the following is characteristic of a mild traumatic brain injury? A. Long-term memory loss B. Onset of frequent seizures C. Memory loss of events immediately preceding or following the trauma incident D. Extensive and permanent neurological impairments

C. Memory loss of events immediately preceding or following the trauma incident

Which of the following best describes the etiology of stuttering? A. Premature babies B. Genetics C. Multiple systems involved: genetics, environmental, abnormalities in phonatory system D. Environmental causes

C. Multiple systems involved: genetics, environmental, abnormalities in phonatory system

Which of the following is most likely to occur in an infant with an unrepaired cleft palate? A. Aspiration pneumonia B. Choking C. Nasal regurgitation D. Tongue thrust

C. Nasal regurgitation

A new admission s/p hip fracture with baseline dementia is added to the SLP caseload. Each day during therapy, the patient completes a puzzle independently for 30 minutes. Which of the following best describes this therapy scenario. A. Responsible training B. Spaced retrieval therapy C. Not skilled therapy

C. Not skilled therapy

Computer software that has been developed to facilitate speech and language treatment can best be used: A. in group sessions when the SLP's caseload precludes working individually with clients B. by clients in place of services that would otherwise be provided by an SLP C. by clients under the direction of SLPs D. when SLPs are unavailable

C. by clients under the direction of SLPs

CN ___, ___, ___, ___, & ___ are involved with articulation.

CN V, VII, IX, X, XII

The cranial nerve that innervates the larynx and also innervates the levator veli palatini, palatoglossus, palatopharyngeus is the ___.

CN X

What structure supplies blood to the brain?

Circle of Willis

Which of the following assessment descriptions represents an s/z ratio that is indicative of a vocal pathology? A. 1.0 B. 1.1 C. 1.3 D. 1.5

D. 1.5

Identify the number of morphemes in the following sentence: "It was undeniable that she was the happiest girl in her class." A. 16 B. 17 C. 14 D. 15 E. None of the above

D. 15

When referring to speech sound disorders, which of the following are examples of traditional SODA errors? A. Substitutions B. Assimilations C. Omissions D. A & C E. A, B, and C

D. A & C

Which of the following grammatical morphemes are acquired first in typically developing children? A. present progressive -ing B. possessive 's C. regular plural -s D. A & C

D. A & C

To justify providing individual treatment for a 2½ year old with apraxia of speech, which of the following would be LEAST important for the SLP to include in the evaluation report? A. A description of the child's typical interaction with peers B. Relevant prognostic data C. Information about apraxia of speech D. A description of the language development of the child's older siblings

D. A description of the language development of the child's older siblings

Miss. Malone, a 65-year-old retired attorney, presents with severe nonfluent aphasia secondary to a stroke that occurred more than a year ago. Speech-language intervention helped her regain the ability to answer simple questions but not initiate speech. Citing Miss. Malone's frustration at her limitations, her family requests that she be evaluated for an augmentative and alternative communication (AAC) device. During Miss. Malone's assessment, the SLP determines that she could potentially benefit from AAC services and initiates discussion of intervention goals. Once goals are established and Miss. Malone has her device, she begins to learn how to operate her device to produce common spoken messages. Over the course of intervention, Miss. Malone is ultimately able to produce novel sentences. Which of the following statements reflects the best strategy for the development of Miss. Malone's intervention goals? A. The SLP must decide which goals are appropriate, because the SLP is the communication expert. B. As the patient determines what she needs and wants to say, she must determine the goals. C. Goal setting should be a collaborative process involving the patient and the SLP. D. A number of individuals including, the patient, SLP, and close family members, should collaborate to set goals.

D. A number of individuals including, the patient, SLP, and close family members, should collaborate to set goals.

A 3-year-old child presents for an evaluation of communication skills. When the SLP says "Sit in your seat," the child responds by saying [tɪ ɪ ti]. When the SLP asks the child to "put the big block in the box," the child responds by saying [bɪ bɑ ɪ bɑ]. Based on the responses, the child's primary problem with communication is most likely which of the following? A. Oral-motor weakness B. Poor auditory discrimination C. A fluency disorder D. A receptive and/or expressive language impairment

D. A receptive and/or expressive language impairment

A prospective client is described as a man in his forties who is under chronic stress. He uses his voice extensively in daily life has a hard-driving personality, and exhibits glottal fry. The client has the classic profile of a person at high risk for A. spastic dysphonia B. acute laryngitis C. vocal nodules D. contact ulcers

D. contact ulcers

An SLP is planning treatment for a 5-year-old child with multiple speech-production errors. The most effective strategy the clinician can use to treat the child is to: A. arrange error sounds by developmental pattern and correct them sound by sound B. start with sounds the child can make and use them as bridges to error sounds C. teach sounds in isolation, then use nonsense syllables, and then build to words D. delineate phonological processes in operation and address them through minimal-contrast pairs

D. delineate phonological processes in operation and address them through minimal-contrast pairs

Esophageal and tracheoesophageal (TEP) techniques for producing alaryngeal voice are similar in that both: A. redirect expiratory airflow from the lungs to the esophagus B. result in a vocal fundamental frequency similar to that of laryngeal phonation C. require the speaker to occlude the stoma with either a thumb or a valve D. depend on adequate vibration of the pharyngoesophageal (PE) segment

D. depend on adequate vibration of the pharyngoesophageal (PE) segment

When an SLP assesses the communication skills of a bilingual child, it is important for the SLP to evaluate both languages primarily to: A. establish a baseline of skills in order to track changes B. determine which language to treat C. establish rapport with the child D. diagnose a communication disorder

D. diagnose a communication disorder

A 4-month-old-infant who has a low birth weight but passed a neonatal hearing screening was evaluated for development of communication skills. The speech-language pathologist found that the infant followed moving objects visually, showed interest in mouthing and banging objects, and began sucking in anticipation of eating, but failed to localize to environmental sounds. On the report to the infant's primary care physician, the most appropriate recommendation by the speech-language pathologist is A. consideration of auditory amplification B. hearing-loss counseling for the parents C. careful parent monitoring of the child's speech-language development D. evaluation of auditory function by an audiologist

D. evaluation of auditory function by an audiologist

Omissions of past tense -ed, possessive -s, and 'f' substitutions for voiceless th- in the final position of words are examples of errors most related to which of the following? A. Language disorder B. Spanish-influenced English C. Standard American English D. African American English

D. African American English

Following a screen, you suspect that a patient has dysarthria. What is important to include in the assessment process? A. oral facial motor exam B. Cranial nerve assessment C. Diadochokinetic rates D. All of the above

D. All of the above

Which of the following is an etiology for cortical dementia? A. Amyotrophic lateral sclerosis B. Parkinson's disease C. Huntington's disease D. Alzheimer's disease

D. Alzheimer's disease

An SLP plans to give Cody, an 8-year-old boy with autism spectrum disorder, tangible reinforcement in conjunction with the use of manual signs during an object labeling task. At first, reinforcement will be presented every time Cody produces a sign correctly. After several sessions, however, reinforcement will be given after every third instance of correct labeling. Which of the following choices best describes the SLP's reinforcement schedule? A. Fixed-ratio schedule followed by variable-ratio schedule B. Fixed-ratio schedule followed by continuous schedule C. Continuous-ratio schedule followed by fixed-interval schedule D. Continuous-ratio schedule followed by fixed-ratio schedule

D. Continuous-ratio schedule followed by fixed-ratio schedule

Which of the following muscles is responsible for changing vocal pitch? A. Posterior cricoarytenoid B. Thyroarytenoid C. Lateral cricoarytenoid D. Cricothyroid

D. Cricothyroid

A child with discourse problems is most likely to need remediation directed at which of the following? A. Morphology B. An initial lexicon C. Gestural communication D. Cohesive devices

D. Cohesive devices

Laborious, halting, telegraphic utterances are typical of clients with which of the following types of aphasia? A. Conduction B. Anomic C. Wernicke D. Transcortical motor

D. Transcortical motor

During the course of treatment, as a child manipulates various toys in symbolic play, the clinician vocalizes the child's actions (for example, "You're putting the pan on the stove."). The intervention technique used by the clinician is identified as: A. the mand model B. reauditorization C. self-talk D. parallel talk

D. parallel talk

The to-and-fro movement of air molecules because of a vibrating object is referred to as ___.

Oscillation

Muscles that contribute to velopharyngeal closure through tensing or elevating the velum are the:

Palatoglossus, tensor veli palatini, and levator veli palatini

What facial muscle do we use to smile?

Risorius

What muscle is circled?

Risorius

The structure that regulates posture, equilibrium, and coordinated fine-motor movements is the ___.

cerebellum

Respiration relies on the muscles of inspiration and expiration. The thick, dome-shaped muscle that separates the abdomen from the thorax is called the

diaphragm

an important structure adjacent to the brainstem that contains the hypothalamus and thalamus is called the ___.

diencephalon

The neurons that transmit information away from the brain are called

efferent

Primary muscle of the lips

orbicularis oris

The corpus striatum is composed of three nuclear masses, which are the ___.

globus pallidus, caudate nucleus, and basal ganglia

/r/ and /l/ can be categorized as ___.

liquids

The two properties of a medium that affect sound transmission are:

mass and elasticity

A sinusoidal wave is a sound wave:

with horizontal and vertical symmetry with one peak and one valley with a single frequency that is a result of simple harmonic motion

The clinical interview with the family is used to ______.

- Obtain information and data - Provide support & build rapport - Inform the family/client

If a speaker said, "I just love 'em and leave 'em," the phrase "leave 'em" could be transcribed as:

/liv mˌ/

The laryngopharynx and the oropharynx add resonance to the sounds produced by the larynx. The nasopharynx adds noticeable resonance to which sounds?

/m/, /n/, & /ŋ/

Place the phonemes below in the order of typical acquisition. /p/ /θ/ /k/ /ʒ/

/p/ /k/ /θ/ /ʒ/

A semivowel that can be categorized as a voiced bilabial glide that is +anterior and +continuant is the:

/w/

Two laws applicable to public schools are the Individuals with Disabilities Education Act (IDEA) and Section 504 of the Rehabilitation Act. While these laws are similar in that they both address the communication needs of students with disabilities, there are clear differences between them. For each statement, determine whether it applies to IDEA, 504, or both. 1. A child is eligible under this law if he or she has any physical or mental condition that substantially limits a major life activity. 2. This law provides an eligible student with an IEP 3. A child must have one of 13 specified disabilities to receive services under this law. 4. This law prevents discrimination against persons with disabilities. 5. The Office of Special Education and Rehabilitative Services administers this law. 6. The Office for Civil Rights administers this law.

1. 504 2. IDEA 3. IDEA 4. Both 5. IDEA 6. 504

An SLP works with a toddler on combining words and assessing a variety of semantic relations. Match each toddler utterance with the semantic relation it best reflects. 1. "Doggie eat" 2. "Big cookie" 3. "More cookie" 4. "Eat cookie" A. Agent + Action B. Action + Object C. Attribute + Entity D. Recurrence

1. A 2. C 3. D 4. B

Match each treatment strategy with the type of dysarthria for which the strategy is most effective. 1. Spastic 2. Ataxic 3. Flaccid 4. Hyperkinetic A. Using rhythmic or metered cueing B. Implementing sensory tricks C. Practicing relaxation exercises D. Performing pushing-pulling exercises

1. C 2. A 3. D 4. B

Place the swallowing function with the cranial nerve that is primarily responsible for making the function possible. 1. Closing the vocal folds for airway protection 2. Closing the lips to contain the bolus 3. Closing the jaw for chewing 4. Controlling the tongue movement during bolus transport in the mouth A. CN V B. CN VII C. CN X D. CN XII

1. C 2. B 3. A 4. D

An SLP conducts a videofluoroscopic swallowing study (VFSS) for a patient referred for suspected dysphagia secondary to stroke. The SLP finds that the patient has dysphagia characterized by delayed onset of the pharyngeal response, limited lingual retraction, limited oropharyngeal propulsion, and the need to swallow two or more times to clear boluses. Some aspiration of hypopharyngeal residue occurs after the swallow from the pyriform sinuses. During the VFSS, several compensatory maneuvers (chin-down posture) were attempted to test their efficacy. The SLP is developing a treatment plan to eliminate aspiration and to improve lingual strength, retraction, and oropharyngeal propulsion through a combined strategy using compensatory and restorative maneuvers, including a progressive resistive exercise program of isometric tongue-press exercises using an instrument that measures the pressure of tongue contact with the hard palate (IOPI or SwallowStrong). The SLP measures the patient's baseline maximum tongue-press pressure generation with the instrument and begins treatment. Place the following steps of the progressive resistive exercise component of treatment in the correct order. 1. Obtain measure of maximum tongue-press pressure generation 2. Perform VFSS to evaluate efficacy of tongue-press progressive resistive exercise program on swallow impairments 3. Calculate an exercise target that is below the patient's maximum pressure generation, and begin progressive resistance exercises 4. Increase exercise target and continue 3 sets of 10 reps per day, 3 days/week of exercises at the same treatment target.

1. Obtain measure of maximum tongue-press pressure generation 3. Calculate an exercise target that is below the patient's maximum pressure generation, and begin progressive resistance exercises 4. Increase exercise target and continue 3 sets of 10 reps per day, 3 days/week of exercises at the same treatment target. 2. Perform VFSS to evaluate efficacy of tongue-press progressive resistive exercise program on swallow impairments

Place an "S" or a "C" below to indicate whether each characteristic listed is symptomatic of stuttering or cluttering. 1. Excessive frequency of part- and whole-word repetitions 2. Excessive frequency of revisions 3. Active attempts to avoid or conceal communication difficulties 4. Reduced intelligibility in conjunction with rapid rate 5. Misarticulation of multisyllable words 6. Limited concern and awareness of communication difficulties 7. Use of word avoidance and circumlocution in response to anticipated disfluency

1. S 2. C 3. S 4. C 5. C 6. C 7. S

Place the following aspects of phonological awareness in developmental order starting with the earliest skill to emerge.: 1. Listing words that start with the same sound 2. Recognizing words that rhyme 3. Counting syllables in single words 4. Creating words by blending onset and rime

2. Recognizing words that rhyme 3. Counting syllables in single words 4. Creating words by blending onset and rime 1. Listing words that start with the same sound

The acoustic reflex is triggered in a person with typical hearing when the listener is exposed to a sound above approximately A. 85 dB HTL B. 130 dB HTL C. 1000 Hz D. 6500 Hz

A. 85 dB HTL

Which of the following is an example of deaffrication? A. "Chew" is pronounced /ʃu/ B. "Round" is pronounced /waʋnd/ C. "Dog" is pronounced /dɔd/ D. "Van" is pronounced /fæn/

A. "Chew" is pronounced /ʃu/

Which of the following statements best represents a syntactic structure characteristic of Spanish-influenced English? A. "She no do laundry today." B. "He is going?" C. "Lady her shoes." D. "She be runnin' fast."

A. "She no do laundry today."

Which of the following, if observed in the speech of an African American child, is most likely to represent a dialectical variation rather than an articulation error? A. /f/ for /θ/ in postvocalic position B. /θ/ for /s/ in all positions C. Affricates for fricatives in word-final position D. Dentals for velars in word-initial position

A. /f/ for /θ/ in postvocalic position

An SLP works at a hospital with the adult acute-care inpatient team. During a clinical bedside swallow evaluation, a patient asks the SLP how common it is for adults to experience a swallowing disorder. The SLP tells the patient that recent research indicates the prevalence of adults with swallowing problems each year is A. 1 in 25 B. 1 in 50 C. 1 in 100 D. 1 in 200

A. 1 in 25

According to ASHA and the Joint Committee on Infant Hearing (JCIH) recommendations of 2007, for babies who fail the newborn hearing screening, the follow-up diagnostic audiologic evaluation should be completed no later than A. 3 months of age B. 6 months of age C. 9 months of age D. 12 months of age

A. 3 months of age

An SLP will be working with a new mother to evaluate a term infant's feeding and swallowing skills to determine the infant's readiness for oral feeding. The infant has been diagnosed with a unilateral, complete cleft lip without cleft palate. The mother asks the SLP about what caused the infant's cleft lip. The SLP explains that while we know that both genetic and environmental factors are likely involved with this congenital birth abnormality, there is no consensus in research about a single cause of cleft lip. The SLP completes a comprehensive oral sensorimotor and behavioral observation examination prior to initiating an oral-feeding trial. The following is documented in the SLP's assessment notes: Right-sided complete cleft lip; otherwise unremarkable oral peripheral mechanism examination; medical chart notes no associated neurological difficulties or diagnosed syndromes; primitive reflexes present (e.g.for example, rooting); normal observation of posture, positioning, tone, and motor activity; infant presents awake, alert, and calm; baseline vital signs at rest are normal and no changes in respiratory rate, heart rate, or oxygen saturation noted with nonnutritive sucking; no respiratory stridor noted; mild external support needed to increase lip closure at introduction of pacifier nipple, and mother with strong desire to breastfeed. The mother expressed some anxiety and concern surrounding feeding an infant with a cleft lip. During which of the following weeks of pregnancy did the infant's craniofacial structures not develop completely? A. 4-to 7 B. 8-to 11 C. 12-to 15 D. 16-to 19

A. 4-to 7

Which of the following statements best reflects the role of stimulability in generating a prognosis for remediation of gliding in a 9-year-old child? A. A child who is not stimulable for /r/, as in the word run will require treatment for the sound to be acquired. B. Even if the child is not stimulable for /r/, as in the word run, the sound will still develop without treatment. C. If the child is stimulable for /l/, as in the word last, the SLP can expect the /r/, as in the word run to improve at the same rate. D. Stimulability does not play a role in determining a prognosis for remediation of speech sound disorders.

A. A child who is not stimulable for /r/, as in the word run will require treatment for the sound to be acquired.

Alex's mom notices that Alex is making mistakes on specific speech sounds while talking. She identifies that he is unable to correctly pronounce "k, m, and g." Alex is 5 years old. What should Alex's mom do next? A. A complete SLP eval B. Oral Motor Exercises C. Nothing, this is normal for his age D. Language therapy

A. A complete SLP eval

A speech language pathologist evaluated Sophia Allen, a 6-year-old girl. The SLP completed a case history with Sophia's mother and conducted a comprehensive speech and language assessment. During the case study the SLP learned that Sophia has had a normal developmental history and is currently healthy with no known neurological deficits. Sophia's mother first became concerned when Sophia could not describe her school day. Sophia's sentences lacked detail or were composed of very basic words and consisted of simple sentence structures. Sophia's first-grade teacher reported that her speech sounds immature compared to that of her classmates and that she frequently does not follow directions. During the assessment Sophia passed the oral mechanism examination and hearing screening. Sophia received a standard score of 99, placing her in the 47th percentile on a commonly used receptive vocabulary test. Based on Sofia's case history and presenting problem, which of the following is most likely the etiology of her suspected language disorder? A. A developmental disorder B. A psychogenic disorder C. An auditory processing disorder D. An acquired disorder

A. A developmental disorder

Which of the following procedures best establishes whether velopharyngeal dysfunction causing hypernasality is present in a patient with flaccid dysarthria? A. A nasopharyngoscopy B. A videofluoroscopic swallow study C. A laryngoscopy D. A manometry

A. A nasopharyngoscopy

A correct-response rate of 51 percent on a two-choice picture-pointing task would most likely indicate which of the following? A. A random pointing response B. Successful intervention C. Development of crucial discrimination skills by the client D. Readiness to progress to a three-picture point task

A. A random pointing response

Which of the following is the ratio of reinforcement that will most quickly cause a newly acquired behavior to be habituated? A. A random ratio of tokens to correct responses B. A ratio of 1 token to 1 correct response C. A ratio of 1 token to 4 correct responses only D. A ratio of 1 token to 10 correct responses only

A. A random ratio of tokens to correct responses

Which of the following statements regarding statistical significance is true? A. A statistically significant difference can occur between experimental groups and control groups even if the magnitude of difference between the groups is quite small. B. If a statistically significant difference between groups occurs, it means that large and important change occurs in at least one of the groups. C. Statistically significant findings in a clinical research study suggest that the treatment is successful and highly recommended. D. The terms "statistically significant" and "clinically significant" are interchangeable.

A. A statistically significant difference can occur between experimental groups and control groups even if the magnitude of difference between the groups is quite small.

No smiling by 6 months, no gesturing by 12 months, no words by 16 months, and a lack of interest in other children by 24 months are examples of early signs of what? A. ASD B. Down Syndrome C. Fragile X D. Aphasia

A. ASD

A 62-year-old male presents to an outpatient SLP following an extended stay at a rehabilitation facility. He had a left-hemisphere stroke three months ago and currently has moderate Broca's aphasia and severe apraxia of speech. It is difficult for him to participate in conversation because of the combination of his word-finding deficits and apraxia of speech. The patient's normal speech pattern consists of one- to two-word phrases, at times including paraphasias. He is able to write but often misspells words. During evaluation, the SLP finds the patient is able to initiate communication, recognize and categorize picture symbols, and comprehend familiar words and phrases. In general, his communication is fragmented and inefficient, but he will try anything to get his message across. Which of the following AAC treatment strategies would be most appropriate for the patient to start with? A. Accessing stored messages in a speech-generating device B. Learning sign language C. Answering multiple-choice questions and writing answers D. Using a 20-item picture board to convey essential wants and needs

A. Accessing stored messages in a speech-generating device

An SLP uses evidence-based practice by integrating the perspectives and values of the client, patient, or caregivers into the treatment plan. Which of the following best reflects simultaneous incorporation of the host culture's perspectives and values and maintenance of the native culture's perspectives and values? A. Acculturation B. Assimilation C. Enculturation D. Ethnocentrism

A. Acculturation

Which of the following is the minimal contrast approach most appropriately used to treat? A. Phonological disorders B. Dysphagia C. Dysarthria D. Global aphasia

A. Phonological disorders

A 24-year-old male self-refers for a fluency evaluation. His presenting complaint is stuttering. During conversation at the initial assessment, he speaks intelligibly at a typical rate and produces no overt stutter-like disfluencies. He reports that he often expects to stutter while conversing, but that he usually can prevent or conceal the occurrence of the expected fluency disruptions either by substituting a word or by inserting a pause or "um" before the word upon which he expects to be disfluent. He states that these strategies are useful and that he would like to be able to "talk without thinking about talking." He reports that he attended speech therapy from elementary school through high school and that it helped him reduce disfluency significantly during therapy activities, but his disfluency frequency did not change much during activities outside of therapy. He fears that coworkers will react negatively to hearing him stutter. Consequently, he talks as little as possible at work. Which of the following approaches would be the most direct way to address the patient's fear of how coworkers might react to his disfluent speech? A. Application of principles from cognitive behavioral therapy B. Fluency-shaping therapy based on prolonged speech C. Pharmacological management D. Application of progressive relaxation therapy principles

A. Application of principles from cognitive behavioral therapy

Researchers are conducting an experiment that depends on the use of professional judgments of the speech produced by 30 research subjects. Which of the following actions should the researchers take to determine interjudge reliability? A. Asking two SLPs to each independently rate the 30 subjects B. Asking an SLP to rate the 30 subjects C. Asking one SLP to rate the 30 subjects before treatment and another SLP to rate them after treatment D. Asking an SLP to rate the 30 subjects and then rate them again two weeks later

A. Asking two SLPs to each independently rate the 30 subjects

Reduplication and denasalization are examples of which type of typical phonological processes? A. Assimilation B. Substitution C. Syllable structure D. Metathesis

A. Assimilation

Christel and Sharon are 14-month-old fraternal twins. Christel has more intelligible words than her sister, although she talks far less frequently. Sharon speaks almost exclusively in running jargon that nobody understands. Their parents are worried about Sharon's development because her language skills appear to be behind those of her sister. Which of the following is an SLP's best response to the parents' concern? A. At Sharon's age, jargon is normal and may even continue for another three to four months. B. Christel's language development is more abnormal because she should be beyond single words. C. Both girls were exposed to the same language environments, so the relative lack of intelligible words indicates a language delay. D. Jargon should not persist past the age of 12 months and thus Sharon may be presenting a language disorder.

A. At Sharon's age, jargon is normal and may even continue for another three to four months.

Which of the following types of cerebral palsy is characterized by slow, arrhythmic writhing and involuntary movements of the extremities? A. Athetosis B. Spasticity C. Hypotonia D. Bulbar palsy

A. Athetosis

A client exhibits weakness, atrophy, and fasciculations of the right side of the tongue and lower face. The client also has right vocal-fold weakness and nasal regurgitation of fluid when swallowing. These problems are the result of damage to which part of the nervous system? A. Brain stem B. Cerebellum C. Left cerebral cortex D. Right cerebral cortex

A. Brain stem

An SLP recommends a patient perform a chin-down posture (CDP) when swallowing liquids. Which THREE of the following statements accurately represent published evidence regarding the chin-down posture? A. CDP reduced thin liquid aspiration caused by delayed pharyngeal response in people who had a stroke. B. CDP increased the speed and completeness of oral transit in people with dysphagia after a stroke. C. CDP was shown to narrow the width of the laryngeal inlet and widen the vallecular space during swallowing. D. CDP resulted in an increased upper esophageal sphincter opening diameter in patients with Parkinson's disease. E. CDP does not reduce aspiration after the swallow in patients who had a stroke and aspirate pyriform sinus residue.

A. CDP reduced thin liquid aspiration caused by delayed pharyngeal response in people who had a stroke. C. CDP was shown to narrow the width of the laryngeal inlet and widen the vallecular space during swallowing. E. CDP does not reduce aspiration after the swallow in patients who had a stroke and aspirate pyriform sinus residue.

Which of the following should a typically developing 9-month-old child begin to demonstrate? A. Canonical babbling B. Cooing C. Vocatives D. Differentiated cries

A. Canonical babbling

A child has been demonstrating significant progress in producing the targeted responses in the therapy room; however, observation in the classroom indicates little mastery of the targeted responses. Which of the following would be the most appropriate revision to the treatment plan? A. Delivering treatment in the classroom B. Increasing the frequency of treatment in the therapy room C. Increasing the duration of treatment in the therapy room D. Changing the IEP goals

A. Delivering treatment in the classroom

Which of the following would be most likely to help a client who has aphonia? A. Development of phonation through coughing or throat clearing B. Pairing the production of /s/ and /z/ C. Respiratory exercises D. Easy initiation of phonation

A. Development of phonation through coughing or throat clearing

Inhalation during respiration is primarily carried out through the movement of which of the following muscles or muscle groups? A. Diaphragm B. Internal intercostals C. External intercostals D. Scalenes

A. Diaphragm

Michael is a 32-month-old boy who has been receiving early intervention services over the past ten months for delayed speech and expressive-language development. Although his birth was reportedly unremarkable, Michael does have a history of recurrent otitis media with effusion. His parents described him as having been a "well-behaved and quiet baby." When Michael began receiving services, he communicated mainly through gestures and crude vocalizations. An open resting mouth position with slight tongue protrusion was sometimes noted. However, his receptive-language skills were found to be age appropriate and he showed no oral motor deficits during feeding. Michael's expressive-language skills have shown some progress since he began working with the speech-language pathologist, but he remains poorly intelligible. Michael's imitation of tongue, lip, and jaw movements is characterized by inconsistent groping and errors of sequencing not observed in his spontaneous oral movements. Michael has an age-appropriate vocabulary and produces utterances of up to five words. Articulation errors, especially metathesis of phones and syllables, increase as his utterance length increases. Michael's intelligibility is greatest at the single-word level. Automatic speech and highly familiar utterances are much more intelligible than his imitated productions. Michael demonstrates speech behaviors that are typical of children diagnosed with which of the following? A. Childhood apraxia of speech B. Conductive hearing loss C. Hyperkinetic dysarthria D. Autism spectrum disorder

A. Childhood apraxia of speech

Which of the following is the most common phonological problem evidenced by young children aged 18-29 months? A. Cluster reduction B. Velar fronting C. Nasal assimilation D. Dimunitization

A. Cluster reduction

A speech language pathologist evaluated Sophia Allen, a 6-year-old girl. The SLP completed a case history with Sophia's mother and conducted a comprehensive speech and language assessment. During the case study the SLP learned that Sophia has had a normal developmental history and is currently healthy with no known neurological deficits. Sophia's mother first became concerned when Sophia could not describe her school day. Sophia's sentences lacked detail or were composed of very basic words and consisted of simple sentence structures. Sophia's first-grade teacher reported that her speech sounds immature compared to that of her classmates and that she frequently does not follow directions. During the assessment Sophia passed the oral mechanism examination and hearing screening. Sophia received a standard score of 99, placing her in the 47th percentile on a commonly used receptive vocabulary test. Which of the following methods will best measure Sophia's progress on her language goals and objectives? A. Completing a language sample analysis B. Administering a standardized receptive and expressive language battery C. Administering a vocabulary assessment tool D. Administering a reading test

A. Completing a language sample analysis

Which of the following techniques is most effective when treating phonation in a patient with spastic dysarthria? A. Completing head and neck relaxation exercises B. Engaging in lip-stretching exercises C. Providing instruction in phonetic placement D. Working on pitch-range exercises

A. Completing head and neck relaxation exercises

At the start of a speech-language evaluation, James, a 3-year-old patient, fails a hearing screening. There is no reported hearing deficit, and the child appears to easily understand instructions from the SLP. Which of the following is the most appropriate next step for the evaluating SLP? A. Completing the speech-language evaluation and referring James for an audiologic evaluation B. Stopping the speech-language evaluation and referring James for an audiologic evaluation C. Continuing with the speech-language evaluation and noting that the screening was failed likely because of poor cooperation D. Interviewing the parents, reading any available outside reports, and basing a diagnosis on those data

A. Completing the speech-language evaluation and referring James for an audiologic evaluation

An SLP works at a voice clinic and learns that a patient, who has arrived for an initial voice evaluation, has not received medical evaluation by a laryngologist. Which of the following actions is the best plan for referring the patient to a laryngologist for a medical evaluation? A. Completing the voice evaluation and referring the patient to a laryngologist for a medical evaluation prior to initiating therapeutic intervention B. Completing the voice evaluation and treatment plan, as the SLP is not required to obtain medical information from a physician and can diagnose and treat voice disorders without referring the patient C. Completing the voice evaluation and referring the patient to a laryngologist while initiating therapeutic intervention, knowing that the patient will see the laryngologist soon D. Deferring the voice evaluation until after the patient has seen a laryngologist for a medical evaluation

A. Completing the voice evaluation and referring the patient to a laryngologist for a medical evaluation prior to initiating therapeutic intervention

Which type of validity measures the degree to which a new test correlates with an established test of known validity? A. Concurrent validity B. Criterion validity C. Content validity D. Predictive validity

A. Concurrent validity

After completing an evaluation of a 5-year-old patient, an SLP finds that the child's speech contains numerous phonological error patterns. If the SLP follows a developmental sequence in planning intervention, which of the following is most appropriate to target first? A. Consonant assimilation B. Gliding C. Deaffrication D. Cluster reduction

A. Consonant assimilation

Which of the following is an example type of vocabulary used to program AAC devices and includes highly functional words/phrases and social greetings. A. Core B. Fringe C. Static D. All of the above E. None of the above

A. Core

Which of the following conditions is primarily characterized by premature closure of the sutures of the skull? A. Craniosynostosis B. Craniopharyngioma C. Deformational plagiocephaly D. Positional plagiocephaly

A. Craniosynostosis

Contractible auxiliary is a later developing morpheme. Which of the following sentences contains an example of contractible auxiliary? A. Daddy's jumping up and down B. It's hot in the summer C. Are you going? D. None of the above

A. Daddy's jumping up and down

A 55-year-old female presents for a speech evaluation because of concerns with recent changes in the clarity of her speech. She has also noticed occasional coughing when drinking, and her friends have commented that her voice sounds different. A recent visit to her primary care doctor could not determine the cause of her speech changes or coughing. Select the THREE most important factors for the SLP to explore during the patient interview. A. Difficulties with hearing, vision, and fine or gross motor skills B. History of a psychological or psychiatric disorder C. Employment history and current stressors in her life D. Onset and duration of her speech changes E. Consistency of the symptoms

A. Difficulties with hearing, vision, and fine or gross motor skills D. Onset and duration of her speech changes E. Consistency of the symptoms

A large metropolitan school district wants to determine the prevalence of developmental stuttering among all enrolled students during the past year. Which of the following approaches is most appropriate for accomplishing the task? A. Dividing the total number of students who currently stutter by the total number of students who were enrolled during the past year B. Subtracting the total number of students who received treatment for stuttering during the past year from the total number of students who currently stutter C. Multiplying the total number of students who currently stutter by the total number of students enrolled during the past year D. Dividing the total number of newly identified students who stutter by the total number of students enrolled during the past year

A. Dividing the total number of students who currently stutter by the total number of students who were enrolled during the past year

To innervate a muscle, the brain sends signals down via which motor neurons? A. Efferent Neurons B. Afferent Neurons C. Rostral Neurons D. Dendrites

A. Efferent Neurons

During a speech-language evaluation at a preschool, a child has difficulty with receptive language tasks and responds with only one-word utterances to expressive language items. The child's eye contact is poor throughout the session. When observed with the other children in class, the child does not engage with peers but prefers self-stimulating behaviors such as flapping arms and throwing papers up in the air. According to the child's teacher, these are typical behaviors for the child. Given the data obtained, which of the following steps is most appropriate for the SLP to take next? A. Engaging in an interprofessional practice (IPP) to determine the appropriate diagnosis of ASD B. Conducting the full evaluation when the child is more cooperative so that accurate recommendations can be made C. Initiating expressive language therapy to focus on expanding conversational utterances D. Training the teacher to deliver language stimulation tasks because the child does not respond well to the SLP

A. Engaging in an interprofessional practice (IPP) to determine the appropriate diagnosis of ASD

An 82-year-old female patient is admitted to a skilled nursing facility following an acute hospital stay. Her diagnoses include urinary tract infection, frequent falls with subsequent hip fracture, and chronic obstructive pulmonary disease (COPD). No surgery was required for the hip fracture. Before admission to the hospital, she was living independently and able to complete all activities of daily living (ADLs) and instrumental activities of daily living (IADLs) on her own. She is referred to the facility SLP for cognitive screening because she has difficulty carrying over new information, confusion regarding weight-bearing status, and difficulty processing directions. The SLP administers the Montreal Cognitive Assessment, and the patient scores 17/30. The patient's main deficits are in the areas of short-term memory, executive functioning, and planning. After the patient is appropriately treated both medically and therapeutically, cognitive deficits remain and the interdisciplinary team does not recommend that the patient return home independently. The patient's family is frustrated with this recommendation and seeks input from other sources. Which of the following treatment procedures is most appropriate for the patient? A. Engaging in spaced retrieval B. Naming divergent items C. Following specific directions D. Completing word searches

A. Engaging in spaced retrieval

A public-school-based speech-language pathologist is employed in a state that sets the maximum caseload at 65. However, the clinician's caseload is currently at 64 with a waiting list of 10 additional students. The school principal insists that the speech-language pathologist enroll the 10 students immediately, because the district cannot locate another clinician to assist with the caseload. Which of the following is the most appropriate way for the speech-language pathologist to address the situation? A. Enroll 1 of the 10 students and provide the principal with a written statement of caseload needs, mentioning the amount, type, and frequency of treatment B. Refer the 10 students to a speech-language pathologist working in a private setting C. Suggest that the principal ask a school speech-language pathologist from another school district to take the 10 students D. Maintain current caseload until an additional speech-language pathologist is hired

A. Enroll 1 of the 10 students and provide the principal with a written statement of caseload needs, mentioning the amount, type, and frequency of treatment

Which of the following treatment goals related to pragmatics best addresses a young child's use of language? A. In a play situation, the child will request a turn, either verbally or through gestures, 90 percent of the time. B. In a treatment session, the child will produce a two-word combination 90 percent of the time. C. In conversation, the child will produce correct velars 90 percent of the time. D. When telling a story, the child will use the standard dialect's irregular past tense forms of five specific verbs 90 percent of the time.

A. In a play situation, the child will request a turn, either verbally or through gestures, 90 percent of the time.

Shana's speech pathologist described her challenges as the "inability to recognize and manipulate the spoken parts of sentences and words." This means that Shana is having difficulty with _____. A. Phonological Awareness B. Print Knowledge C. Phonemic Awareness D. Morphological Awareness

A. Phonological Awareness

Alan, a 62-year-old, right-handed African American male, sustained a traumatic brain injury mostly affecting his right hemisphere and bi-lateral frontal lobes. Alan was hospitalized for 24 days before being discharged to an inpatient brain injury rehabilitation program. His spouse attends his initial evaluation sessions. The SLP reports impairments in memory, poor awareness of deficits, and some changes in his communication skills. The SLP already has current information about Alan's performance on a standardized language battery suggesting minimal impairments. Therefore, the SLP completes an evaluation of Alan's functional communication skills. The SLP's assessment involves testing his functional use of humor, facial expressions, nonverbal communication strategies, and understanding functional written materials. The SLP hopes to use this information to determine the impact of Alan's impairments on his daily life. The SLP determines that Alan would benefit from an external memory aid, specifically a memory notebook. The SLP wants to be sure that Alan can learn to use the aid but is concerned that because of his memory impairment, he will struggle to retain the basic information about how to use the aid. The SLP discourages Alan from guessing and intervenes with support before Alan can make a mistake when using the device. Which of the following cognitive rehabilitation practices is most appropriate for the SLP to use to help increase Alan's success in learning basic information about his external memory aid? A. Errorless learning B. Attention process training C. Method of vanishing cues D. Expanded rehearsal

A. Errorless learning

Naturalistic teaching chiefly involves which of the following? A. Establishing successful and useful communication B. Using multiple trials and training techniques C. Using more adult-initiated interactions than child-initiated interactions D. Using differential reinforcement, fading, and modeling

A. Establishing successful and useful communication

An 8-year-old male diagnosed with encephalitis was identified with a resultant in a profound bilateral hearing loss. He underwent a bilateral cochlear implantation and needs aural rehabilitation. Which THREE of the following are appropriate recommendations for intervention? A. Evaluating the child's functional communication performance B. Participating in auditory perception training C. Encouraging the child to rely on American Sign Language D. Counseling the child and family E. Limiting the child's exposure to noisy environments

A. Evaluating the child's functional communication performance B. Participating in auditory perception training D. Counseling the child and family

Which of the following is characteristic of child‑directed speech? A. Exaggerated pitch contours B. Imprecise articulation C. Increased speech rate D. Shortening the pauses between words

A. Exaggerated pitch contours

Brennan, a 50-year-old senior corporate executive, is referred to an SLP with a diagnosis of chronic traumatic encephalopathy (CTE). His presenting complaints are increasing forgetfulness, difficulties learning new material, and managing money. Onset of these problems was gradual, beginning an undetermined number of years prior to the evaluation. Brennan is a college graduate who attended school on a football scholarship, after which he played professionally for two years. Although a cooperative patient, Brennan's initial assessment was incomplete due in largely to his slow responses to stimuli. Still, mild deficits in problem solving, short-term memory, and deductive reasoning were noted. Further assessment is recommended, to be followed by treatment. After hearing the SLP's recommendation, Brennan's wife is concerned that Brennan will soon be unable to care for himself. She stated she is willing to bring him back as often as necessary in hopes of improving his condition. Which of the following statements is true regarding the recommendation for Brennan's additional assessment? A. Given the likelihood of further cognitive deficits and Brennan's delayed responses, diagnostic tasks should be prioritized so that Brennan's cognitive skills are tested first. B. Given that Brennan is a professional in a high-pressure occupation, it is important that further testing not be stressful to his speech-language system. C. Given the likelihood of Brennan becoming frustrated, additional assessment should be limited to family reports. D. Given Brennan's history, further evaluation should be completed only after he receives a head CT scan.

A. Given the likelihood of further cognitive deficits and Brennan's delayed responses, diagnostic tasks should be prioritized so that Brennan's cognitive skills are tested first.

A 75-year-old man presents with conversational speech characterized by prolonged silent intervals and hypophonia. Resonance is normal, but voice quality is rough and tremulous. Pitch is relatively unaffected. There is no evidence of speech deterioration over time. Which of the following diagnoses is the most reasonable based on the patient's data? A. Hypokinetic dysarthria B. Ataxic dysarthria C. Hyperkinetic dysarthria D. Flaccid dysarthria

A. Hypokinetic dysarthria

Oropharyngeal dysphagia in a child with Down syndrome is most likely caused by which of the following factors? A. Hypotonia B. Digestive problems C. Pneumonia D. Aversive feeding behaviors

A. Hypotonia

Miss. Malone, a 65-year-old retired attorney, presents with severe nonfluent aphasia secondary to a stroke that occurred more than a year ago. Speech-language intervention helped her regain the ability to answer simple questions but not initiate speech. Citing Miss. Malone's frustration at her limitations, her family requests that she be evaluated for an augmentative and alternative communication (AAC) device. During Miss. Malone's assessment, the SLP determines that she could potentially benefit from AAC services and initiates discussion of intervention goals. Once goals are established and Miss. Malone has her device, she begins to learn how to operate her device to produce common spoken messages. Over the course of intervention, Miss. Malone is ultimately able to produce novel sentences. Which THREE of the following are additional communication strategies that are valid for Miss. Malone? A. Incorporating multimodal communication strategies, such as gestures and writing B. Teaching regular communication partners how to listen and best respond to her C. Limiting social engagements, since language demands are beyond the severely impaired AAC user D. Encouraging her to take an active role in initiating communication E. Phasing out device use to increase the rate of spoken-language recovery

A. Incorporating multimodal communication strategies, such as gestures and writing B. Teaching regular communication partners how to listen and best respond to her D. Encouraging her to take an active role in initiating communication

Which of the following will most effectively decrease the fundamental frequency? A. Increasing the mass of the vocal folds B. Increasing the subglottal pressure C. Raising the position of the larynx within the neck D. Lengthening the vocal folds

A. Increasing the mass of the vocal folds

A school-based SLP wants to discharge a fifth-grade student. The student has mastered all his goals except /r/forward slash r forward slash in conversational speech. The SLP worked with the student for the entire school year, and the student is able to use /r/forward slash r forward slash accurately 75 percent of the time. However, the goal is to reach 90 percent accuracy. Because the student worked on the skill all year and still has not met the established goal, the SLP feels the student performs at his highest possible level. The SLP does not think it is in the student's best interest to continue pulling the student out of class to address the goal. The parents are upset and are questioning the SLP's decision. Which of the following statements from the American Speech-Language-Hearing Association (ASHA) Code of Ethics can be used to back up the SLP's decision? A. Individuals who hold the Certificate of Clinical Competence shall evaluate the effectiveness of services provided, technology employed, and products dispensed, and they shall provide services or dispense products only when benefit can reasonably be expected. B. Individuals shall honor their responsibility to hold paramount the welfare of persons they serve professionally or who are participants in research and scholarly activities, and they shall treat animals involved in research in a humane manner. C. Individuals shall use every resource, including referral and/or interprofessional collaboration when appropriate, to ensure that quality service is provided. D. Individuals shall provide all clinical services and scientific activities competently.

A. Individuals who hold the Certificate of Clinical Competence shall evaluate the effectiveness of services provided, technology employed, and products dispensed, and they shall provide services or dispense products only when benefit can reasonably be expected.

The mechanics of breathing are very important for speech. The diaphragm contracts and the thoracic cavity expands during which phase of respiration? A. Inhalation B. Exhalation C. Both D. Neither

A. Inhalation

A kindergarten teacher requests a speech screening for a student who is 5 years, 2 months old. The teacher observes that the student has a lisp during conversation and reading activities. Results of the screening note an interdental sound production for /s/, as in the word sit and /z/, as in the word zebra in all positions of words. The oral peripheral screening appears unremarkable and adequate for speech production. Which of the following summary recommendations is most appropriate based on the student's screening results? A. Interdental production of /s/ and /z/ sounds are within range of typical development. Continue to monitor and reassess in one year. B. Interdental production of /s/ and /z/ sounds are not within the range of typical development. An evaluation is recommended. C. Interdental production of the /s/ is within the range of typical development, while /z/ sound errors are not. RTI intervention is recommended. D. Correct production of /s/ and /z/sounds is emerging, and the student should be rescreened in three months.

A. Interdental production of /s/ and /z/ sounds are within range of typical development. Continue to monitor and reassess in one year.

The speech reception threshold (SRT) is a basic component of an evaluation of hearing function. Which of the following statements about the SRT is most accurate? A. It is measured in decibels and corresponds to the intensity level at which spondaic words can be recognized approximately 50% of the time. B. It makes use of test materials that are limited to monosyllabic words. C. It provides information on how well speech is understood at conversational levels. D. It is useful in validating acoustic intermittence measures.

A. It is measured in decibels and corresponds to the intensity level at which spondaic words can be recognized approximately 50% of the time.

According to empirical research with people who stutter, which TWO of the following are true regarding the age of symptom onset for most cases? A. It is usually earlier for girls than for boys. B. It is similar for girls and boys. C. It most often occurs in the range of 2 to 5 years old. D. It most often occurs in the range of 6 to 9 years old.

A. It is usually earlier for girls than for boys. C. It most often occurs in the range of 2 to 5 years old.

Fela is a third-grade student in a public school. She is a speaker of African American Vernacular English (AAVE) who has difficulty with the Standard American English (SAE) dialect used in her classroom. Her teacher believes that Fela's language skills are affecting her academic performance and has referred her to the school's speech-language pathologist. Which of the following is an appropriate rationale for providing language intervention for Fela? Select all that apply. A. It will likely foster better communication with Fela's linguistically and culturally diverse peers. B. It will likely improve Fela's code-switching ability with her teacher and other adult speakers of SAE. C. It may expand Fela's later academic and vocational opportunities. D. It will likely lead Fela to adopt SAE as her primary dialect.

A. It will likely foster better communication with Fela's linguistically and culturally diverse peers. B. It will likely improve Fela's code-switching ability with her teacher and other adult speakers of SAE. C. It may expand Fela's later academic and vocational opportunities.

After sustaining a CVA, Ms. Williams, age 75, was referred to an SLP for a speech and language evaluation. While Ms. Williams was describing the cookie-theft picture, the SLP observed that her grammatical structure appeared to be intact and her prosody was normal but that many of her sentences were meaningless, did not fit the context, and included nonsensical paraphasic errors. Additional testing also revealed that Ms. Williams exhibited poor repetition and naming skills, did not respond appropriately to many simple commands, and had difficulty reading. Ms. Williams appeared happy and talked excessively. She did not appear to be aware of her communication deficits. What is the most likely location of the lesion? A. Left posterior superior temporal gyrus B. Left inferior frontal gyrus C. Left superior frontal gyrus D. Left inferior parietal gyrus

A. Left posterior superior temporal gyrus

Which of the following will most effectively increase the fundamental frequency of the voice? A. Lengthening the vocal folds B. Decreasing the stiffness of the vocal folds C. Decreasing subglottal pressure D. Decreasing cricothyroid muscle activity

A. Lengthening the vocal folds

An SLP working with a 45-year-old patient with acquired expressive communication deficits is emphasizing reengagement by focusing on realistic short-term goals of the patient's choice. The SLP is primarily using which of the following approaches to therapy? A. Life Participation Approach to Aphasia (LPAA) B. Visual Action Therapy (VAT) C. Supported Communication Intervention (SCI) D. Prompts for Reconstructing Oral Muscular Phonetic Targets (PROMPT)

A. Life Participation Approach to Aphasia (LPAA)

Which THREE of the following variables are considered risk factors for late language emergence? A. Male gender B. Access to print material C. Low socioeconomic status D. Moderately low birth weight E. Exposure to rich and varied F. vocabulary

A. Male gender C. Low socioeconomic status D. Moderately low birth weight

An SLP plans cognitive treatment for a patient with right-hemisphere disorder due to a stroke. Which of the following patient characteristics is most likely to have an impact on the patient's engagement in treatment and outcomes? A. Moderate anosognosia for identified deficits B. Hemiparesis of the left arm and hand C. Mild word-finding difficulties D. Moderate expressive aprosodia

A. Moderate anosognosia for identified deficits

Which THREE of the following are included in a pragmatic language assessment during conversation? A. New topic initiation B. Speech intelligibility C. Discourse cohesion D. Repair strategies E. Type-token ratio

A. New topic initiation C. Discourse cohesion D. Repair strategies

Broca's aphasia is another term for which of the following: A. Nonfluent aphasia B. Fluent aphasia C. Conduction aphasia D. Global aphasia E. None of the above

A. Nonfluent aphasia

Which of the following is a typical symptom of cerebellar involvement? A. Overshooting or undershooting an intended target B. Rigidity during voluntary motions C. Spasticity during involuntary action D. Word-finding difficulty

A. Overshooting or undershooting an intended target

Which of the following factors contributes to UES opening? Select all that apply. A. Partial relaxation of the cricopharyngeal portion of the inferior constrictor muscle B. Superior and anterior hyolaryngeal excursion C. Velopharyngeal closure D. Posterior and inferior hyolaryngeal excursion

A. Partial relaxation of the cricopharyngeal portion of the inferior constrictor muscle B. Superior and anterior hyolaryngeal excursion

Which of the following statements best explains why thickened liquids for adult patients with dysphagia should be used with caution? A. Patients dislike thickened liquids and therefore do not drink enough, resulting in dehydration. B. The thickened liquid becomes thinner as it sits at the bedside, negating the liquid's benefit as a compensatory diet modification. C. Patients need twice as much thickened liquid because it provides half the hydration that thin liquid provides. D. Thickened liquids are aspirated more frequently than thin liquids.

A. Patients dislike thickened liquids and therefore do not drink enough, resulting in dehydration.

A 28-year-old female self-refers for a voice evaluation. She exhibits a variable dysphonia (it is present in some sentences but not in others) which presents as mild-to-moderate roughness. The SLP desires more objective data about the patient's voice quality. Based on the information obtained so far, which of the following assessments best complements the perceptual assessment to help the SLP determine the underlying physiological impairment(s)? A. Performing acoustic assessment B. Obtaining history of the problem C. Using the Consensus Auditory-Perceptual Evaluation of Voice (CAPE-V) D. Administering the Voice Handicap Index (VHI)

A. Performing acoustic assessment

A child that exhibits consistent, few errors and has only one or two sounds missing from his or her inventory would benefit from which treatment approach? A. Phonetic treatment approach B. Phonemic treatment approach C. Either D. Neither

A. Phonetic treatment approach

A client with anomic aphasia is a native speaker of Spanish with fair proficiency in English. Production of the word "shoes" as [tʃuz] on a repetition task is most likely due to which of the following factors? A. Phonological interference from the speaker's native language B. Semantic interference from the speaker's native language C. Semantic paraphasia due to the aphasia D. Phonemic paraphasia due to the aphasia

A. Phonological interference from the speaker's native language

Amplitude is perceptually related to the intensity of sound as frequency is related to the ____ of a sound. A. Pitch B. Period C. Elasticity D. Harmonic E. None of the above

A. Pitch

Which of the following muscles is primarily responsible for vocal fold abduction? A. Posterior cricoarytenoid B. Cricothyroid C. Interarytenoid D. Lateral cricoarytenoid

A. Posterior cricoarytenoid

Which of the following must a child first be able to do before the child can produce narratives? A. Produce several utterances on the same topic B. Use past and future tenses C. Express a sequence using "then" or "next" D. Produce compound sentences using "and"

A. Produce several utterances on the same topic

Children diagnosed as having specific language impairments are likely to exhibit the greatest deficits in which of the following? A. Production of sentences with appropriate inflectional morphology and syntax B. Acquisition of word meanings C. Comprehension of short sentences D. Motoric aspects of written expression

A. Production of sentences with appropriate inflectional morphology and syntax

An SLP in a public school will be seeing a student who is returning to school after experiencing a traumatic brain injury (TBI). The student has dysphagia, and the SLP will provide dysphagia therapy as part of the student's IEP. The clinician seeks evidence to help develop the intervention. Which of the following is the best resource? A. Reading a published case study of a 15 year old who received dysphagia therapy after a car accident B. Listening to the school principal's experience with another student who experienced a similar TBI C. Incorporating the parents' request that the student eat in the cafeteria with peers D. Reviewing treatment options presented in the newsletter of a dysphagia product vendor

A. Reading a published case study of a 15 year old who received dysphagia therapy after a car accident

Which of the following signs of dysphagia is the most common oral-stage observation when assessing a patient with Parkinson's Disease? A. Repeated nonpropulsive lingual movements B. Eating quickly and impulsively C. Spilling liquids out of mouth because of poor labial seal D. Holding food in mouth because of poor sensation

A. Repeated nonpropulsive lingual movements

A 65-year-old patient is transferred from another facility with a diagnosis of aphasia. The patient's symptoms, however, appear more consistent with apraxia. Which of the following tasks for the patient is most appropriate when assessing verbal apraxia of speech? A. Repeating words of increasing length B. Recalling three common items C. Performing voluntary oral movements D. Following two-step commands

A. Repeating words of increasing length

A clinician who employs active listening is doing which of the following? A. Responding to both the content and the affect of the client's remarks B. Listening very carefully and taking extensive notes C. Conducting a clinician-directed interview D. Directing the client to specific answers to questions

A. Responding to both the content and the affect of the client's remarks

Joe, an 18-year-old male, sustained a severe traumatic brain injury following a motorcycle accident. He was in a coma for approximately four weeks before being transferred to a rehabilitation program. Joe demonstrates severe attention deficits, anomia, disorientation, and poor short-term memory. Which of the following should be the initial target area in the treatment plan? A. Retraining attention as a discrete cognitive process B. Using declarative memory as an active learning strategy C. Using implicit memory as a passive learning strategy D. Reality orientation training

A. Retraining attention as a discrete cognitive process

Which cerebral hemisphere can we suspect was damaged if an individual is exhibiting left visual neglect, discourse and pragmatic deficits, impulsive behaviors and poor awareness? A. Right hemisphere B. Left hemisphere C. Brainstem damage D. A, B, & C

A. Right hemisphere

Mr. Greene recently sustained a cerebrovascular accident (CVAC V A), which impaired his communication. An SLP asked him to describe the illustration above. He said, "Well, it's a black-and-white drawing on heavy paper. There's a window with blinds and a driveway or sidewalk going off into the distance." Mr. Greene's response is most characteristic of which type of language disorder? A. Right-hemisphere language impairment B. Broca's aphasia C. Global aphasia D. Anomia

A. Right-hemisphere language impairment

An adult client exhibits visuospatial disorganization, an inability to initiate interactions, left-side neglect, and lack of facial expression. This combination of symptoms is most likely associated with which of the following? A. Right-hemisphere traumatic brain injury B. Left-hemisphere cerebrovascular accident C. Bilateral traumatic brain injury D. Alzheimer's dementia

A. Right-hemisphere traumatic brain injury

Which of the following statements about dysphagia screening and assessment/evaluation is true? A. Screening identifies the likelihood of dysphagia and the need for further assessment. B. Screening identifies the nature and severity of dysphagia and enables treatment planning. C. Assessment/evaluation is a pass-fail procedure that determines whether or not a patient is aspirating. D. Assessment/evaluation can be performed by non-SLP observers.

A. Screening identifies the likelihood of dysphagia and the need for further assessment.

Clinician: "What do you have there?" Child: "I had a big bus." Clinician: "Is that right?" Child: "Uh, I have a big bus." Clinician: "Now, is that right?" Child: "Yeah." What language-stimulation technique is the clinician using in the above exchange? A. Self-evaluation B. Rephrasing C. Reauditorization D. Parallel talking

A. Self-evaluation

A study is conducted to determine how well a dysphagia screening test predicts aspiration that is later confirmed with a videofluoroscopic swallow study (VFSS). The study results are presented in the table below. Aspiration No Aspiration Failed 45 20 Passed 5 30 Which TWO of the following statements about the results of a screening test are correct? A. Sensitivity is the proportion of people who fail the screening test who are aspirators B. Specificity is the proportion of people who pass the screening test who are aspirators C. Sensitivity is the proportion of people who pass the screening test who are not aspirators D. Specificity is the proportion of people who pass the screening test who are not aspirators

A. Sensitivity is the proportion of people who fail the screening test who are aspirators D. Specificity is the proportion of people who pass the screening test who are not aspirators

An SLP works with a patient who has severe dysphagia. The patient is on a pureed diet and honey-thick liquids. During treatment, the patient becomes tearful and expresses frustration. The patient says, "I feel like I'm not making any progress and I'm never going to eat normal food again. I hate this diet, and I hate doing therapy. I just want to give up." The SLP responds by saying, "What you're feeling is normal, and I understand. You're doing everything you can right now to get better, and giving up isn't the answer." The SLP reaches across the table and places a comforting hand on the patient's arm and has a caring facial expression. Which of the following principles of counseling is the SLP primarily demonstrating? A. Showing congruence B. Having unconditional positive regard C. Offering the patient sympathy D. Providing emotional support

A. Showing congruence

Which of the following is generally considered most effective and appropriate for viewing the vocal folds during phonation? A. Stroboscopy B. Endoscopy C. Fluoroscopy D. Laryngeal mirror examination

A. Stroboscopy

Which of the following sets of minimal pair words best targets the phonological patterns of stopping of fricatives? A. Sun and ton B. Sip and ship C. Star and tar D. Shoe and shoot

A. Sun and ton

Which of the following activities is a principal component of the Lidcombe Program for childhood stuttering? A. Teaching caregivers to provide feedback to their child about the child's fluent and stuttered speech B. Teaching caregivers to support their child's communication attempts but to avoid acknowledging the child's fluency performance C. Teaching children to describe their emotional state to their caregiver when stuttering is anticipated on an upcoming word D. Teaching children to present nonverbal cues to their caregivers when stuttering is anticipated on an upcoming word

A. Teaching caregivers to provide feedback to their child about the child's fluent and stuttered speech

Mr. Aviz, a 62-year-old chemist, is recovering from hip-replacement surgery following a spontaneous fracture. The surgery was completed without complications. He is now in his second postoperative day, and his wife reports alterations in his memory and some confusion not previously noted. Which of the following is the most reliable screening instrument for the SLP to use in this case? A. The Mini-Mental State Examination (MMSE) B. The Test of Problem Solving (TOPS-3) C. The Cognitive Linguistic Quick Test (CLQT) D. The Global Deterioration Scale (GDS)

A. The Mini-Mental State Examination (MMSE)

Linguistic approaches to the treatment of sound-production errors in children are based on the notion that the errors are systematic and rule-based and that the goal of treatment is to modify a child's rule system to approximate the rule system used by adults. Which of the following is a treatment objective that reflects a linguistic approach to treatment? A. The child will contrast alveolar stops with velar stops in meaningful word pairs. B. The child will produce voiceless alveolar sibilants correctly in unstructured conversation. C. The child will coarticulate stop plus liquid clusters as easily as other children of the same age. D. The child will recognize and identify phonetic distortions of his or her error sound.

A. The child will contrast alveolar stops with velar stops in meaningful word pairs.

Which of the following is the most reasonable standard to apply when judging whether a client has achieved generalization of a targeted skill? A. The client uses the targeted skill under stimulus conditions that were not present during the training process and without reinforcement. B. The client maintains the correct production of the targeted skill when the reinforcement schedule is changed. C. The client correctly produces the targeted skill effortlessly and without hesitation. D. The client is able to monitor errors and correct them with only a minimal number of cues from the clinician.

A. The client uses the targeted skill under stimulus conditions that were not present during the training process and without reinforcement.

A study is conducted to determine how well a dysphagia screening test predicts aspiration that is later confirmed with a videofluoroscopic swallow study (VFSS). The study results are presented in the table below. Aspiration No Aspiration Failed 45 20 Passed 5 30 Which THREE of the following statements about this dysphagia screening test are true? A. The screening test's sensitivity is 90% B. The screening test's sensitivity is 69% C. The screening test's specificity is 90% D. The screening test's specificity is 60% E. The number of false negatives was 5 F. The number of false negatives was 20

A. The screening test's sensitivity is 90% D. The screening test's specificity is 60% E. The number of false negatives was 5

A team of SLPs is evaluating whether a new language intervention is suitable for use with children who are on their caseload. The clinicians read an article documenting research on the effect of a novel treatment on the language performance of 30 children with specific language impairment. The researchers administered a standardized language test to the children before and after the treatment program. The children's posttreatment scores on the test were significantly higher than their pretreatment scores. This led the researchers to conclude that the treatment was highly effective. Based on the preceding description, which of the following appears to be a significant limitation of the design of the study? Select all that apply. A. The use of a single-group pretest-posttest design B. The absence of a control group C. The inability to rule out the possibility that the children's language scores improved for reasons other than the treatment D. The use of random assignment to place the children into the treatment group E. The number of participants in the study

A. The use of a single-group pretest-posttest design B. The absence of a control group C. The inability to rule out the possibility that the children's language scores improved for reasons other than the treatment

For which of the following reasons would the therapy for a client whose language loss is due to brain injury differ from the therapy for a client whose language loss is due to a progressive disorder? A. There is a much higher chance of spontaneous recovery for the client with brain injury. B. There is a much higher chance of spontaneous recovery for the client with a progressive disorder. C. The client with brain injury typically has problems with fatigue and mental confusion, whereas the client with a progressive disorder does not. D. The client with a progressive disorder typically has problems with attention and memory, whereas the client with brain injury typically does not.

A. There is a much higher chance of spontaneous recovery for the client with brain injury.

Which of the following indicates the goal of the Health Insurance Portability and Accountability Act (HIPAA) ? A. To ensure a patient's privacy and confidentiality of health-care information B. To maximize health insurance coverage for speech-language pathology services C. To ensure a free and appropriate education D. To ensure a patient's knowledge of health insurance

A. To ensure a patient's privacy and confidentiality of health-care information

An SLP receives a referral from an audiologist for a 72-year-old woman who demonstrates a moderate to severe bilateral sensorineural hearing loss preventing her from participating in social activities. Which of the following best identifies the SLP's role? A. Training in speech reading B. Prescribing hearing aids C. Performing an auditory evaluation D. Using auditory integration training

A. Training in speech reading

A child has been referred to an SLP for a comprehensive language evaluation, with reports of "global language problems." Following evaluation, the SLP determines that the child is demonstrating word-finding difficulties, comprehension deficits and disorganized, in addition to deficits in attention and executive functioning. Which of the following disorders does the child MOST LIKELY have? A. Traumatic Brain Injury B. Cerebral Palsy C. Autism Spectrum Disorder D. Down Syndrome

A. Traumatic Brain Injury

Which of the following benchmarks best aligns with current research on typical communication development for 3-year-old children? A. Understanding approximately 1,000 words B. Using irregular third-person-singular verb forms C. Having a mean length of utterance (MLU) in morphemes of six D. Producing approximately ten consonant phonemes accurately

A. Understanding approximately 1,000 words

An SLP who is treating an adolescent who stutters designs a treatment plan that includes three fluency management strategies: prolonged speech, cancellation, and pullout. Which of the following is true about the use of these treatment strategies? Select all that apply. A. Use of prolonged speech is likely to reduce the frequency of part-word repetitions and sound prolongations significantly. B. Each of the three strategies entails deliberate regulation of speech motor movements. C. The client will seek to apply cancellation whenever he feels anxious about the possibility of stuttering overtly. D. The client will seek to apply pullout during the course of part-word repetition or sound prolongation.

A. Use of prolonged speech is likely to reduce the frequency of part-word repetitions and sound prolongations significantly. B. Each of the three strategies entails deliberate regulation of speech motor movements. D. The client will seek to apply pullout during the course of part-word repetition or sound prolongation.

When conducting a phonetic analysis, a clinician looks to identify sounds that the child ___. A. Uses whether correct or incorrect B. Uses correct only C. Distorts D. None of the above

A. Uses whether correct or incorrect

Michael is a 32-month-old boy who has been receiving early intervention services over the past ten months for delayed speech and expressive-language development. Although his birth was reportedly unremarkable, Michael does have a history of recurrent otitis media with effusion. His parents described him as having been a "well-behaved and quiet baby." When Michael began receiving services, he communicated mainly through gestures and crude vocalizations. An open resting mouth position with slight tongue protrusion was sometimes noted. However, his receptive-language skills were found to be age appropriate and he showed no oral motor deficits during feeding. Michael's expressive-language skills have shown some progress since he began working with the speech-language pathologist, but he remains poorly intelligible. Michael's imitation of tongue, lip, and jaw movements is characterized by inconsistent groping and errors of sequencing not observed in his spontaneous oral movements. Michael has an age-appropriate vocabulary and produces utterances of up to five words. Articulation errors, especially metathesis of phones and syllables, increase as his utterance length increases. Michael's intelligibility is greatest at the single-word level. Automatic speech and highly familiar utterances are much more intelligible than his imitated productions. Given Michael's age, which of the following is likely to be the most effective strategy for speech and language intervention? A. Using structured play to address short-term goals B. Facilitating Michael's ability to self-monitor his speech C. Instructing Michael's parents in structured home exercises for their son D. Introducing augmentative and alternative communication devices

A. Using structured play to address short-term goals

Which of the following are the THREE types of velopharyngeal dysfunction? A. Velopharyngeal insufficiency B. Velopharyngeal mislearning C. Velopharyngeal mismatching D. Velopharyngeal ineffectiveness E. Velopharyngeal incompetence

A. Velopharyngeal insufficiency B. Velopharyngeal mislearning E. Velopharyngeal incompetence

Which of the following is a relaxation technique aimed to minimize tension of vocal hyper function and improve pitch? A. Yawn-sigh B. resistance training C. over-articulation D. Yoga

A. Yawn-sigh

An SLP receives a referral regarding a 4-year-old boy who uses two words spontaneously and functionally, who began walking at 3 years of age, and who responds to his name inconsistently. On the basis of the information alone, the SLP can legitimately conclude that the child's communication profile reflects A. a developmental delay B. autism spectrum disorder C. a chromosomal anomaly D. a metabolic disorder

A. a developmental delay

Before an SLP initiates voice intervention, it is most important that the SLP gather information about the: A. cause of the client's voice problem B. client's breathing patterns C. client's oral motor skills D. fundamental frequency of the client's laryngeal tone

A. cause of the client's voice problem

A 72-year-old male presents to a skilled nursing facility (SNF) following a complicated hospital stay. The patient is morbidly obese and has a history of COPD, depression, hypertension, diabetes, Bell's palsy, encephalopathy, and congestive heart failure. During his stay, he was not intubated but became very weak as a result of extended time in bed. In the hospital, he was placed on an NDD Level 2 diet with nectar-thick liquids due to aspiration of thin liquids and poor oral control of regular solids. The patient worked until he was 50 years old, but his health conditions led to his early retirement. Before his hospitalization, the patient did not leave his house very often, and his wife helped him with all ADLs. While evaluating the patient at the SNF, the SLP noticed that the patient was disoriented and confused, appeared to be hallucinating, grunted loudly when attempting to stand, and presented with aphonia. There was no indication of this condition in the patient's hospital chart, and the patient's wife stated that the patient had no trouble using his voice while in the hospital. An ENT consult stated there was no structural reason for the aphonia. Suspecting that the aphonia was of a psychogenic etiology, the SLP made a referral to the resident psychiatrist and began treatment for both the aphonia and for dysphagia. After a few sessions of speech therapy, the patient has made limited progress with his aphonia. The resident psychiatrist has continued to work with the patient and recommends cognitive behavioral therapy sessions and continued speech therapy. A primary factor causing the patient's psychogenic aphonia is the presence of A. depression B. Bell's palsy C. COPD D. hallucinations

A. depression

In the treatment of acute Wernicke's aphasia, the initial focus should be on: A. encouraging self-monitoring of the adequacy of verbal output B. increasing the rate of speech C. improving the client's ability to elaborate verbally D. increasing the complexity of sentence structures

A. encouraging self-monitoring of the adequacy of verbal output

Immediately following removal of a benign tumor from the base of the brain, a 76-year-old client exhibits severe nasalization and a weak, breathy voice. A four-month postsurgical assessment reveals no improvement. At this time, the remediation strategy for this client should focus on A. evaluation for prosthetic or surgical intervention B. strengthening exercises for the oral articulators C. a trial period using the yawn-sign technique D. complete vocal rest

A. evaluation for prosthetic or surgical intervention

In Bloom and Lahey's model, morphology is considered A. form B. content C. use D. semantics

A. form

To compensate for the effects of normal aging on cognitive functions, older adults will naturally demonstrate: A. greater amounts of bilateral activation of prefrontal brain regions B. significant reduction in the activation of the hippocampus C. increased activation in areas of the cortex responsible for visual-spatial processing D. an increase in semantic abilities that do not rely on long-term memory abilities

A. greater amounts of bilateral activation of prefrontal brain regions

The presence of excess air through the nasal cavity on oral consonants describes which type of resonance? A. hypernasality B. nasal emission C. Hyponasality D. Cul-de-sac resonance

A. hypernasality

Alicia is a fourth-grade student with a language impairment that interferes with her academic performance and social communication. On a standardized test of expressive language ability, she received an overall score of 97.5, placing her in the 65th percentile. Alicia's test result indicates that she A. performed as well as or better than 65% of the individuals on whom the norms were developed B. responded correctly to 65% of the 150 items included in the expressive-language test C. performed as well as or better than 65% of the other fourth graders who took the same test D. responded correctly to 97.5% of the test items, as did 65% of the other children taking the test

A. performed as well as or better than 65% of the individuals on whom the norms were developed

Which type(s) of analysis is used to assess all sounds that a child uses correctly in comparison to the adult model? A. phonemic B. phonetic C. phonological D. none of the above

A. phonemic

Native speakers of a language possess several capacities or abilities that provide insight into their language competence. Sentences such as "Visiting friends can be a nuisance" are especially useful to test a person's ability to A. recognize syntactic ambiguity B. interpret metaphoric language C. distinguish homophones by means of syntactic cues D. recognize a semantically anomalous sentence

A. recognize syntactic ambiguity

An SLP developed a new approach to the treatment of school-age children with specific language impairment. To determine whether this approach is effective, the language abilities of these children following treatment should be compared with the language abilities of A. similar children who did not receive treatment B. similar children who received a different treatment with known efficacy C. similar children who also received this treatment D. these children one year after the termination of treatment

A. similar children who did not receive treatment

The fluency principle that focuses on changing the way one stutters is best described as which of the following? A. stuttering modification B. Speech modification C. Reducing rate of speech D. using prompts to communicate E. None of the above

A. stuttering modification

An SLP is working with a 30-month-old child who is at the one-word production level. The goal of treatment is to facilitate growth in play as a precursor to the acquisition of two-word semantic relationships. To achieve the goal, the SLP can best begin by facilitating: A. the use of pretend play, such as brushing a doll's hair or feeding a doll with a bottle B. the purposeful exploration of toys C. exploration, by mouthing of toys D. mean-end behaviors, such as pulling a string to get a toy

A. the use of pretend play, such as brushing a doll's hair or feeding a doll with a bottle

A single exposure of several hours duration to continuous music with an overall level of 100 dB SPL will most likely produce A. tinnitus and a temporary threshold shift in high frequencies B. tinnitus and a distortion of speech perception C. a temporary threshold shift in the low frequencies D. a permanent threshold shift

A. tinnitus and a temporary threshold shift in high frequencies

An SLP provides home practice for a patient with aphasia as part of discharge plans. The patient is approximately six-months post left-hemisphere stroke that resulted in aphasia. Verbal output has improved significantly since the stroke; however, the patient is still very concerned that written expression is moderately impaired at the single-word level. The homework assignment is to implement copy-and-recall treatment to improve written expression, which involves practicing writing target words (from photographs) and then copying those words multiple times. However, during a follow-up phone call, the SLP determines that the patient is struggling to write the assigned target words because the patient "can't think of the correct letters." Although a model is provided on the back of each target photograph, the patient wants to find a way to practice without looking at the answer. The SLP's best course of action is to suggest that the patient: A. try anagramming the words using letter tiles to form the target word B. practice writing the alphabet in order multiple times before beginning the homework again C. use a cell phone voice-to-text application as a compensatory strategy D. use drawing to convey messages instead of using written expression

A. try anagramming the words using letter tiles to form the target word

A 9-month-old child was observed during a speech-language evaluation. To express herself, the child occasionally touched her mother, gained eye contact, and then gestured toward an object. If the child's development is normal, within the next month or so the child will begin to: A. use consistent sound and intonation patterns as signals for specific intentions B. reach for objects that she desires C. establish joint reference with her caretaker D. use recognizable words and phrases to express her intentions

A. use consistent sound and intonation patterns as signals for specific intentions

Language impairment in a child with Down syndrome is often determined by comparing performance on one or more standardized language tests with the child's mental age, rather than with the child's chronological age. Although mental age should not be used to specify the need for treatment, mental age can legitimately be used as a performance criterion because: A. using chronological age would overidentify language disorders B. using chronological age would underidentify language disorders C. mental age always correlates with verbal performance D. language performance is expected to exceed mental age

A. using chronological age would overidentify language disorders

An endoscopic examination of swallowing is the best instrumental assessment when the SLP wishes to evaluate the: A. volume of and patient's response to pharyngeal secretions B. duration and extent of hyolaryngeal elevation C. penetration of material into the laryngeal vestibule during the swallow D. efficiency of lingual manipulation during bolus formation and anterior-posterior bolus transit

A. volume of and patient's response to pharyngeal secretions

Which of the following instrumental assessment tools provides the most direct dynamic view of velopharyngeal movement during speech? A. Nasopharyngoscopy B. Lateral-view x-ray C. Nasometry measurements D. Aerodynamics

A.Nasopharyngoscopy

An octave is:

An indication of the interval between two frequencies

You have been asked to evaluate a client who has sustained a neurological insult. The chart tells you that his lateral cricoarytenoid is paralyzed in an abducted position. Upon your evaluation, it is likely that you will find ________.

Aphonic

______ is primarily a disorder of articulation and prosody

Apraxia of Speech

Broca's & Wernicke's Area are connected through the _______.

Arcuate Fasciculus

These are composed of a ring of connective tissue and muscle extending from the tips of the arytenoid cartilages to the larynx. They separate the laryngeal vestibule from the pharynx and help preserve the airway.

Aryepiglottic folds (FVF)

Place the names of the following theorists with their associated concepts. A. Piaget B. Skinner C. Vygotsky D. Chomsky 1. Discriminative Stimulus 2. Zone of Proximal Development 3. Language Generativity 4. Object Permanence

B(1), C(2), D(3), A(4)

Which cranial nerve is characterized as both motor and sensory and is responsible for the sensation of food residuals in the larynx, pharynx, and esophagus? A. Accessory B. Vagus C. Glossopharyngeal D. Facial

B. Vagus

In the word "make", the first vowel sound can be transcribed by which of the following diphthongs?: A. /aI/ B. /eI/ C. /au/ D. /eə/ E. None of the above

B. /eI/

Which of the following sounds are typically mastered by the time a child turns 3? A. /k/ and /g/ B. /p/ and /b/ C. /l/ and /s/ D. /ch/ and /sh/

B. /p/ and /b/

Identify the phoneme described based on the following distinctive features: +Voicing, +Labiodental, +Fricative A. /f/ B. /v/ C. /s/ D. A & B

B. /v/

Which of the following list of frequencies are tested during pure tone audiometry? A. 50, 100, 200, 400, 600, 800 B. 250, 500, 1000, 2000, 4000, 8000 C. 100, 500, 800, 1000, 3000 D. 250, 400, 800, 1000, 2000, 6000 E. none of the above

B. 250, 500, 1000, 2000, 4000, 8000

At what ages does treatment for stuttering shift to management of symptoms? A. 3 years B. 7 years C. 18 years D. Treatment should always involve management of symptoms

B. 7 years

When counseling the parents of a child who has an articulation disorder, the SLP can cite developmental norms to show which of the following? A. The child's misarticulation will interfere with reading skills. B. A certain percentage of children of a certain age can correctly articulate the misarticulated sound. C. The misarticulation is caused by faulty learning. D. The misarticulation is not physically based.

B. A certain percentage of children of a certain age can correctly articulate the misarticulated sound.

An elderly patient with multiple diagnoses including Parkinson's disease (PD) presents to a home health SLP for treatment. The SLP recommends that the patient follow a mechanical soft diet with thin liquids, alternate bites and sips, take small bites, and have supervision when eating. Throughout the course of treatment, the patient exhibits a 12 percent weight loss with variable intake levels. To which of the following professionals is it most appropriate for the SLP to refer the patient? A. A gastroenterologist, for possible feeding tube placement B. A dietitian, for nutrition management C. An ENT specialist, for possible esophageal deficits D. A neurologist, for assessment of progression of PD

B. A dietitian, for nutrition management

Which of the following provides the most important diagnostic information to an SLP making a differential diagnosis between childhood apraxia of speech and flaccid dysarthria in a child? A. The child's articulation performance at the sentence level B. A history of the child's development of chewing, eating, and swallowing C. A history of the child's language development D. The child's willingness to function in sociocommunicative events

B. A history of the child's development of chewing, eating, and swallowing

For which individual would a recommendation for an augmentative and alternative communication (AAC) intervention be least appropriate? A. A preschool child with a language-learning disorder and highly unintelligible speech B. A teenager with a repaired cleft palate who continues to experience hypernasality C. A young adult with severe cerebral palsy precluding functional oral communication D. A 55-year-old man who has had a laryngectomy

B. A teenager with a repaired cleft palate who continues to experience hypernasality

Following placement of a tracheostomy tube, a patient recovering from cardiothoracic surgery is successfully weaned from mechanical ventilation. A day later the SLP receives a consult to assess patient candidacy for using a one-way tracheostomy valve. Which of the following observations is the most important contraindication for safe and successful patient tolerance of the one-way valve? A. Oxygen saturation below 95% B. Ability of patient to pass air to the oral cavity while exhaling with the tube cannula occluded by the SLP C. Patient ability to expectorate lung secretions without suctioning D. Patient tolerance of the deflated cuff

B. Ability of patient to pass air to the oral cavity while exhaling with the tube cannula occluded by the SLP

Which cranial nerve controls movement of certain neck muscles and innervates the sternocleidomastoid muscle? A. Vagus nerve B. Accessory nerve C. Facial nerve D. Trigeminal nerve E. A and B

B. Accessory nerve

Which of the following conditions is primarily associated with a short lingual frenulum, heart-shaped tongue tip, and a failure to elevate the tongue tip beyond the mandibular incisors, as revealed during oral-facial examination? A. Bulbar palsy B. Ankyloglossia C. Glossoptosis D. Congenital lip pits

B. Ankyloglossia

An SLP conducts a videofluoroscopic swallowing study (VFSS) for a patient referred for suspected dysphagia secondary to stroke. The SLP finds that the patient has dysphagia characterized by delayed onset of the pharyngeal response, limited lingual retraction, limited oropharyngeal propulsion, and the need to swallow two or more times to clear boluses. Some aspiration of hypopharyngeal residue occurs after the swallow from the pyriform sinuses. During the VFSS, several compensatory maneuvers (chin-down posture) were attempted to test their efficacy. The SLP is developing a treatment plan to eliminate aspiration and to improve lingual strength, retraction, and oropharyngeal propulsion through a combined strategy using compensatory and restorative maneuvers, including a progressive resistive exercise program of isometric tongue-press exercises using an instrument that measures the pressure of tongue contact with the hard palate (IOPI or SwallowStrong). The SLP measures the patient's baseline maximum tongue-press pressure generation with the instrument and begins treatment. When the patient attempted the chin-down posture during the VFSS, which of the following observations was the SLP most likely to make? A. Aspiration from the pyriform sinuses was eliminated. B. Aspiration from the pyriform sinuses was not eliminated. C. The space between the tongue base and posterior pharyngeal wall increased. D. The vallecular space became more narrow, preventing aspiration.

B. Aspiration from the pyriform sinuses was not eliminated.

Which type of fibers connect regions of the cerebral cortex within each hemisphere? A. Projection fibers B. Association Fibers C. Commissural Fibers D. Seperation Fibers E. All of the Above

B. Association Fibers

During an evaluation, a 2½‑year‑old boy exhibits a lack of social reciprocity, an averted eye gaze, and repetitive motor mannerisms. The child's behavior is most typical of which of the following? A. Social‑pragmatic disorder B. Autism spectrum disorder C. Generalized developmental delay D. Rett syndrome

B. Autism spectrum disorder

which language theory is based upon the belief that children learn language through conditioning? A. Semantic Theory B. Behavioral Theory C. Social Interactionism D. Cognitive Theory

B. Behavioral Theory

A 59-year-old male patient with laryngeal cancer presents to an SLP before a laryngectomy. The SLP completes the evaluation and takes time to counsel the patient regarding what to expect during and after the procedure. Which of the following reasons best identifies the importance of a patient meeting with an SLP prior to surgery? A. Exploring alternatives to surgical intervention B. Being educated regarding artificial forms of speech C. Completing therapeutic exercises before the procedure D. Gaining a better understanding of the prognosis

B. Being educated regarding artificial forms of speech

During an examination of his oral mechanism, an adult male is asked to pucker his lips and then to spread them into a wide smile. This ability depends on the bilateral integrity of which of the following cranial nerves? A. CN V B. CN VII C. CN IX D. CN XII

B. CN VII

An SLP collects a play-based language sample from a 3-year-old patient, who uses mostly single words with a few two-word combinations. Which of the following analyses should the SLP plan to apply to obtain the most helpful and appropriate information when forming treatment goals from the sample? A. Analyzing the percentage of correct consonants B. Calculating the mean length of utterance in morphemes C. Listing narrative cohesion markers used by the patient D. Identifying types of "wh" questions that the patient uses

B. Calculating the mean length of utterance in morphemes

Cognitive therapy for stuttering focuses on which of the following? A. Extinguishing the overt, dysfluent speech behavior by withholding positive reinforcement B. Changing distorted beliefs about self-efficacy and the need to speak with complete fluency C. Providing positive reinforcement during periods of fluent speech D. Reducing dysfluent speech behavior by using visual imaging

B. Changing distorted beliefs about self-efficacy and the need to speak with complete fluency

A 12-year-old phonology patient responds very well to continuous verbal reinforcement. The SLP wishes to increase the task level for sibilant production to conversational speech, but is concerned that the accuracy of the patient's productions will diminish without continuous reinforcement. Which of the following adjustments is most logical for the patient's treatment plan? A. Increase task level to conversation and continue with continuous reinforcement for correct productions B. Changing the reinforcement schedule to variable-interval C. Discontinuing reinforcement and providing a performance summary at the end of the conversation D. Shifting the feedback to punishment

B. Changing the reinforcement schedule to variable-interval

Which dental condition is most likely to have a negative effect on articulation? A. Class II malocclusion B. Class III malocclusion C. Missing maxillary lateral incisors D. Missing mandibular central incisors

B. Class III malocclusion

After conducting a language sample on a conversational language user, what pragmatic component may be used to evaluate complete ties? A. Conversational repairs B. Cohesive adequacy C. Eye contact D. Attention E. All of the above

B. Cohesive adequacy

Which of the following strategies best demonstrates an SLP working as a Response to Intervention (RTI) team member to provide a student Tier 2 language supports? A. Assisting the teacher with Individualized Education Program referral paperwork B. Collaborating with the teacher to provide small-group instruction C. Enrolling the student in language therapy to address language needs D. Evaluating the student's speech and language skills

B. Collaborating with the teacher to provide small-group instruction

Alan, a 62-year-old, right-handed African American male, sustained a traumatic brain injury mostly affecting his right hemisphere and bi-lateral frontal lobes. Alan was hospitalized for 24 days before being discharged to an inpatient brain injury rehabilitation program. His spouse attends his initial evaluation sessions. The SLP reports impairments in memory, poor awareness of deficits, and some changes in his communication skills. The SLP already has current information about Alan's performance on a standardized language battery suggesting minimal impairments. Therefore, the SLP completes an evaluation of Alan's functional communication skills. The SLP's assessment involves testing his functional use of humor, facial expressions, nonverbal communication strategies, and understanding functional written materials. The SLP hopes to use this information to determine the impact of Alan's impairments on his daily life. The SLP determines that Alan would benefit from an external memory aid, specifically a memory notebook. The SLP wants to be sure that Alan can learn to use the aid but is concerned that because of his memory impairment, he will struggle to retain the basic information about how to use the aid. The SLP discourages Alan from guessing and intervenes with support before Alan can make a mistake when using the device. Which of the following tools is most appropriate to evaluate Alan's functional communication skills? A. Western Aphasia Battery-Revised B. Communication Activities of Daily Living-Second Edition C. Boston Diagnostic Aphasia Examination-Third Edition D. Test of Everyday Attention

B. Communication Activities of Daily Living-Second Edition

Which of the following is most important for an SLP to do when assessing a child who has an acquired brain injury? A. Evaluate pragmatics through a structured language test B. Compare premorbid performance with present performance C. Ensure administration of an intelligence test D. Compare nonverbal performance with verbal performance

B. Compare premorbid performance with present performance

The nurse reports coughing, wet vocal quality, and shortness of breath on your new patient that experienced a CVA last week. Which of the following is the best plan? A. Refer for instrumental test prior to seeing the patient B. Complete bedside exam first, then refer for instrumental testing if appropriate C. Recommend NPO

B. Complete bedside exam first, then refer for instrumental testing if appropriate

As part of a study, the speech of several subjects with Wilson's disease is evaluated using an acoustic analysis system. The same subjects are assessed again, but with a different system from another manufacturer. Which of the following can be assessed by comparing the two data sets? A. Content validity B. Concurrent validity C. Test-retest reliability D. Interjudge reliability

B. Concurrent validity

A patient is diagnosed with a cerebrovascular accident of the temporal lobe. The patient exhibits deficits in repetition, written expression, and auditory comprehension. In addition, literal paraphasias are noted. These findings are consistent with what type of aphasia? A. Broca's B. Conduction C. Anomic D. Global

B. Conduction

An SLP just completed an aphasia evaluation with an aphasia battery. The results of the battery show that the patient with aphasia produces very fluent speech but makes many phonemic errors. When producing phonemic errors, the patient stops to try and get the word right with multiple attempts (e.g.for example., for the word screwdriver, the patient said "Cewdriver, dewdiver, screwdriver."). The patient also had relatively good comprehension but a pronounced repetition impairment. The patient's results are most consistent with which of the following type of aphasia? A. Broca's aphasia B. Conduction aphasia C. Wernicke's aphasia D. Anomic aphasia

B. Conduction aphasia

Which of the following errors is likely to persist the longest in the speech of children who are learning Standard American English (SAE) as a first language and are following the normal developmental course for speech and language acquisition? A. Assimilation B. Consonant cluster reduction C. Final-consonant deletion D. Velar fronting

B. Consonant cluster reduction

For a patient with potential pharyngeal phase dysphagia and pooling of secretions following open-heart surgery with suspected recurrent laryngeal nerve damage, which of the following instrumental examinations for swallowing provides the most direct view for evaluating the patient based on history and suspected difficulties? A. Upper gastrointestinal (GI) series B. Fiberoptic endoscopic evaluation of swallowing (FEES) C. Videofluoroscopic swallowing study (VFSS) D. Videostroboscopic examination of vocal folds

B. Fiberoptic endoscopic evaluation of swallowing (FEES)

A child with developmental apraxia of speech is not making progress in a school setting. The parents wish to schedule additional therapy. An SLP in a local clinic evaluates and determines that a more individualized treatment plan is warranted. Which of the following actions is most appropriate for the SLP at the local clinic to take to ensure the child receives necessary treatment? A. Recommending the parents stop school treatment given the lack of progress B. Contacting the child's school SLP to coordinate treatment plans C. Encouraging private SLP treatment using a smaller group setting D. Suggesting additional treatment at the local SLP clinic using the same tasks used by the school SLP

B. Contacting the child's school SLP to coordinate treatment plans

An SLP designs a study to examine the effect of a new therapy technique in comparison to the effect of the current standard approach. Patients are recruited and randomly assigned to either a control group or an experimental group. The outcomes of each group will then be compared. Which of the following best describes the SLP's research project? A. Cohort study B. Controlled trial C. Cross-sectional study D. Case-Control study

B. Controlled trial

A 7-month-old infant presents to an SLP for a swallow evaluation prompted by parental concerns related to the child coughing and choking frequently when swallowing thin liquids. The parents have attempted to feed the child small amounts of pureed foods, but the child spits them out. Which of the following developmental milestones is a typically developing 7 month old most likely to demonstrate during a swallow evaluation? A. Showing the ability to manipulate food B. Creating vertical jaw movements C. Using the rooting reflex when food is seen D. Engaging tongue elevation patterns

B. Creating vertical jaw movements

Mr. Lewis, a fourth-grade teacher, has referred a 9-year-old student to the school speech-language pathologist (SLP) because he is concerned that the student's speech is interfering with spelling performance. The SLP observes that the student substitutes the voiceless labiodental fricative for the voiceless interdental fricative in the word-final position and that the student reverses /s/ and /k/ in words such as "ask." Which of the following areas needs to be considered first? A. Auditory discrimination B. Dialectal differences C. Disordered phonological system D. Dyslexia

B. Dialectal differences

A 9-year-old child is one year post tonsillectomy and adenoidectomy and presents with moderate hypernasality and consistent nasal emission. The child was referred for assessment, and velopharyngeal incompetency was identified. Which of the following would be the next appropriate step for the SLP to take? A. Continue to monitor resonance as the child still may spontaneously improve B. Discuss surgical management options with the craniofacial team C. Initiate therapy to discriminate between hypernasal speech and oral speech D. Slow down rate of speech to help velopharyngeal closure

B. Discuss surgical management options with the craniofacial team

An SLP assesses a 3-year-old child with poor intelligibility. The SLP attempts to determine whether the child has a severe phonological disorder or childhood apraxia of speech (CAS). Which TWO of the following characteristics are primarily indicative of CAS? A. Presenting with decreased strength and muscle coordination B. Displaying inconsistencies in articulation performance C. Making the same errors independent of length of speech utterance D. Speaking with a disrupted rate, rhythm, and stress of speech

B. Displaying inconsistencies in articulation performance D. Speaking with a disrupted rate, rhythm, and stress of speech

___ Syndrome occurs when there is an additional chromosome. Characteristic features include hypotonia, slanted eyes, complex cardiac hx, and language/articulation deficits. A. Noonan syndrome B. Down syndrome C. CHARGE D. Apert syndrome

B. Down syndrome

A patient is referred to speech therapy after a total laryngectomy. The SLP teaches the patient to speak by taking air in through the mouth, trapping it in the throat, and then releasing it. Which of the following treatments is the SLP primarily implementing with the patient? A. Tracheoesophageal puncture B. Esophageal speech C. Neck electrolarynx D. Intraoral artificial larynx

B. Esophageal speech

What type of muscle is the palatoglossal muscle? A. intrinsic tongue muscle B. Extrinsic tongue muscle C. Laryngeal muscle D. Tracheoesophageal muscle E. none of the above F. All of the above

B. Extrinsic tongue muscle

Which of the following is a distinguishing characteristic of articulation therapy for a 2-year-old patient with a repaired cleft palate and compensatory errors? A. Cycling through multiple speech goals in a given session B. Having a play-based learning approach to address goals C. Focusing on carryover of speech productions D. Addressing speech targets with high frequency and intensity

B. Having a play-based learning approach to address goals

A 70-year-old female has dysphagia characterized by poor posterior oral containment of the bolus during the oral preparatory stage, causing aspiration before the swallow. Cognition and the pharyngeal stage of the swallow are intact. Which of the following is the most appropriate treatment approach for the client? A. Providing a puree diet with thickened liquids B. Having the client flex her head forward (perform the chin-down posture) during oral preparation and transit stages of the swallow C. Having the client turn (rotate) her head to the right when swallowing D. Providing a diet that consists of thin liquids

B. Having the client flex her head forward (perform the chin-down posture) during oral preparation and transit stages of the swallow

Miss. Malone, a 65-year-old retired attorney, presents with severe nonfluent aphasia secondary to a stroke that occurred more than a year ago. Speech-language intervention helped her regain the ability to answer simple questions but not initiate speech. Citing Miss. Malone's frustration at her limitations, her family requests that she be evaluated for an augmentative and alternative communication (AAC) device. During Miss. Malone's assessment, the SLP determines that she could potentially benefit from AAC services and initiates discussion of intervention goals. Once goals are established and Miss. Malone has her device, she begins to learn how to operate her device to produce common spoken messages. Over the course of intervention, Miss. Malone is ultimately able to produce novel sentences. Regarding Miss. Malone's proposed AAC assessment, which of the following statements is true? A. Her choice of devices will be severely limited by her difficulties with speech onset. B. Her family should be involved in the AAC assessment as well as any subsequent therapy. C. Given her advanced level of education, only high-tech devices should be considered. D. Given the severity of her impairment, only low-tech devices should be considered.

B. Her family should be involved in the AAC assessment as well as any subsequent therapy.

An SLP evaluates the speech of a child with suspected velopharyngeal dysfunction. Loading sentences with which of the following types of stimuli would be most helpful to include in the speech evaluation? A. Nasal phonemes B. High-pressure oral stops and fricatives C. Sustained /α/ sounds D. Liquids and glides

B. High-pressure oral stops and fricatives

According to current research, which of the following is most contributing to a rise in oropharyngeal cancers in the United States? A. Cigarettes B. Human papillomavirus (HPV) C. Alcohol D. Smokeless tobacco

B. Human papillomavirus (HPV)

A speech-and-hearing clinic has recently opened, but referrals are slow in coming. Which of the following would be most appropriate and effective for the clinic's director to do first? A. Ask local hospitals to provide names of recent clients likely to need speech-language services B. Identify and define the major consumer groups and referral sources, and develop a plan to reach them C. Identify the weaknesses in the competition, and inform consumer groups and referral sources of the weaknesses D. Wait for demand to increase on its own, because marketing speech-language services is against the ASHA Code of Ethics

B. Identify and define the major consumer groups and referral sources, and develop a plan to reach them

A 60-year-old male is referred to the SLP in an acute rehabilitation center for a videofluoroscopic swallowing study (VFSS) following a first-time stroke: a lateral medullary stroke damaging the right nucleus ambiguus and other centers. The clinical examination revealed ptosis of the left eye, soft palate elevation to the right of midline, symmetrical lingual protrusion, and a breathy dysphonia with precise articulation of all consonants. The patient is unable to swallow his saliva and coughs when he attempts to do so. Based on the stroke type and clinical examination observations, which THREE of the following swallow impairments would the SLP likely observe on the VFSS? A. Incomplete clearance of material from the oral cavity B. Impaired or absent opening of the upper esophageal sphincter C. Anterior loss of bolus contents during the oral preparatory stage D. Nasopharyngeal regurgitation during the pharyngeal stage E. Adequate excursion of the hyolaryngeal complex F. Unilateral pharyngeal paralysis

B. Impaired or absent opening of the upper esophageal sphincter D. Nasopharyngeal regurgitation during the pharyngeal stage F. Unilateral pharyngeal paralysis

An influenza epidemic that began last month may cause a spike in new cases. The occurrence of new cases of a new disease in a population over a specific period of time refers to ____. A. Prevalance B. Incidence C. Both D. Neither

B. Incidence

An SLP has been working on narrative structure with a middle school student on the SLP's caseload. Recently, the child wrote a story about her weekend, which reads "My family was going to a friends birthday party. We were late to the birthday party." Using this information, the SLP would like to assign a narrative episode level to this child's story. Which of the following narrative levels BEST describes the child's story. A. Abbreviated episode B. Incomplete Episode

B. Incomplete Episode

Establishment of which of the following is most important in ensuring that the results of any diagnostic test of speech or language are replicable? A. Content validity B. Interjudge reliability C. Split-half reliability D. Face validity

B. Interjudge reliability

Which of the following is an accurate statement about whispered speech? A. It is produced by approximating the arytenoid cartilages so that their medial surfaces are in direct contact. B. It is composed largely of aperiodic sounds. C. Spectrographic analysis of it reveals no discernible formants for the vowels. D. Most people can produce longer utterances per breath using it than they can using conventional phonation.

B. It is composed largely of aperiodic sounds.

The figure above shows the oral, pharyngeal, and esophageal structures involved in swallowing. Which of the following best describes the transit of the bolus at the moment depicted in the figure? A. The bolus is traveling through the esophagus. B. The bolus is being propelled from the oral cavity by the tongue and has entered the pharynx. C. The bolus has passively exited the oral cavity without propulsion by the tongue. D. Laryngeal penetration of some of the bolus is evident.

B. The bolus is being propelled from the oral cavity by the tongue and has entered the pharynx.

Brennan, a 50-year-old senior corporate executive, is referred to an SLP with a diagnosis of chronic traumatic encephalopathy (CTE). His presenting complaints are increasing forgetfulness, difficulties learning new material, and managing money. Onset of these problems was gradual, beginning an undetermined number of years prior to the evaluation. Brennan is a college graduate who attended school on a football scholarship, after which he played professionally for two years. Although a cooperative patient, Brennan's initial assessment was incomplete due in largely to his slow responses to stimuli. Still, mild deficits in problem solving, short-term memory, and deductive reasoning were noted. Further assessment is recommended, to be followed by treatment. After hearing the SLP's recommendation, Brennan's wife is concerned that Brennan will soon be unable to care for himself. She stated she is willing to bring him back as often as necessary in hopes of improving his condition. Which of the following is the most appropriate response the SLP can give Brennan's wife to address her concerns? A. Brennan's cognitive functioning will continue to deteriorate, but the process is gradual and any severe deficits are unlikely to manifest for at least twenty years. B. It is difficult to predict the pattern of Brennan's deteriorating cognitive function because the progression of CTE varies from person to person. C. There is doubt that Brennan has CTE, given that he played professional football for only a short period. D. The progression of CTE is reversible if Brennan undergoes therapy during its initial phase.

B. It is difficult to predict the pattern of Brennan's deteriorating cognitive function because the progression of CTE varies from person to person.

____ is a perturbation measure of frequency instability. A. Shimmer B. Jitter C. Harmonics D. Fundamental E. None of the Above F. A and B

B. Jitter

A 42-year-old male teacher is referred for a voice evaluation. History and perceptual voice assessments reveal an eight-month history of progressive dysphonia, which is currently characterized by a rough and breathy voice. Acoustic and aerodynamic assessments reveal aperiodic voice signal, reduced frequency range, increased subglottal air pressure, and increased transglottal airflow. The patient complains of voice fatigue at the end of the day and pain during phonation. The patient reports moderate alcohol use but is not currently a smoker. He has no previous history of chronic voice problems, surgery, or neurological disease. Based on the patient's history, which of the following assessments will best allow the SLP to assess vocal fold vibratory dynamics during phonation? A. Videofluoroscopic assessment B. Laryngeal videostroboscopy C. Nasoendoscopy D. Ultrasound

B. Laryngeal videostroboscopy

An evaluation is performed on a 5 year old with frequent hoarseness. Which of the following measures is primarily included in the endoscopic examination? A. Amplitude perturbation B. Laryngeal visualization C. Laryngeal airway resistance D. Perceptual judgment

B. Laryngeal visualization

Which of the following views make up a standard videofluoroscopic swallow study? Select all that apply. A. Frontal B. Lateral C. Transverse D. Anterior-posterior

B. Lateral D. Anterior-posterior

A 90-year-old female patient residing in a long-term care facility is referred to the facility SLP for a swallow evaluation. The patient has advanced dementia, is under hospice care, and is currently on a pureed diet with nectar-thick liquids. During the evaluation, the SLP finds that the patient is showing signs and symptoms of aspiration on all consistencies, including pudding-thick liquids. The patient's cognitive condition makes an instrumental exam inappropriate. The SLP's recommendation is nothing by mouth (NPON P O) except for pleasure. The family and patient have decided against feeding-tube placement. Which of the following is a primary ethical responsibility of the SLP in this situation? A. Asking the family what diet level they would like the patient to consume B. Making a recommendation based on clinical judgment C. Deferring to a doctor or nurse to help with the decision D. Refusing to write diet orders for the patient because of the NPO recommendation

B. Making a recommendation based on clinical judgment

A client has been determined to have poor upper esophageal sphincter opening secondary to decreased hyolaryngeal excursion, following a lateral medullary stroke. Cognitive functions are within normal limits. Which of the following interventions would be the most appropriate recommendation for this client as an initial course of treatment for the underlying disorder? A. Thermotactile stimulation B. Mendelsohn maneuver C. Thickened liquids D. Chin-down posture (head/neck flexion)

B. Mendelsohn maneuver

During a clinical swallow assessment by an SLP, a patient was noted to have the following symptoms. - An immediate cough after swallowing thin liquids - Prolonged mastication with regular-consistency foods - No coughing when fed thickened liquids - Postswallow residue Which of the following recommendations is most appropriate for the patient in this situation? A. Neuromuscular electrical stimulation B. Modified barium swallow study C. High-resolution pharyngeal manometry D. Cervical auscultation of swallowing

B. Modified barium swallow study

A speech language pathologist evaluated Sophia Allen, a 6-year-old girl. The SLP completed a case history with Sophia's mother and conducted a comprehensive speech and language assessment. During the case study the SLP learned that Sophia has had a normal developmental history and is currently healthy with no known neurological deficits. Sophia's mother first became concerned when Sophia could not describe her school day. Sophia's sentences lacked detail or were composed of very basic words and consisted of simple sentence structures. Sophia's first-grade teacher reported that her speech sounds immature compared to that of her classmates and that she frequently does not follow directions. During the assessment Sophia passed the oral mechanism examination and hearing screening. Sophia received a standard score of 99, placing her in the 47th percentile on a commonly used receptive vocabulary test. To obtain the most valuable information related to Sophia's future reading skills, the SLP should assess which of the following? A. Receptive vocabulary B. Morphological awareness C. Articulation skills D. Comprehension of one-step directions

B. Morphological awareness

Matthew presents with significant grammatical errors such as incorrect use of prefixes, suffixes, and pronoun errors. These are common signs of a language disorder of which language component? A. Pragmatics B. Morphology C. Semantics D. both A and C

B. Morphology

Successful use of an alternative and augmentative communication system is based on such factors as selecting appropriate vocabulary, seating and positioning, and having a reliable method of controlling the system. To facilitate the most effective use of the system, clinicians most often advocate which of the following approaches? A. Unimodal B. Multimodal C. Bimodal D. Gestural

B. Multimodal

Which language theory is supported by the following evidence?: deaf babies babble, grammar rules are not directly taught, and children follow a sequence of developmental milestones. A. Cognitive theory B. Nativist/Generative view C. Constructionist/interactionist view D. Behavioral theory

B. Nativist/ Generative view

Obligatory errors that result from velopharyngeal insufficiency may benefit most from which of the following treatment plans? A. Articulation therapy before surgery B. Need surgery to correct form first before before speech therapy C. Phonemic-based treatment D. All of the above

B. Need surgery to correct form first before before speech therapy

Which of the following is the best action to take initially with a client who presents with poor oral control of liquids and solids, coughing and choking while eating and drinking, and a history of hospitalizations associated with pneumonia? A. Thickening liquids so that the client will be better able to control oral movements for swallowing B. Obtaining a modified barium-swallow study to determine appropriate interventions C. Evaluating the client's ability to eat a variety of foods in order to determine which foods are safest D. Prescribing that the client be NPO, since aspiration is present

B. Obtaining a modified barium-swallow study to determine appropriate interventions

What does the letter 'O' in SODA errors of articulation stand for? A. Ordinary errors B. Omission errors C. Occlusion errors D. Occasional errors E. None of the above

B. Omission errors

An 82-year-old female patient is admitted to a skilled nursing facility following an acute hospital stay. Her diagnoses include urinary tract infection, frequent falls with subsequent hip fracture, and chronic obstructive pulmonary disease (COPD). No surgery was required for the hip fracture. Before admission to the hospital, she was living independently and able to complete all activities of daily living (ADLs) and instrumental activities of daily living (IADLs) on her own. She is referred to the facility SLP for cognitive screening because she has difficulty carrying over new information, confusion regarding weight-bearing status, and difficulty processing directions. The SLP administers the Montreal Cognitive Assessment, and the patient scores 17/30. The patient's main deficits are in the areas of short-term memory, executive functioning, and planning. After the patient is appropriately treated both medically and therapeutically, cognitive deficits remain and the interdisciplinary team does not recommend that the patient return home independently. The patient's family is frustrated with this recommendation and seeks input from other sources. Which of the following diagnoses is most appropriate for this patient? A. Cognitive communication deficit B. Other symbolic dysfunction C. Age-related cognitive decline D. Mild cognitive impairment

B. Other symbolic dysfunction

Which of the following is an example of a speech treatment technique that over emphasizes articulatory movements to improve intelligibility? A. Vocal function exercises B. Over-articulation C. Bearing down D. Sustained vowel prolongation

B. Over-articulation

Which of the following should be the primary focus of early language intervention for at-risk infants? A. Establishing object permanence through play activities B. Training primary caregivers to facilitate language learning C. Creating readiness activities in the context of play D. Enhancing social communication through play activities

B. Training primary caregivers to facilitate language learning

Brennan, a 50-year-old senior corporate executive, is referred to an SLP with a diagnosis of chronic traumatic encephalopathy (CTE). His presenting complaints are increasing forgetfulness, difficulties learning new material, and managing money. Onset of these problems was gradual, beginning an undetermined number of years prior to the evaluation. Brennan is a college graduate who attended school on a football scholarship, after which he played professionally for two years. Although a cooperative patient, Brennan's initial assessment was incomplete due in largely to his slow responses to stimuli. Still, mild deficits in problem solving, short-term memory, and deductive reasoning were noted. Further assessment is recommended, to be followed by treatment. After hearing the SLP's recommendation, Brennan's wife is concerned that Brennan will soon be unable to care for himself. She stated she is willing to bring him back as often as necessary in hopes of improving his condition. Based on the long-term prognosis for CTE, which of the following interventions by the SLP is most likely to benefit Brennan? A. Word retrieval strategies B. Patient/family counseling C. Executive function retraining D. Spaced retrieval training

B. Patient/family counseling

The commonly used chin-down posture was initially developed to eliminate thin-liquid aspiration in people with delayed pharyngeal stage onset after having a stroke, and when its efficacy was investigated, it was found to be 50% effective. For whom is the chin-down posture ineffective at eliminating thin-liquid aspiration? A. People who have cricopharyngeal dysfunction B. People who aspirate residue from the pyriform sinuses C. People who have impaired lingual function D. People who aspirate residue from the valleculae

B. People who aspirate residue from the pyriform sinuses

Fiber-optic instrumentation is appropriate for evaluation of which of the following types of disorders? A. Articulation B. Phonation C. Proprioception D. Respiration

B. Phonation

Management of which one of the following examination observations alone will produce the best improvement in the pneumonia risk of an adult patient with chronic dysphagia? A. Poor oral bolus control and containment B. Poor oral hygiene and dependence for oral care C. Delayed onset of the pharyngeal stage of swallowing D. Impaired distension of the upper esophageal sphincter

B. Poor oral hygiene and dependence for oral care

Which of the following is a key component when differentially diagnosing ASD from a social communication disorder (SCD)? A. Echolalia only common in SCD B. Presence of only social communicative impairments in SCD C. No differential factors D. Formulaic language only in SCD

B. Presence of only social communicative impairments in SCD

According to the Bernoulli Effect, constriction of the glottis increases airflow speech which in-turn decreases what? A. Velocity B. Pressure C. Tone D. Flow E. None of the Above

B. Pressure

Which of the following treatments is most appropriate to promote expressive and receptive language performance in conversation for an individual with aphasia? A. Melodic intonation therapy B. Promoting Aphasics' Communication Effectiveness C. Response elaboration training D. Semantic feature analysis

B. Promoting Aphasics' Communication Effectiveness

An SLP will be working with a new mother to evaluate a term infant's feeding and swallowing skills to determine the infant's readiness for oral feeding. The infant has been diagnosed with a unilateral, complete cleft lip without cleft palate. The mother asks the SLP about what caused the infant's cleft lip. The SLP explains that while we know that both genetic and environmental factors are likely involved with this congenital birth abnormality, there is no consensus in research about a single cause of cleft lip. The SLP completes a comprehensive oral sensorimotor and behavioral observation examination prior to initiating an oral-feeding trial. The following is documented in the SLP's assessment notes: Right-sided complete cleft lip; otherwise unremarkable oral peripheral mechanism examination; medical chart notes no associated neurological difficulties or diagnosed syndromes; primitive reflexes present (e.g.for example, rooting); normal observation of posture, positioning, tone, and motor activity; infant presents awake, alert, and calm; baseline vital signs at rest are normal and no changes in respiratory rate, heart rate, or oxygen saturation noted with nonnutritive sucking; no respiratory stridor noted; mild external support needed to increase lip closure at introduction of pacifier nipple, and mother with strong desire to breastfeed. The mother expressed some anxiety and concern surrounding feeding an infant with a cleft lip. Which THREE of the following treatment strategies is most appropriate for the patient if the SLP finds the infant is safe for continued oral breastfeeding and follows up with the infant and mother with therapy and education prior to discharge home from the hospital? A. Introducing external pacing with the infant's intake rate to support airway protection B. Providing the mother with contact information for the local cleft lip and palate team for ongoing care and support C. Incorporating adaptive seating for the infant during oral feedings D. Positioning the infant in the optimal feeding position at the mother's breast E. Completing education with the infant's mother about feeding interactions, strategies, and oral care

B. Providing the mother with contact information for the local cleft lip and palate team for ongoing care and support D. Positioning the infant in the optimal feeding position at the mother's breast E. Completing education with the infant's mother about feeding interactions, strategies, and oral care

Which cranial nerve is responsible for the motor function of mastication? A. Abducens B. Trigeminal C. Vagus D. Trochlear E. None of the above

B. Trigeminal

Six months ago, an SLP evaluated 4-year-old Molly's speech fluency during conversation. At that time, she displayed physically relaxed repetitions of words and phrases (occurring at a frequency of 2 per 100 words), and interjections such as "um" (occurring at a frequency of 1 per 100 words). She did not display any sound prolongations or facial grimaces; she did not produce any pitch rises or phonatory breaks; and she did not appear to avoid any sounds or words. Results from several formal tests suggested that her articulation and language development were within normal limits. Molly reportedly began producing repetitions and interjections at age 24 months, and the frequency of these disfluency types reportedly has remained stable since then. The SLP did not recommend speech-language intervention following the previous evaluation; however, she did provide the parents with information about fluency development, symptoms of stuttering, and general suggestions for how to facilitate children's fluency. A reevaluation is scheduled for next week. Which of the following is most appropriate for the SLP to do if Molly's speech fluency has remained the same since the previous evaluation? A. Recommend that Molly immediately begin fluency therapy, in which the focus is on reducing the frequency of repetitions and interjections in her conversational speech B. Recommend that Molly be released from the SLP's active caseload C. Recommend that Molly be referred for psychological counseling, with a focus on helping Molly improve speech fluency by learning how to manage anxiety more effectively D. Recommend monthly evaluations of Molly's speech fluency until she is five years old

B. Recommend that Molly be released from the SLP's active caseload

Six months ago, an SLP evaluated 4-year-old Molly's speech fluency during conversation. At that time, she displayed physically relaxed repetitions of words and phrases (occurring at a frequency of 2 per 100 words), and interjections such as "um" (occurring at a frequency of 1 per 100 words). She did not display any sound prolongations or facial grimaces; she did not produce any pitch rises or phonatory breaks; and she did not appear to avoid any sounds or words. Results from several formal tests suggested that her articulation and language development were within normal limits. Molly reportedly began producing repetitions and interjections at age 24 months, and the frequency of these disfluency types reportedly has remained stable since then. The SLP did not recommend speech-language intervention following the previous evaluation; however, she did provide the parents with information about fluency development, symptoms of stuttering, and general suggestions for how to facilitate children's fluency. A reevaluation is scheduled for next week. Which of the following is most appropriate for the SLP to do if Molly's speech fluency has remained the same since the previous evaluation? A. Recommend that Molly immediately begin fluency therapy, in which the focus is on reducing the frequency of repetitions and interjections in her conversational speech B. Recommend that Molly be released from the SLP's active caseload C. Recommend that Molly be referred for psychological counseling, with a focus on on helping Molly improve speech fluency by learning how to manage anxiety more effectively D. Recommend monthly evaluations of Molly's speech fluency until she is five years old

B. Recommend that Molly be released from the SLP's active caseload

The parents of an 8-year-old boy who stutters are upset because their child was placed in the lowest reading group at school because of his poor performance on an oral reading fluency assessment in the classroom. The parents are very sure that their child is reading at an average to above-average level compared to his classmates. The SLP's informal observations of the boy's reading performance during fluency therapy sessions are consistent with the parents' report. Which of the following is the most appropriate way for the SLP to advocate for the boy? A. Offer to conduct a reading assessment with the boy and report the score to the boy's teacher. B. Recommend that the teacher reassess the child's reading level with an assessment that uses silent reading. C. Wait until the child's speech improves in therapy, and then ask the child's teacher to re administer the oral reading assessment. D. Encourage the child to continue practicing oral reading to demonstrate to his teacher that individuals who stutter can read as well as other people.

B. Recommend that the teacher reassess the child's reading level with an assessment that uses silent reading.

Marc has been diagnosed with mild dysphagia. When chewing, a significant amount of food falls into the lateral sulci, and gets stuck. What impairment will likely be written on the swallow report? A. Reduced labial closure B. Reduced buccal tension and tone C. Aspiration before the swallow D. Reduced tongue strength

B. Reduced buccal tension and tone

An SLP works with a 4-year-old patient diagnosed with autism spectrum disorder. The patient is nonverbal, does not have a functional communication system, and does not follow simple directions. Which of the following communicative target goals is most appropriate for the SLP to prioritize for the patient? A. Matching identical pictures in an array of pictures B. Requesting wants in any modality C. Labeling of common items verbally D. Imitating a sequence of two actions

B. Requesting wants in any modality

An SLP tries to elicit correct production of the target sound /s/. Which of the following words provides the best coarticulatory conditions to meet the SLP's goal? A. Spoon B. Sea C. Sob D. Soup

B. Sea

What type of hearing loss is present when air and bone conduction thresholds are equal and both display a hearing loss? A. Mixed loss B. Sensorineural C. Conductive D. Sloping

B. Sensorineural

Which of the following strategies is most appropriate for an SLP to try in an effort to minimize the perception of mild nasal emission in a patient? A. Increasing fundamental frequency B. Using light articulatory contacts C. Reducing mouth opening D. Increasing rate of speech

B. Using light articulatory contacts

A typically developing child who is 4 years and 5 months old was referred to a school-based SLP for an evaluation because of poor speech intelligibility. Assessment results indicated a moderate phonological impairment with receptive language skills developing as expected for the child's age. Based on the assessment, eligibility for speech-language services was determined. In accordance with the Individuals with Disabilities Education Act (IDEA), which of the following service plans is most appropriate for the SLP to recommend? A. Services will be provided by the SLP when the student enters kindergarten. B. Services will be provided by the SLP in the least restrictive environment. C. Services will be provided by the school's intensive-needs preschool staff. D. Services will be provided by a private SLP with consultation from the school SLP.

B. Services will be provided by the SLP in the least restrictive environment.

An 81-year-old female presents to a rehabilitation hospital following a severe L MCA stroke. The patient has poststroke right-sided weakness, right neglect, and suspected nonfluent aphasia. Receptive language appears to be mildly impacted. The patient has a complex medical history, including TBI following a car accident two years ago, obesity, diabetes, seizure disorder, congestive heart failure, and hypertension. Socially, the patient has lived alone for the past three years following her husband's death. Since her car accident she mainly watches TV at home and leaves her house only for dialysis. She has meals on wheels and home health aides to assist her with showering. Because of her TBI, her behavior is often variable and erratic. The highest level of education she attained was seventh grade. She stayed home to take care of her children and did not have a career. Which of the following medical factors should the SLP give the most weight when determining the patient's communication prognosis? A. Frequency of seizures B. Severity of the stroke C. Level of obesity D. Severity of diabetes

B. Severity of the stroke

An aphasia evaluation of a 68-year-old retired woman reveals utterances of one to two words, no apraxia of speech, and relatively good comprehension. When the patient makes production errors, they are typically semantic errors and she attempts to correct them, often with success. Her goal is to use more words to convey ideas using complete sentences. She is active and wants to communicate in various settings, such as at her volunteer job, with her family, and when traveling. Which of the following treatments best prioritizes the patient's language needs and personal goals? A. Providing a script for the patient to recite that allows her to tell people about her stroke B. Starting Verb Network Strengthening Treatment (VNeST) to increase word retrieval in untrained discourse C. Engaging the patient in semantic feature analysis (SFA) to teach her self-cuing techniques and aid with word retrieval D. Having the patient use melodic intonation treatment (MIT) to increase fluency of commonly uttered phrases

B. Starting Verb Network Strengthening Treatment (VNeST) to increase word retrieval in untrained discourse

Which manner of articulation is made by a complete vocal tract closure, followed by a sudden release of constriction? A. Fricative B. Stop C. Affricate D. Liquid

B. Stop

A school district does not use a standard therapy log for SLP services. An SLP who works for the district develops a form to document therapy services provided. Which THREE of the following are key to include on the form to document the provision of speech and language IEP services? A. The date a student's IEP was established B. Student performance during the therapy session C. Names of other students in the group D. Types of services provided within a therapy session E. Signature from the SLP verifying delivery of services

B. Student performance during the therapy session D. Types of services provided within a therapy session E. Signature from the SLP verifying delivery of services

Which of the following is a technique used in between subject research design methods where each subject has an equal probability of being assigned either to an experimental or to a control group? A. Subject Matching B. Subject Randomization C. Sequence Randomization D. Counterbalancing

B. Subject Randomization

Kelli is a 10-year-old patient with velopharyngeal dysfunction. She presents with hypernasality, audible nasal emission, and weak pressure for oral consonants. She also displays glottal stops and pharyngeal fricatives. She is in speech therapy, and her parents would like to know what to expect after she has pharyngeal flap surgery next week. Which of the following outcomes is most appropriate for the SLP to counsel Kelli's parents to expect? A. Surgery should eliminate the glottal stops but not the pharyngeal factors. B. Surgery should decrease the hypernasality and audible nasal emission. C. Surgery should eliminate the need for ongoing speech therapy. D. Surgery should decrease hypernasality and nasal emission and eliminate articulation errors.

B. Surgery should decrease the hypernasality and audible nasal emission.

"Book read me." "Me TV see." The utterances above, spoken by a 3-year-old monolingual English-speaking child, indicate that the child most likely has a problem with which of the following? A. Semantic redundancy B. Syntactic relationships C. Morphological relationships D. Pragmatics

B. Syntactic relationships

Which of the following best describes a level 1 (i.e. strongest) research design? A. Well-designed randomized control B. Systematic review of multiple randomized controlled studies C. Case reports D. Expert opinions E. None of the above

B. Systematic review of multiple randomized controlled studies

Which of the following best represents an interprofessional treatment model? A. The SLP designs an intervention plan to be implemented by an SLP assistant. B. The SLP and general education teacher plan weekly language activities that they deliver jointly to the classroom. C. The SLP conducts the evaluation and the team designs the treatment program. D. Two district SLPs design language screening tools for the Response to Intervention (RTI) team.

B. The SLP and general education teacher plan weekly language activities that they deliver jointly to the classroom.

Doing which of the following would likely yield the most useful information about the effectiveness of an intervention strategy? A. Reviewing the results of standardized assessment instruments B. Utilizing a single-subject design (ABAB) or a reversal procedure (ABA) C. Utilizing the subjective, objective, assessment, and plan (SOAP) format D. Requesting that a colleague reevaluate the client

B. Utilizing a single-subject design (ABAB) or a reversal procedure (ABA)

A mother reports that her three-year-old son's speech seems normal at school, but at home his speech is either loud or fast, or he stops talking altogether. The mother further reports that she, her husband, and their two older children speak quickly and interrupt each other frequently. She is concerned that her son is not developing the speech and language skills to keep up. Which of the following statements is most accurate with respect to the mother's concern? A. Based on the child's age, speech is typically intermittent and thus he is very likely developing language skills normally. B. The child's speech neuromotor system can keep up with the language systems of children his own age but not with the more advanced systems in his household. C. The child's situational lack of language is more consistent with a cognitive delay than a speech or language deficit. D. The child is employing compensatory speech production techniques at school but not at home.

B. The child's speech neuromotor system can keep up with the language systems of children his own age but not with the more advanced systems in his household.

During an examination of the oral peripheral mechanism of an adult who has had a right hemisphere stroke in the territory of the middle cerebral artery, testing for lingual motor function reveals protrusion of the tongue to the left of midline. Which of the following is true? A. The patient has a lesion affecting the left hypoglossal nucleus. B. The patient exhibits unilateral left lingual weakness. C. Left vocal fold paralysis should be expected because of the site of the lesion. D. The right hypoglossal nerve is damaged.

B. The patient exhibits unilateral left lingual weakness.

During videofluoroscopic evaluation of an adult patient, the SLP notices material in both pyriform sinuses after the patient swallows. Upon review, the SLP sees that both pyriform sinuses clear during the pharyngeal swallow but then refill quickly as the structures return to rest. Which of the following statements best explains this observation? A. Pharyngeal stripping was not adequate to fully clear the pyriform sinuses, so there was material in the pyriform sinuses after each swallow. B. The patient had small pharyngeal pouches (pharyngoceles) that filled with barium during the swallow and then emptied into the pyriform sinuses after each swallow. C. Hyolaryngeal elevation was not sufficient, leaving material in the pyriform sinuses after each swallow. D. The patient had a tracheoesophageal fistula, and contrast was deposited in the pyriform sinuses after each swallow.

B. The patient had small pharyngeal pouches (pharyngoceles) that filled with barium during the swallow and then emptied into the pyriform sinuses after each swallow.

The following components of a case history were collected from a medical record review and interview with a patient and the patient's caregiver. The patient has a prior history of all of the following conditions, but only three of them are significant predictors of an elevated likelihood of clinically significant dysphagia. Which THREE of the following historical items at the time of the referral to the SLP are suggestive of an elevated likelihood of clinically significant dysphagia in a 70-year-old adult? A. The patient had pneumonia as a child and then again 10 years ago. B. The patient has a history of stroke or other neurological disease. C. The patient has been losing weight since recovering from anterior cervical spine fusion. D. The patient was cured of pharyngeal cancer with radiation therapy 30 years ago. E. The patient was intubated for surgery and routinely extubated postoperatively 24 hours before the SLP consult. F. The patient has a history of spasmodic dysphonia and underwent botulinum toxin injection 10 years ago.

B. The patient has a history of stroke or other neurological disease. C. The patient has been losing weight since recovering from anterior cervical spine fusion. D. The patient was cured of pharyngeal cancer with radiation therapy 30 years ago.

This investigation was motivated by observations that when persons with dysarthria increase loudness, their speech improves. Some studies have indicated that this improvement may be related to an increase of prosodic variation. Studies have reported an increase of fundamental frequency (F0) variation with increased loudness, but there has been no examination of the relation of loudness manipulation to specific prosodic variables that are known to aid a listener in parsing out meaningful information. This study examined the relation of vocal loudness production to selected acoustic variables known to inform listeners of phrase and sentence boundaries: specifically, F0 declination and final-word lengthening. Ten young, healthy women were audio-recorded while they read aloud a paragraph at what each considered normal loudness, twice-normal loudness, and half-normal loudness. Results showed that there was a statistically significant increase of F0 declination, brought about by a higher resetting of F0 at the beginning of a sentence and an increase of final-word lengthening from the half-normal loudness condition to the twice-normal loudness condition. These results suggest that when some persons with dysarthria increase loudness, variables related to prosody may change, which in turn contributes to improvement in communicative effectiveness. However, until this procedure is tested with individuals who have dysarthria, it is uncertain whether a similar effect would be observed. Watson and Hughes are cautious when suggesting that the speech of some persons with dysarthria improves because of the prosodic changes that result from increasing vocal loudness. Of the following, which is the most likely reason for this caution? A. Only women were studied. B. The prosody of persons with dysarthria may not show similar loudness effects. C. Several acoustic variables related to speech prosody were not included. D. Reciting a paragraph aloud is unlike spontaneous speech.

B. The prosody of persons with dysarthria may not show similar loudness effects.

An SLP administers a language test to all kindergarten children in a particular school and finds that 40 percent fall below the tenth percentile of the normative sample. Which of the following is the most reasonable interpretation of this finding? A. Forty percent of the kindergarten children in the school have language disorders. B. The school's kindergarten population differs from the standardization population in the language skills measured. C. The SLP administered the test incorrectly. D. The standardization population is not representative of the nation as a whole.

B. The school's kindergarten population differs from the standardization population in the language skills measured.

A fourth-grade student presents to a school SLP with receptive-expressive language disorder. The student has difficulty with asking "wh-" questions. Specifically, the student tends to ask only "who" and "what" questions. When probed, the student will ask "why" and "when" questions but often uses the terms incorrectly. When writing the annual Individualized Education Program for the student, which of the following goals is most appropriate and measurable? A. The student will ask six related "wh-" questions with 100% accuracy. B. The student will ask six related "wh-" questions with minimal cueing in two out of three tries. C. The student will ask six related "wh-" questions with no cueing and with 100% accuracy. D. The student will ask six related "wh-" questions with no cueing in one out of three tries.

B. The student will ask six related "wh-" questions with minimal cueing in two out of three tries.

Which of the following structures is a relay center for sensory information? A. The reticular formation B. The thalamus C. The putamen D. The hippocampus

B. The thalamus

The sensorimotor integration of the muscles of the lower face depends on which two of the cranial nerves? A. The accessory and hypoglossal B. The trigeminal and facial C. The vagus and glossopharyngeal D. The phrenic and facial

B. The trigeminal and facial

A 72-year-old male presents to a skilled nursing facility (SNF) following a complicated hospital stay. The patient is morbidly obese and has a history of COPD, depression, hypertension, diabetes, Bell's palsy, encephalopathy, and congestive heart failure. During his stay, he was not intubated but became very weak as a result of extended time in bed. In the hospital, he was placed on an NDD Level 2 diet with nectar-thick liquids due to aspiration of thin liquids and poor oral control of regular solids. The patient worked until he was 50 years old, but his health conditions led to his early retirement. Before his hospitalization, the patient did not leave his house very often, and his wife helped him with all ADLs. While evaluating the patient at the SNF, the SLP noticed that the patient was disoriented and confused, appeared to be hallucinating, grunted loudly when attempting to stand, and presented with aphonia. There was no indication of this condition in the patient's hospital chart, and the patient's wife stated that the patient had no trouble using his voice while in the hospital. An ENT consult stated there was no structural reason for the aphonia. Suspecting that the aphonia was of a psychogenic etiology, the SLP made a referral to the resident psychiatrist and began treatment for both the aphonia and for dysphagia. After a few sessions of speech therapy, the patient has made limited progress with his aphonia. The resident psychiatrist has continued to work with the patient and recommends cognitive behavioral therapy sessions and continued speech therapy. Which of the following approaches is the SLP's most appropriate response to the resident psychiatrist's recommendation? A. Discharging the order for the new evaluation because therapy has already started B. Working collaboratively to further address the patient's aphonia C. Sending treatment recommendations to the psychiatrist D. Telling the patient that further speech therapy is inappropriate because of his psychological diagnosis

B. Working collaboratively to further address the patient's aphonia

A 5-year-old patient presents to an SLP with severe apraxia of speech. The SLP has been intensively working with the patient for two years, and the patient has not demonstrated any progress in a month. The SLP recommends a speech-generating device for the patient to use at home and at school, but the insurance company denied the request. Which of the following actions best allows the SLP to advocate for the patient to receive a device? A. Teaching the parent SLP terminology to use when talking to the insurance company B. Writing a letter to the insurance company to refute the basis for the denial C. Asking the parent to buy the device without approval and apply for reimbursement D. Sending a letter to the school's special education teacher asking the teacher to contact the insurance company on the SLP's behalf

B. Writing a letter to the insurance company to refute the basis for the denial

An SLP sees a college-educated 22-year-old man who has sustained a brain injury as a result of a motor vehicle accident eighteen months earlier. The man was unconscious for five days and had posttraumatic amnesia for three months. In the last year he has held three unskilled jobs, from which he was released for unspecified reasons. Based on the information given, the man's most pervasive condition is most likely impaired: A. visual construction B. attention and memory C. speech D. language

B. attention and memory

Language intervention for a child at the one-word stage should be most strongly influenced by a consideration of the child's: A. motor skills B. cognitive skills C. syntactic skills D. articulation skills

B. cognitive skills

Classifying errors based on voice, place, and manner of articulation relates to what system of speech sound disorders? A. traditional SODA system B. distinctive features C. Phonological processes D. None of the above

B. distinctive features

Individuals diagnosed as having hemifacial microsomia are also most likely to have: A. laryngeal dysfunction B. ear malformation C. webbed fingers and toes D. widely spaced eyes

B. ear malformation

Dysarthria in a patient with lower motor neuron damage generally manifests as A. spastic dysarthria B. flaccid dysarthria C. ataxic dysarthria D. hyperkinetic dysarthria

B. flaccid dysarthria

Sensorineural hearing loss resulting from Ménière's disease causes A. a sudden hearing loss without warning B. fluctuating levels of hearing loss C. complete hearing loss at all frequencies D. bilateral progressive hearing loss

B. fluctuating levels of hearing loss

An 82-year-old female patient is admitted to a skilled nursing facility following an acute hospital stay. Her diagnoses include urinary tract infection, frequent falls with subsequent hip fracture, and chronic obstructive pulmonary disease (COPD). No surgery was required for the hip fracture. Before admission to the hospital, she was living independently and able to complete all activities of daily living (ADLs) and instrumental activities of daily living (IADLs) on her own. She is referred to the facility SLP for cognitive screening because she has difficulty carrying over new information, confusion regarding weight-bearing status, and difficulty processing directions. The SLP administers the Montreal Cognitive Assessment, and the patient scores 17/30. The patient's main deficits are in the areas of short-term memory, executive functioning, and planning. After the patient is appropriately treated both medically and therapeutically, cognitive deficits remain and the interdisciplinary team does not recommend that the patient return home independently. The patient's family is frustrated with this recommendation and seeks input from other sources. After the patient completes treatment, the most important referral for the SLP to make is to a A. psychiatrist B. neurologist C. gastroenterologist D. gerontologist

B. neurologist

Children who are born with clefts often have articulation and phonological disorders due to a structural etiology. Speech errors that are due abnormal structure and not due to incorrect placement are known as: A. compensatory errors B. obligatory errors C. developmental errors D. traditional errors

B. obligatory errors

Federal laws regarding freedom of access to information stipulate that client records kept or written by health care professionals can be A. reviewed only by other health-care professionals B. reviewed only by the clients themselves unless the client provides written permission to share with others C. reviewed by anyone who submits a formal written request D. released only by subpoena

B. reviewed only by the clients themselves unless the client provides written permission to share with others

Eye movements, twitching, and phonation breaks are overt examples of ___ behaviors. A. core behaviors B. secondary behaviors C. primary behaviors D. none of the above

B. secondary behaviors

Children with semantic-pragmatic language disorders often have difficulty learning deictic words because such terms A. require an understanding of indirect directives B. take the perspective of the communication partner C. refer to object permanence D. have many alternate forms

B. take the perspective of the communication partner

It is most appropriate for a SLP to treat hyperadduction of the vocal folds by having the client A. increase muscular effort through pushing or lifting while vocalizing B. use light and gentle vocal-fold contacts C. attempt quick onset of phonation D. use drills with exaggerated contrastive stress on words

B. use light and gentle vocal-fold contacts

Which of the following symptoms are typical of anomic aphasia? A. deficits in repetition B. word finding deficits only C. writing deficits only D. reading deficits only

B. word finding deficits only

Which of the following is an example of a modeled trial? A. "This is the opposite of a cat. What is this?" B. "What animal is this a picture of?" C. "This is a dog. Can you say the word 'dog'?" D. "This is a picture of a /da.../.forward slash d a ellipses forward slash"

C. "This is a dog. Can you say the word 'dog'?"

To best apply the multiple oppositions method for phonological errors when treating a child, an SLP should use pairs of words such as: A. "son" and "ton," "fun" and "pun," and "zoo" and "do" to help discriminate stops from fricatives B. "torn" and "sore," "soup" and "two," and "fast" and "pat" to help produce final consonants C. "doe" and "go," "doe" and "though," and "doe and Joe" to address using /d/, as in the word dog for various sounds D. "key" and "tea," "cone" and "tone," and "cap" and "tap" to address the use of velar fronting

C. "doe" and "go," "doe" and "though," and "doe and Joe" to address using /d/, as in the word dog for various sounds

Which of the following vowels is characterized as a high, front vowel in the English language? A. /u/ B. /o/ C. /i/ D. /e/ E. None of the above

C. /i/

A child exhibits the following production errors. w/r θ/s t/ʃ t/tʃ z/dʒ t/k d/g If a target sound for initial intervention is to be selected on the basis of established developmental norms, then that sound will be A. /s/ B. /ʃ/ C. /k/ D. /tʃ/

C. /k/

Which of the following groups of phonemes can best be described as the Six-Sound Test by Ling? A. /u/, /e/, /o/, /a/, /æ/, /ə/ B. /f/, /s/, /θ/, /ʃ/, /z/, /h/ C. /m/, /a/, /u/, /i/, /s/, /ʃ / D. /d/, /g/, /e/, /o/, /s/, /ʃ/

C. /m/, /a/, /u/, /i/, /s/, /ʃ /

Identify the phoneme described based on the following distinctive features: +Voicing, +Alveolar, +Stop A. /s/ B. /d/ C. /t/ D. None of the above E. All of the above

C. /t/

Under IDEA, how many IEP meetings are required to be scheduled per year? A. 3 B. 2 C. 1 D. 6 E. All of the above F. No requirements

C. 1

At about what age do infants begin to send intentional and purposeful messages via babbling and gesturing? A. 3 months B. 6 months C. 10 months D. 2 years old

C. 10 months

By 48 months, a child's speech is about ___% intelligible. A. 80% B. 50% C. 100% D. 75%

C. 100%

Jen understands simple opposites and questions. Her mother reports that Jen is constantly asking 'why'?. Based on typical development, what age is Jen closest to? A. 8 months B. 12 months C. 2 1/2 years D. 4 years

C. 2 1/2 years

At about what age do typically developing children have about 200-300 words in their expressive vocabulary? A. 12 months B. 18 months C. 24 months D. 3 years old

C. 24 months

How many continuing education hours are required across a three-year certification renewal period to maintain a Certificate of Clinical Competence in Speech-Language Pathology? A. 10 contact hours B. 20 contact hours C. 30 contact hours D. 40 contact hours

C. 30 contact hours

There are ___ intrinsic muscles and ___ extrinsic muscles of the tongue A. 3 intrinsic, 5 extrinsic B. 4 intrinsic, 3 extrinsic C. 4 intrinsic, 4 extrinsic D. 5 intrinsic, 5 extrinsic E. None of the above

C. 4 intrinsic, 4 extrinsic

A 4 1/2-year-old boy has significant speech and language difficulties exemplified by poor oral-motor control, slight difficulty in swallowing, high palate, poor tongue mobility, and fasciculation on protrusion. During an evaluation, the speech-language pathologist notes very poor paper-and-pencil grasp, poor posture, and an inability to complete performance tasks requiring fine motor control. To which of the following should the child be referred in order to obtain additional diagnostic information? A. An otolaryngologist B. A physical therapist C. A neurologist D. A physiologist

C. A neurologist

A clinician is starting treatment with a new client with specific language impairment (SLI). Which of the following is the most appropriate source of information for the SLP to use in developing the treatment plan? A. Advice from a senior colleague in the speech clinic B. An all-day seminar presented by a leading clinician in the area of SLI C. A peer-reviewed article describing a large-scale study comparing several methods of treatment for SLI D. A conference presentation of unpublished cutting-edge research comparing several methods of treatment for SLI

C. A peer-reviewed article describing a large-scale study comparing several methods of treatment for SLI

This investigation was motivated by observations that when persons with dysarthria increase loudness, their speech improves. Some studies have indicated that this improvement may be related to an increase of prosodic variation. Studies have reported an increase of fundamental frequency (F0) variation with increased loudness, but there has been no examination of the relation of loudness manipulation to specific prosodic variables that are known to aid a listener in parsing out meaningful information. This study examined the relation of vocal loudness production to selected acoustic variables known to inform listeners of phrase and sentence boundaries: specifically, F0 declination and final-word lengthening. Ten young, healthy women were audio-recorded while they read aloud a paragraph at what each considered normal loudness, twice-normal loudness, and half-normal loudness. Results showed that there was a statistically significant increase of F0 declination, brought about by a higher resetting of F0 at the beginning of a sentence and an increase of final-word lengthening from the half-normal loudness condition to the twice-normal loudness condition. These results suggest that when some persons with dysarthria increase loudness, variables related to prosody may change, which in turn contributes to improvement in communicative effectiveness. However, until this procedure is tested with individuals who have dysarthria, it is uncertain whether a similar effect would be observed. Which of the following best describes the experimental design of the Watson and Hughes study? A. A multiple-baseline design B. A between-subjects design C. A within-subjects design D. A mixed between- and within-subjects design

C. A within-subjects design

Which of the following statements best indicates a screening tool with high sensitivity? A. The majority of the children who passed the screening were found to have a communication disorder. B. Many of the children who passed the screening did not have a communication disorder. C. All of the children who failed the screening were found to have a communication disorder. D. Most of the children who failed the screening did not have a communication disorder.

C. All of the children who failed the screening were found to have a communication disorder.

the child was evaluated by the same test twice, first with form A, and again with form B. The scores of both tests should be consistent based on what concept? Which accounts for the stability between multiple forms of the same test? A. Concurrent validity B. Split-half reliability C. Alternate form reliability

C. Alternate form reliability

Which of the following approaches most accurately helps an SLP know if a treatment plan for an adolescent with language disorders is having an impact on academic learning? A. Reviewing the results of state standards-based assessments B. Administering the Peabody Picture Vocabulary Test (PPVT) C. Analyzing performance on classroom-based assessments D. Using the Clinical Evaluation of Language Fundamentals (CELF-5) assessment

C. Analyzing performance on classroom-based assessments

Ms. Brown, a 70-year-old retired female, was admitted to the hospital following a CVA. The SLP conducted a comprehensive evaluation that revealed the presence of left-side neglect, anosognosia (denial of impairment), and visuospatial problems, including prosopagnosia (difficulty recognizing familiar faces). Although her auditory comprehension and repetition skills were good, she experienced difficulty with topic maintenance and turn taking. Based on the clinical features described, Ms. Brown's diagnostic classification would most likely be: A. Wernicke's aphasia B. Conduction aphasia C. Cognitive-communicative disorder consistent with right hemisphere damage D. Cognitive-communicative disorder consistent with left hemisphere damage

C. Cognitive-communicative disorder consistent with right hemisphere damage

Joanna is a 36-month-old child who is enrolled in a treatment program for stuttering. The SLPS L P is involving Joanna's parents and 5-year-old brother in treatment activities. Every time Joanna's brother participates in a treatment activity, Joanna begins to stutter much more severely than she does when interacting with her parents. Which of the following actions is the most appropriate for the SLPS L P to take next? A. Helping the parents devise strategies for limiting how often Joanna and her brother converse with each other at home B. Engaging Joanna in a role-playing activity that seeks to identify how she would like her brother to interact with her during conversations C. Analyzing recorded conversations between Joanna and her brother to identify discourse factors that might affect the severity of Joanna's stuttering D. Teaching her brother to produce statements that support Joanna's attempts to talk

C. Analyzing recorded conversations between Joanna and her brother to identify discourse factors that might affect the severity of Joanna's stuttering

The doctor explained to Sal's family that when he overdosed, there was a period of time where his brain was deprived of oxygen. What type of injury did Sal suffer? A. Ataxia B. CVA C. Anoxia D. Recurrent laryngeal nerve damage

C. Anoxia

An otolaryngologist has referred a 45-year-old man for voice treatment following medialization thyroplasty for a paralyzed vocal fold. Which of the following is the most appropriate therapeutic strategy for the SLP to use? A. Assisting the patient to produce a soft glottal attack B. Digitally manipulation of the patient's neck to reduce strap-muscle tension C. Assisting the patient to produce a hard glottal attack D. Employing techniques aimed at increasing airflow

C. Assisting the patient to produce a hard glottal attack

An SLP completes a speech and language screening of a three-and-a-half-year-old boy. He presented with the following skills. - Age-appropriate articulation skills - Restricted mean length of utterance of approximately 2.0 - Echolalia - Difficulty following directions - Difficulty responding to name - Minimal eye contact Based on the screening information above, which of the following diagnostic categories is the most appropriate to consider when preparing for the diagnostic evaluation? A. Social communication disorder B. Developmental speech and language disorder C. Autism spectrum disorder D. Auditory processing disorder

C. Autism spectrum disorder

Hyperkinetic dysarthria is associated with damage to what site? A. Cerebellum B. UMN C. Basal ganglia D. LMN E. All of the above

C. Basal ganglia

One treatment option for adductor spasmodic dysphonia involves the percutaneous injection of which of the following into the thyroarytenoid muscle? A. Gelfoam B. Collagen C. Botulinum toxin D. Autologous fat

C. Botulinum toxin

Which cranial nerve innervates (unilaterally) all intrinsic muscles of the tongue and all but one extrinsic muscle? A. CN XI B. CN IX C. CN XII D. CN V

C. CN XII

Sally recently suffered a stroke. Her balance and coordination present as if she is drunk. Based on these symptoms, which type of stroke did Sally most likely have? A. Brainstem Stroke B. Left MCA C. Cerebellar Stroke D. None of the Above

C. Cerebellar Stroke

Which of the following statements regarding risk factors for developmental reading disorders is most accurate? A. There are no risk factors for developmental reading disorders that an SLP can assess. B. Only those children with a family history of reading disorders are at risk for developing a reading disorder. C. Children with speech sound disorders (SSDs) are at increased risk of having a reading disorder. D. Children with SSDs are only at increased risk of a reading disorder if they do not receive speech therapy.

C. Children with speech sound disorders (SSDs) are at increased risk of having a reading disorder.

A 75-year-old patient with a history of chronic obstructive pulmonary disease underwent a three-vessel coronary artery bypass graft surgery. The patient needed to be intubated for three days following surgery. After extubation, the medical team requested a swallow consultation to determine if it was safe for the patient to take oral medications and initiate oral nutrition. Based on the patient's condition, which of the following assessment procedures is most appropriate for an SLP to practice? A. Administering a clinical bedside swallow evaluation and recommending an oral diet as there were no clinical signs or symptoms of aspiration during the evaluation B. Deferring the evaluation for 24 hours because the patient was just extubated to allow a possible improvement in the patient's swallow function before evaluation C. Completing a clinical bedside swallow evaluation to determine patient readiness for an instrumental evaluation and then completing the instrumental evaluation if the patient is ready D. Determining any preexisting dysphagia with the patient and the patient's family; if there is no history of dysphagia, recommending to advance the patient's diet

C. Completing a clinical bedside swallow evaluation to determine patient readiness for an instrumental evaluation and then completing the instrumental evaluation if the patient is ready

Which of the following reflexive responses is primarily responsible for expelling material from the airway of healthy adult individuals? A. Rooting reflex B. Gag reflex C. Cough reflex D. Swallowing reflex

C. Cough reflex

A videofluoroscopic study of a client with dysphagia revealed post-swallow vallecular residue occupying more than 50 percent vallecular height. Which of the following is the most likely overt symptom that the client will experience? A. Watery eyes during swallowing B. Oral pocketing of foods C. Coughing after swallowing D. Esophageal reflux

C. Coughing after swallowing

Control over the fundamental frequency of the laryngeal tone is most closely related to the activity of which of the following muscles? A. Posterior cricoarytenoid B. Lateral cricoarytenoid C. Cricothyroid D. Sternocleidomastoid

C. Cricothyroid

A ___ type of assessment is used to identify what the client can and cannot do. This assessment type may or may not be standardized. A. Norm-referenced B. informal assessment C. Criterion referenced D. None of the above E. All of the above

C. Criterion referenced

A 67-year-old male patient with no history of swallowing problems has undergone a cardiothoracic surgical procedure. Postoperatively, he is found to be aspirating while swallowing and is diagnosed with a left vocal-fold paralysis and left pharyngeal paresis. Which of the following is the most likely etiology? A. An intraoperative CVA in the right pons B. Damage to the right recurrent laryngeal nerve C. Damage to the left recurrent laryngeal nerve D. A left hemispheric stroke

C. Damage to the left recurrent laryngeal nerve

Which of the following risk factors has the highest independent predictive value for pneumonia in elderly people with dysphagia? A. Drinking more than six glasses of water per day B. Experiencing aspiration of saliva at night C. Depending on others for feeding and oral care D. Having a history of acute hemispheric stroke more than one year ago

C. Depending on others for feeding and oral care

A 24-year-old male self-refers for a fluency evaluation. His presenting complaint is stuttering. During conversation at the initial assessment, he speaks intelligibly at a typical rate and produces no overt stutter-like disfluencies. He reports that he often expects to stutter while conversing, but that he usually can prevent or conceal the occurrence of the expected fluency disruptions either by substituting a word or by inserting a pause or "um" before the word upon which he expects to be disfluent. He states that these strategies are useful and that he would like to be able to "talk without thinking about talking." He reports that he attended speech therapy from elementary school through high school and that it helped him reduce disfluency significantly during therapy activities, but his disfluency frequency did not change much during activities outside of therapy. He fears that coworkers will react negatively to hearing him stutter. Consequently, he talks as little as possible at work. The patient's assessment results are most consistent with which of the following diagnoses? A. Neurogenic stuttering B. Cluttering C. Developmental stuttering D. Specific language impairment

C. Developmental stuttering

___ describes a breathing technique focused on relaxation and includes a slow, deep breath through your nose as your stomach expands outward and concludes with a slow exhale through your lips as your stomach moves inward. A. Shallow breathing B. Clavicular breathing C. Diaphragmatic breathing

C. Diaphragmatic breathing

Which of the following syndromes results from the presence of an extra copy of chromosome 21? A. 22q11.2 deletion syndrome B. Fragile X syndrome C. Down syndrome D. Velocardiofacial syndrome

C. Down syndrome

A school-age patient being assessed for a language disorder listens to a short passage and is asked "wh" questions based on the passage. The SLP reviews the patient's responses, records any errors, and then teaches the patient strategies to answer "who" and "what" questions. The SLP makes careful notes about how much assistance the client requires to achieve a correct response. After several sessions, the SLP reassesses the patient's responses to "wh" questions. Which of the following types of assessment is primarily exemplified in the scenario? A. Standardized assessment B. Criterion-referenced assessment C. Dynamic assessment D. Curriculum-based assessment

C. Dynamic assessment

Which of the following activities is an example of a metalinguistic strategy used to assist a student with language difficulties? A. Providing a definition of a word B. Placing story elements in sequential order C. Editing the writing of same-aged peers D. Discussing similarities and differences between two pictures

C. Editing the writing of same-aged peers

An SLP designs a series of treatment activities for Jake, a 4 year old who presents with severely delayed phonological development. During one of the activities, the SLP asks Jake to say various word pairs in which two phonologically dissimilar target sounds are contrasted (for example, "chip" and "rip"). Which of the following choices best describes the treatment approach that is being used? A. Multiple-oppositions approach B. Minimal-pairs approach C. Maximal-oppositions approach D. Metaphonological approach

C. Maximal-oppositions approach

A 23-year-old client recently completed an intensive 3-week-long summer program for stuttering and now exhibits stuttering symptoms on fewer than 3 percent of syllables during both in-clinic conversations with the SLP and beyond-clinic conversations with family members. The SLP is concerned that the client might relapse now that intervention has ended and wishes to enact a plan to help the client maintain fluency gains. Which of the following plans is the most likely to result in maintenance of the fluency gains? A. Recommending that the client participate in an annual intensive review of stuttering management skills B. Recommending that the client practice fluency management skills during monthly telephone calls with the SLP C. Ensuring the client understands the need for continued use of stuttering management techniques and referring to a local SLP for ongoing treatment D. Scheduling the client for reenrollment in the intensive 3-week-long intervention program

C. Ensuring the client understands the need for continued use of stuttering management techniques and referring to a local SLP for ongoing treatment

Hearing loss in infants who are born with a cleft palate is usually related to which of the following? A. The infant's inability to create positive pressure in the oral cavity B. Malformation of the middle-ear ossicles associated with malformation of the palate C. Eustachian tube dysfunction D. Cochlear dysfunction

C. Eustachian tube dysfunction

Which of the following phonological processes is a child expected to suppress by 3 years of age? A. Displaying stopping patterns B. Using vocalization substitutions C. Exhibiting weak syllable deletions D. Having consonant cluster reductions

C. Exhibiting weak syllable deletions

Which of the following communication disorders is most frequently associated with significant dysphagia? A. Aphasia B. Ataxic dysarthria C. Flaccid dysarthria D. Organic voice tremor

C. Flaccid dysarthria

For which of the following conditions is it most appropriate for the SLP to recommend that the patient's primary-care physician refer the patient to a prosthodontist for construction of a palatal-lift appliance? A. Submucous cleft palate B. Unrepaired cleft of the secondary palate C. Flaccid paralysis of the soft palate D. Congenitally short palate

C. Flaccid paralysis of the soft palate

Muscle tension dysphonia is a condition that occurs as a result of inappropriate use of the muscles around the larynx. Which etiology would muscle tension dysphonia be classified as? A. Neurogenic B. Organic C. Functional D. All of the Above

C. Functional

The lowest pure tone component of a sound is also called the sounds ___. A. Harmonic B. Amplitude C. Fundamental Frequency D. B & C E. None of the above F. A and B

C. Fundamental Frequency

Which of the following most accurately represents the etiology of cleft palate? A. Genetic factors alone B. Environmental influences alone C. Genetic factors interacting with environmental influences D. Medications taken by the mother during pregnancy

C. Genetic factors interacting with environmental influences

Protection of confidential health information is part of which law? A. ESSA B. IDEA C. HIPAA D. ASHA code of ethics

C. HIPAA

A 72-year-old male presents to a skilled nursing facility (SNF) following a complicated hospital stay. The patient is morbidly obese and has a history of COPD, depression, hypertension, diabetes, Bell's palsy, encephalopathy, and congestive heart failure. During his stay, he was not intubated but became very weak as a result of extended time in bed. In the hospital, he was placed on an NDD Level 2 diet with nectar-thick liquids due to aspiration of thin liquids and poor oral control of regular solids. The patient worked until he was 50 years old, but his health conditions led to his early retirement. Before his hospitalization, the patient did not leave his house very often, and his wife helped him with all ADLs. While evaluating the patient at the SNF, the SLP noticed that the patient was disoriented and confused, appeared to be hallucinating, grunted loudly when attempting to stand, and presented with aphonia. There was no indication of this condition in the patient's hospital chart, and the patient's wife stated that the patient had no trouble using his voice while in the hospital. An ENT consult stated there was no structural reason for the aphonia. Suspecting that the aphonia was of a psychogenic etiology, the SLP made a referral to the resident psychiatrist and began treatment for both the aphonia and for dysphagia. After a few sessions of speech therapy, the patient has made limited progress with his aphonia. The resident psychiatrist has continued to work with the patient and recommends cognitive behavioral therapy sessions and continued speech therapy. Which of the following strategies can the SLP use to most effectively engage the patient in monitoring his progress? A. Recording their sessions for the patient to listen to later B. Reinforcing that the aphonia is not the patient's fault C. Helping the patient recognize his control over his vocal quality D. Asking the patient to keep a journal of why he feels he cannot use his voice

C. Helping the patient recognize his control over his vocal quality

Patient is a 77-year-old female with a left hemisphere stroke. Sensorimotor clinical examination revealed right lower facial droop, impaired oral sensation, absence of dentition, need for frequent prompts to engage the patient in evaluation procedures, and aphasia. Bedside swallow evaluation with pureed food and nectar-thick liquids revealed food remnants in right lateral sulcus, drooling from the right oral cavity, and intermittent cough that was not present before the swallow evaluation. Prior medical history includes COPD, diabetes, and congestive heart failure. Patient does not want an instrumental examination. Based on the data above, which of the following is most likely to lead to a clinically significant dysphagia-related adverse event? A. Impaired oral sensorimotor function B. Prior medical history C. Impaired level of alertness D. Aphasia

C. Impaired level of alertness

The head lift exercise published by Shaker et al. (2002) was designed to produce which of the following changes in swallow biomechanics? A. Increase in laryngeal closure duration B. Strengthening of the base of the tongue C. Increase in diameter and duration of the upper esophageal sphincter (UES) opening D. Prolonged relaxation of the cricopharyngeal portion of the inferior constrictor muscle

C. Increase in diameter and duration of the upper esophageal sphincter (UES) opening

An SLP in a private practice receives a letter from the state licensing office informing her that her license expired the previous week. The SLP was unaware that she had been practicing with a lapsed license for seven working days. Which THREE of the following actions are the best course of action for the SLP? A. Continuing to see patients as scheduled B. Discontinuing treatment documentation C. Informing the practice administrator D. Reassigning patients to a licensed SLP E. Alerting patients to the lapse in licensure

C. Informing the practice administrator D. Reassigning patients to a licensed SLP E. Alerting patients to the lapse in licensure

A 5-year-old girl with a repaired cleft palate has recently undergone a pharyngeal flap operation to correct velopharyngeal incompetence, but she continues to use glottal stops, pharyngeal fricatives, and mid-dorsum palatal stops. Which of the following is the most appropriate action for the SLP to take? A. Recommending that the child be examined by a cleft palate team to determine the cause of the persistent articular errors B. Referring the child back to the surgeon to determine the need for a revision of the pharyngeal flap C. Initiating articulation treatment to teach the correct placement for the stops and fricatives D. Initiating articulation treatment to teach correct production of nonpressure consonant sounds

C. Initiating articulation treatment to teach the correct placement for the stops and fricatives

Which type of stroke is caused by a blocked artery? A. Hemorrhagic stroke B. Cerebellar stroke C. Ischemic stroke D. Subarachnoid stroke

C. Ischemic stroke

Which of the following best describes the speech reception threshold (SRT)? A. It is the intensity level at which spondee words are recognized at or near 100% of the time. B. It is the lowest intensity level at which spondee words can be detected and recognized as speech. C. It is the intensity level at which spondee words are recognized at least 50% of the time. D. It is the lowest intensity level at which spondee words can be discriminated from one another.

C. It is the intensity level at which spondee words are recognized at least 50% of the time.

Which of the following is an example of a reversible passive? A. The balloon was broken by the pin. B. The kite was flown by Ben Franklin. C. Jared was bullied by Michael. D. The girl put her doll by her friend.

C. Jared was bullied by Michael.

An SLP is evaluating a preschool child referred for concerns regarding stuttering. In which of the following areas is performance likely to be weaker in a preschooler who stutters than in a typical preschooler? A. Hearing acuity B. Voice C. Language D. Swallowing

C. Language

Which organ at the top of the neck is responsible for breathing, sound production and aspiration prevention? A. Thyroid gland B. Parathyroid gland C. Larynx D. Pharynx E. None of the Above

C. Larynx

When completing a videofluoroscopic swallow study on a geriatric patient, which of the following findings is most appropriately considered typical development? A. Vallecular retention of more than 50 percent of vallecular height B. Nasopharyngeal regurgitation C. Liquid bolus enters pharynx before hyolaryngeal excursion begins D. Oral pocketing

C. Liquid bolus enters pharynx before hyolaryngeal excursion begins

Which of the following formants typically characterizes a high vowel? A. High-frequency second formant (F2) B. High-frequency first formant (F1) C. Low-frequency first formant (F1) D. Low-frequency second formant (F2)

C. Low-frequency first formant (F1)

Which of the following procedures would be effective in remediating a falsetto voice for an adult male with a severe bilateral hearing loss? A. Development of phonation from coughing B. Pushing exercises C. Manual depression of the larynx D. Manual elevation of the larynx

C. Manual depression of the larynx

Which of the following procedures is most important for an SLP to consider when assessing the social aspect of a school-age child's communication skills? A. Collecting a language sample to assess narrative skills B. Assessing expressive vocabulary used by the child on a picture vocabulary test C. Observing the child interacting with peers in natural settings, such as during recess D. Looking for a discrepancy between the child's IQ and communication skills to determine eligibility

C. Observing the child interacting with peers in natural settings, such as during recess

Which of the following muscles opposes lip retraction (spreading) ? A. Risorius B. Levator labii superioris C. Orbicularis oris D. Zygomaticus major

C. Orbicularis oris

Which of the following muscles produces the opposing action to those that produce velopharyngeal closure? A. Musculus uvulae B. Levator veli palatini C. Palatoglossus D. Stylopharyngeus

C. Palatoglossus

Which of the following constitutes the major component of an audiologic rehabilitation program for infants with a moderate sensorineural hearing loss? A. Development of cognitive skills B. Development of social skills C. Parent-mediated auditory stimulation D. Gestural stimulation

C. Parent-mediated auditory stimulation

The following sentence exhibits what type of disfluency? 'My m-m-m-mouth is w-w-w-watering when I look at that sandwich.' A. Whole phrase repetition B. Sound prolongations C. Part word repetitions D. Frequent interjections

C. Part word repetitions

Which of the following areas needs to be evaluated first for a 5 year old who says [pun] for "spoon" and [top] for "soap"? A. Auditory discrimination B. Dialectal differences C. Phonological system D. Receptive language

C. Phonological system

The smallest unit of language that can change meaning of words but does not have meaning on it's own is known as: A. Morphology B. Syntax C. Phonology D. Semantics E. None of the above

C. Phonology

A number of research reports have described poor auditory memory in children with language impairments. Which of the following can most appropriately be concluded from these studies? A. Poor auditory memory can be improved by language-intervention programs that focus on teaching vocabulary and word meanings. B. Poor auditory memory is a reflection of a language impairment, and clinicians and researchers cannot effect improvement. C. Poor auditory memory could be a reflection or a cause of a language impairment or could be related to some other factor, and further research is needed to determine which is the case. D. Children with language impairments normally do not have well-developed representational skills.

C. Poor auditory memory could be a reflection or a cause of a language impairment or could be related to some other factor, and further research is needed to determine which is the case.

A two-year-old child produces the following utterances during a play-based conversation with her mother. No kitty --> It's not a kitty. My doggie --> This toy dog is mine. No goat --> It's not a goat. Kitty house --> The cat is in the house. Big kitty --> It's a big cat. Doggie ball --> The dog pushed the ball. Which of the following semantic-syntactic constructions does the child show evidence of using? Select all that apply. A. Disappearance B. Denial C. Possessor + Possession D. Attribute + Entity E. Agent + Action

C. Possessor + Possession D. Attribute + Entity E. Agent + Action

Which of the following conditions is singularly caused by a genetic abnormality? A. Cleft lip and palate B. Specific language impairment C. Prader-Willi syndrome D. Cerebral palsy

C. Prader-Willi syndrome

An assessment of a patient with suspected neurodegenerative disease who presents with language problems as the primary symptom reveals the following. - Impaired single-word retrieval in speech - Naming that includes phonological errors - Impaired repetition of sentences and phrases - Spared single-word comprehension and object knowledge - Spared motor speech abilities Which of the following disorders is the most accurate diagnosis of the patient? A. Broca's aphasia B. Anomic aphasia C. Primary progressive aphasia D. Transcortical motor aphasia

C. Primary progressive aphasia

Early intentional communication typically emerges in the months leading up to a child's first birthday. Which of the following communicative functions typically emerges first in this period? A. Requesting B. Commenting C. Protesting D. Greeting

C. Protesting

A 28-year-old classroom teacher complaining of frequent voice loss is seen by an SLPS L P and an otolaryngologist. It is determined the patient's symptoms are linked to significant vocal demands. Which of following recommendations is the most appropriate first step to treat the underlying disorder? A. Advising the patient to undergo complete voice rest until the nodules improve or resolve B. Educating the patient on the importance of hydration and behavioral antireflux strategies C. Recommending voice amplification with resonant voice therapy D. Training strategies to increase vocal loudness

C. Recommending voice amplification with resonant voice therapy

A 2-year-old child pronounces "bottle" as [baba]. Which of the following best describes the speech simplification process evident in this utterance? A. Gliding B. Stopping C. Reduplication D. Devoicing

C. Reduplication

A teacher asks the speech-language pathologist for advice regarding a child who talks excessively during class, rarely listens to instructions, and does work only intermittently. Attempts at alternative seating for the child have not been successful. Of the following, which is the most appropriate recommendation that the speech-language pathologist can provide to the teacher? A. Have the school counselor consider working with the child and the parents on self-control and discipline B. Put the child on a management system for classroom behavior C. Refer the child for evaluation by members of the child-study team D. Suggest remedial speech-language services for the child to improve the child's interactive communication

C. Refer the child for evaluation by members of the child-study team

An 81-year-old female presents to a rehabilitation hospital following a severe L MCA stroke. The patient has poststroke right-sided weakness, right neglect, and suspected nonfluent aphasia. Receptive language appears to be mildly impacted. The patient has a complex medical history, including TBI following a car accident two years ago, obesity, diabetes, seizure disorder, congestive heart failure, and hypertension. Socially, the patient has lived alone for the past three years following her husband's death. Since her car accident she mainly watches TV at home and leaves her house only for dialysis. She has meals on wheels and home health aides to assist her with showering. Because of her TBI, her behavior is often variable and erratic. The highest level of education she attained was seventh grade. She stayed home to take care of her children and did not have a career. Which of the following tasks is most appropriate to include when assessing the patient's phrase length? A. Completing divergent naming tasks B. Maintaining a conversation with a known partner C. Responding to open-ended questions D. Repeating sentences of varying lengths

C. Responding to open-ended questions

Which of the following speech-sampling contexts best assesses hyponasality? A. Producing sentences with oral sonorants B. Counting in numerical order from 60 to 70 C. Saying single words with nasal consonants D. Repeating words with oral-pressure consonants

C. Saying single words with nasal consonants

According to research on the development of Brown's morphemes in young children, which of the following is a determinant of acquisition order? A. Phonological ease of production B. Figurative-language ability C. Semantic and syntactic complexity D. Sequencing and segmentation strategies

C. Semantic and syntactic complexity

Which of the following therapeutic techniques is most appropriate to treat hyperfunctional voice disorders? A. Pushing, pulling, and isometrics B. Lee Silverman Voice Treatment C. Semi-occluded vocal tract exercises D. Coughing and throat clearing

C. Semi-occluded vocal tract exercises

A 65-year-old-male who is hospitalized with pneumonia completed a videofluoroscopic swallowing study. Findings showed aspiration after the swallow due to a reduced upper esophageal sphincter opening diameter separating the bolus tail, which was then retained in the pyriform sinuses. Which of the following interventions was designed to and is shown to improve upper esophageal sphincter opening? A. Effortful swallow B. Valsalva (breath hold) C. Shaker (head lift) exercise D. Lingual resistance exercise

C. Shaker (head lift) exercise

Which of the following treatment examples is based upon motor learning feedback? A. Multiple Oppositions B. Minimal Pairs C. Shaping from a sound already in the client's inventory D. Maximal oppositions

C. Shaping from a sound already in the client's inventory

Excessive nasality is associated with inadequate velopharyngeal closure. An SLP is training a client to self-monitor nasality during speech. Which of the following tactics will best allow the speaker to determine whether there is excessive nasal airflow? A. Looking in a mirror while speaking B. Being aware of vowel-sound productions C. Speaking/phonating while alternately leaving the nostrils open and pinching them closed D. Monitoring production of consonant blends

C. Speaking/phonating while alternately leaving the nostrils open and pinching them closed

A speech and language assessment was conducted on a 6-year-old child. Results indicated normal receptive, expressive, and pragmatic language skills. The child used stopping, fronting, deaffrication, and final consonant deletion. The SLP will likely find which of the following to be the most useful in planning for the child's treatment program? A. Mean length of utterance B. Diadochokinetic rate C. Stimulability information D. Literacy information

C. Stimulability information

A third-party reimburser asks the speech-language pathologist to demonstrate in a diagnostic statement that a child's communication problems have a physiological cause. Which of the following observations, if included in the statement, would best satisfy the request? A. The child's dentition is not yet fully developed but is within normal limits for a child of that age. B. The child has developmental delay, exhibiting speech that is not appropriate for a child of that age. C. The child demonstrates a motor-speech disorder and is unable to perform voluntarily the oral movements required for speech production. D. The child exhibits stridency deletion, consonant cluster reduction, stopping, and fronting.

C. The child demonstrates a motor-speech disorder and is unable to perform voluntarily the oral movements required for speech production.

Which of the following describes an important diagnostic distinction between apraxia of speech and dysarthria? A. Apraxia of speech is a result of lower motor neuron lesions, whereas dysarthria is the result of upper motor neuron lesions. B. Clients with apraxia of speech lack the ability to monitor reactive speech, whereas clients with dysarthria lack the ability to sequence volitional speech movements. C. Strength and coordination of the speech musculature are intact in clients with apraxia of speech, whereas slowness, weakness, incoordination, or altered tone of the speech musculature are associated with dysarthria. D. Apraxia of speech is characterized by distortions, omissions, and substitutions, whereas dysarthria is characterized by inconsistent, highly variable misarticulations.

C. Strength and coordination of the speech musculature are intact in clients with apraxia of speech, whereas slowness, weakness, incoordination, or altered tone of the speech musculature are associated with dysarthria.

Compensatory swallowing maneuvers and interventions are designed to mitigate a biomechanical impairment during the swallow; however, some of them have unintended consequences that have been documented in scientific literature. Which of the following swallow maneuvers has been found to produce cardiac arrhythmia (irregular heartbeat) in patients with stroke or cardiovascular diseases? A. Mendelsohn maneuver B. Chin-down posture C. Supraglottic swallow maneuver D. Head rotation toward the weak side

C. Supraglottic swallow maneuver

Which THREE of the following statements are true regarding the treatment of swallowing disorders? A. Thickened liquids cause dehydration because they require more water to be metabolized. B. The chin-down posture eliminates thin-liquid aspiration from the pyriform sinuses. C. Supraglottic swallow may cause cardiac arrhythmia in patients with coronary artery disease or stroke history. D. Mass practice and task specificity are important principles of exercise therapy. E. Periodontal disease due to poor oral hygiene increases aspiration risk in people with dysphagia. F. Shaker and jaw-opening exercises are designed to increase the upper esophageal sphincter opening.

C. Supraglottic swallow may cause cardiac arrhythmia in patients with coronary artery disease or stroke history. D. Mass practice and task specificity are important principles of exercise therapy. F. Shaker and jaw-opening exercises are designed to increase the upper esophageal sphincter opening.

An 81-year-old female presents to a rehabilitation hospital following a severe L MCA stroke. The patient has poststroke right-sided weakness, right neglect, and suspected nonfluent aphasia. Receptive language appears to be mildly impacted. The patient has a complex medical history, including TBI following a car accident two years ago, obesity, diabetes, seizure disorder, congestive heart failure, and hypertension. Socially, the patient has lived alone for the past three years following her husband's death. Since her car accident she mainly watches TV at home and leaves her house only for dialysis. She has meals on wheels and home health aides to assist her with showering. Because of her TBI, her behavior is often variable and erratic. The highest level of education she attained was seventh grade. She stayed home to take care of her children and did not have a career. Which of the following medical conditions, in combination with her stroke, is likely to have the greatest impact on the patient's communication abilities? A. Seizure disorder B. Diabetes C. TBI D. Obesity

C. TBI

To provide greater independence for a client who has a brain injury and is in a late stage of speech-language treatment, which of the following techniques is most appropriate? A. Increasing memory-retention span B. Using word-repetition drills C. Teaching compensatory strategies D. Training visual-perceptual skills

C. Teaching compensatory strategies

Which of the following statements accurately describes state licensure requirements that must be met for an SLP to legally provide services via telepractice within the state or across state lines? A. An SLP is required to hold a specialty certification to provide services via telepractice because this service delivery method requires additional skills. B. No state licensure requirements apply to individual SLPs providing services via telepractice, as long as their employers can obtain reimbursement. C. The SLP should check with all relevant state licensure boards before delivering telepractice services. D. As long as the SLP has the ASHA Certificate of Clinical Competence and is licensed in the SLP's state of residence, the SLP can legally provide services via telepractice anywhere.

C. The SLP should check with all relevant state licensure boards before delivering telepractice services.

Studies of the anatomy of human vocal folds and of mucosal behavior during phonation have led to the current cover-body characterization of the vocal fold. Which of the following is included in the vocal-fold transition? A. The epithelium and superficial lamina propria B. The epithelium and deep lamina propria C. The intermediate and deep layers of the lamina propria D. The lamina of the thyroid cartilage

C. The intermediate and deep layers of the lamina propria

A disfluent 4-year-old child is referred to an SLP for assessment. Which of the following is most important for the SLP to consider in deciding whether the child is developmentally nonfluent or stuttering? A. The length of time the child has been disfluent B. The rate at which the child talks C. The nature and frequency of the child's disfluencies D. The child's comments to the clinician about the disfluencies

C. The nature and frequency of the child's disfluencies

Which of the following best describes the rationale for using standardized, norm-referenced instruments to assess speech-language functioning? A. They enable the clinician to generate weekly statements about a client's treatment progress. B. They provide the clinician with the information that is necessary to generate a specific and comprehensive treatment plan. C. They enable the clinician to understand and make informed statements about how a client's performance compares with the performance of other people. D. They provide the clinician with reliable information about how well a client is likely to respond to treatment.

C. They enable the clinician to understand and make informed statements about how a client's performance compares with the performance of other people.

A 72-year-old right-handed female is evaluated following a left-hemisphere stroke. Findings reveal a moderate Broca's aphasia. Which of the following is the primary goal of intervention for the client? A. To improve the client's pragmatics skills and abilities B. To increase the client's ability to express functional comprehension abilities C. To improve the client's ability to express production of syntactically appropriate sentences D. To enable the client's ability to use a digital augmentative communication system

C. To improve the client's ability to express production of syntactically appropriate sentences

A ___ is an alternative hole created through an incision in the neck to access the air way. A. Laryngectomy B. Partial Laryngectomy C. Tracheostomy D. All of the above E. None of the above

C. Tracheostomy

a 79 year old female was admitted to the hospital with left sided weakness, facial droop, and slurred speech. After 72 hours her symptoms cleared and she was discharged. Sandra most likely sustained which of the following: A. Large ischemic CVA B. Parkinson's Disease C. Transient Ischemic Attack

C. Transient Ischemic Attack

A 4-year-old child presents with general speech patterns that include liquid gliding, stridency deletion, final-consonant deletion, and consonant-cluster reduction. Which of the following is the speech-language pathologist's most appropriate recommendation for the child? A. Treatment is not needed, because the child's speech will improve during the next year. B. Treatment should focus on the production of /p/, because /p/ is one of the earliest acquired phonemes. C. Treatment should use a phonological approach and focus initially on the production of final consonants. D. Formal treatment is not needed not, but the parents should be counseled to read aloud to the child, correct the child's errors, elicit correct repetitions, and, when needed, interpret to other people what the child is trying to say.

C. Treatment should use a phonological approach and focus initially on the production of final consonants.

When assessing the ability of a student with cerebral palsy to access an augmentative and alternative communication device, which of the following physical factors must be evaluated first? A. Upper extremity range of motion B. Lower extremity strength C. Trunk stability and control D. Fine motor dexterity

C. Trunk stability and control

Which of the following tympanogram types will show a reduced peak height with normal pressure? A. Type A B. Type B C. Type As D. Type C

C. Type As

Kimberly, a patient who suffered a brain stem stroke and has had a fairly successful recovery, complained of food sticking in her throat and needing to swallow two to three times to clear liquid and solid boluses. She underwent a videofluoroscopic swallowing study that showed a limited duration of upper esophageal sphincter opening. The SLP decides to employ an intervention plan that includes the Mendelsohn maneuver. Which of the following strategies is most likely to be effective in accomplishing this goal? A. Completing a fiberoptic endoscopic evaluation of swallowing (FEES) to determine if the patient is performing the maneuver properly B. Performing cervical auscultation (CA) to quantify improved swallowing with the maneuver C. Using submental surface electromyography (sEMG) as biofeedback during attempts to perform the maneuver D. Teaching the patient to use palpation of her neck and a mirror to help her master the maneuver

C. Using submental surface electromyography (sEMG) as biofeedback during attempts to perform the maneuver

Which of the following strategies to treat compensatory articulation errors is most appropriate for a child with hypernasality and glottal stop substitutions following surgery for velopharyngeal insufficiency? A. Using oral motor exercises B. Using a straw to elicit /s/ C. Using tactile cues to elicit phonemes D. Using negative practice

C. Using tactile cues to elicit phonemes

A 35-year-old female is referred for an outpatient swallowing evaluation after having a thyroidectomy. Her complaints include coughing when drinking and dysphonia. Which of the following evaluation options is the most appropriate post-treatment follow-up for the patient? A. Completing a clinical swallow evaluation B. Setting up an appointment for a pharyngeal manometry test C. Using the fiberoptic endoscopic evaluation of swallowing (FEES) D. Recording the patient during a videofluoroscopic swallow study (VFSS)

C. Using the fiberoptic endoscopic evaluation of swallowing (FEES)

This investigation was motivated by observations that when persons with dysarthria increase loudness, their speech improves. Some studies have indicated that this improvement may be related to an increase of prosodic variation. Studies have reported an increase of fundamental frequency (F0) variation with increased loudness, but there has been no examination of the relation of loudness manipulation to specific prosodic variables that are known to aid a listener in parsing out meaningful information. This study examined the relation of vocal loudness production to selected acoustic variables known to inform listeners of phrase and sentence boundaries: specifically, F0 declination and final-word lengthening. Ten young, healthy women were audio-recorded while they read aloud a paragraph at what each considered normal loudness, twice-normal loudness, and half-normal loudness. Results showed that there was a statistically significant increase of F0 declination, brought about by a higher resetting of F0 at the beginning of a sentence and an increase of final-word lengthening from the half-normal loudness condition to the twice-normal loudness condition. These results suggest that when some persons with dysarthria increase loudness, variables related to prosody may change, which in turn contributes to improvement in communicative effectiveness. However, until this procedure is tested with individuals who have dysarthria, it is uncertain whether a similar effect would be observed. Which of the following represent(s) the independent variable or variables used in the Watson and Hughes study? A. Prosody of dysarthric speech B. F0 declination and final-word lengthening C. Vocal loudness D. Speech intelligibility and communicative effectiveness

C. Vocal loudness

Which of the following communication diagnoses would most likely require a treatment program that is focused on improving auditory language comprehension? A. Right-hemisphere communication impairment B. Global aphasia C. Wernicke's aphasia D. Broca's aphasia

C. Wernicke's aphasia

SLPs often have a responsibility to communicate with the parents of children with severe disabilities. According to mourning theory, when are parents normally most receptive to information and advice provided by professionals regarding their child? A. When the parents are working through their feelings about the child's disabilities B. When the parents fully realize the extent of the child's disabilities and the limitations of treatment and education C. When the parents have acquired greater confidence in their capacity to care for the child and greater motivation to cope with the child's disabilities D. When the parents are making decisions regarding future care and protection of the child

C. When the parents have acquired greater confidence in their capacity to care for the child and greater motivation to cope with the child's disabilities

A 55-year-old woman, recently hospitalized for probable cerebrovascular accident (CVA), is referred for evaluation of stuttering speech. The initial conversation with the client indicates that speech is characterized by frequent initial-phoneme repetitions and prolongations as well as associated mildly effortful eye blinking. Which of the following pieces of information is crucial to accurate speech diagnosis and decisions regarding management of the speech problem? A. The site and extent of the lesion associated with the suspected CVA B. Whether the client has any associated dysphagia or dysphonia C. Whether the dysfluencies began before or after the suspected CVA D. Whether the client feels frustrated by the dysfluencies

C. Whether the dysfluencies began before or after the suspected CVA

____ is an esophageal phase swallowing disorder that is characterized by food collecting in a pouch. Frequent regurgitation is also common to occur. A. GERD B. Tracheoesophageal fistula C. Zenker's diverticulum D. Aspiration pneumonia

C. Zenker's diverticulum

Which motor speech disorder is characterized by a loss of coordination of articulators despite having normal muscle function? A. Dysarthria B. Anarthria C. apraxia of speech D. aphasia

C. apraxia of speech

Which motor speech disorder is characterized by inconsistent articulation errors, groping, and difficulty with repetition tasks of increased complexity? A. dysarthria B. anarthria C. apraxia of speech D. aphasia

C. apraxia of speech

The Family Educational Rights and Privacy Act (FERPA) guarantees parents access to their child's educational records. However, this mandate does not apply to the daily records kept by an SLP working in a school setting if these records: A. are not used for treatment-related reimbursement from state or local government B. remain within the school district in which the SLP works C. are kept in the sole possession of the SLP D. are not distributed to anyone outside of the child's school

C. are kept in the sole possession of the SLP

Compared with children who do not have language disorders, children with language disorders tend to A. take more conversational turns B. initiate topics and direct the flow of conversation more C. ask fewer open-ended questions D. initiate more indirect requests

C. ask fewer open-ended questions

Treatment for apraxia of speech most appropriately emphasizes: A. coordination of respiration with phonation and articulation B. auditory discrimination, resonance, and respiration C. auditory-visual stimulation, oral-motor repetition, and phonetic placement D. rate of speech, range of movement, strength, and coordination of the oral mechanism

C. auditory-visual stimulation, oral-motor repetition, and phonetic placement

A physician told the spouse of a client that melodic intonation therapy (MIT) would improve the client's speech considerably. The most appropriate next action by the SLP would be to: A. provide MIT, as recommended B. tell the physician that it is inappropriate for the physician to make recommendations for a speech treatment C. consider the potential value of incorporating MIT into the client's treatment D. explain MIT to the client's spouse to assist in the decision-making process about the type of therapy to use

C. consider the potential value of incorporating MIT into the client's treatment

John is a 4 1/2 year old whose consonantal inventory includes word-initial [ w ], [ m ], [ n ], [ p ], [ b ], [ t ], [ d ], and [ f ]. He uses [ t ] for /k/, [ d ] for /ɡ/, [ b ] for /v/, and [ f ] for /θ/. He produces no consonant clusters. His word-final consonantal inventory consists of [m] and [n]. His word shape inventory includes V, CV, CVC, and CVCV. The information given most strongly indicates that the child has: A. childhood apraxia of speech B. an oral motor impairment C. delayed phonological development D. a significant high-frequency hearing loss

C. delayed phonological development

A patient with the recent onset of idiopathic unilateral vocal fold paralysis with a large glottal gap is evaluated by an otolaryngologist and SLP. The treatment most appropriate for this patient is A. voice therapy B. injection augmentation C. injection augmentation with subsequent voice therapy D. thyroplasty with subsequent voice therapy

C. injection augmentation with subsequent voice therapy

When completing speech sound evaluations, it is best practice for an SLP to include a sample of connected speech because connected speech samples .. A. provide more reliable information than standardized tests B. contain a representative sample of all the phonemes in a language C. may result in different types of errors than those elicited by single-word elicitation tasks D. are easier to gloss than single-word productions

C. may result in different types of errors than those elicited by single-word elicitation tasks

A 6-year-old child has difficulty producing both regular and irregular plural forms. Intervention for this problem would best target language at the level of A. syntax B. phonology C. morphology D. semantics

C. morphology

A 60-year-old man says that he has "trouble thinking of names and words" and that it is interfering with his job performance. The problem began three months ago after he had a minor cerebrovascular accident. He does not report any other problems. His conversation is characterized by some hesitancies, latencies, repetitions, interjections, and self-corrections. On the basis of an interview and the results of an aphasia battery, it is concluded that he has a mild aphasia. The most appropriate course of action is to: A. advise the client to wait for three more months in order to allow spontaneous recovery to take place B. begin a treatment program designed to decrease dysfluencies in his speech C. offer a treatment program designed to improve word-retrieval skills D. encourage the client to increase his reading of information related to his profession in order to facilitate recall of professional terminology

C. offer a treatment program designed to improve word-retrieval skills

Johnny was evaluated by the school SLP. He consistently says 'top' for 'stop', and 'poon' for 'spoon'. His mistakes are characteristic of which phonological processing errors? A. assimilation processes B. substitution processes C. syllable structure processes D. Metathesis E. none of the above

C. syllable structure processes

A 6-year-old child produces [t] for /s/, [d] for /z/, [p] for /f/, and [b] for /v/. Intervention for this problem would target language at the level of A. morphology B. syntax C. phonology D. semantics

C. phonology

To best facilitate the functional and meaningful use of linguistic forms, a language-intervention program for a child with language impairments should A. focus on comprehension tasks that are just above the child's linguistic level B. stress structured, game-like situations that use imitative tasks C. present and elicit the forms in naturalistic contexts D. stress imitation of the forms to be learned

C. present and elicit the forms in naturalistic contexts

Minimal word pairs are typically used in phonological treatment in order to: A. stabilize phonological patterns in a system B. build awareness of the phonetic realizations of phonological contrasts C. probe generalization of taught sound patterns to words that have not been used in instruction D. facilitate automatic production of words

C. probe generalization of taught sound patterns to words that have not been used in instruction

According to literature on men's and women's voices, the prevalence of benign vocal fold lesions in women can best be explained by differences in A. daily dairy consumption B. speaking duration C. rates of vocal fold vibration D. neck circumference

C. rates of vocal fold vibration

Which configuration of hearing loss is characterized by air conduction thresholds of low frequencies that are at least 20 dB poorer than high frequencies? A. flat B. precipitous C. rising D. sloping

C. rising

For a patient's use of consonant cluster reduction, the SLP would most appropriately recommend intervention, including words such as: A. knight B. laugh C. stop D. wish

C. stop

After an evaluation of a patient's communication needs, an SLP pursues acquisition of a high-tech AAC device. The patient's private health insurance rejects the request, stating that the device is "not medically necessary." The SLP can best advocate for the patient by immediately appealing: A. to the insurance company to determine a lower-technology device that is covered by the payer B. the denial and providing a review of how the patient's quality of life is impacted by the device C. the denial and providing data regarding the medical necessity for the device D. to a member of the state government for coverage of high-tech AAC devices

C. the denial and providing data regarding the medical necessity for the device

When treating a client who is using an electronic augmentative-communication device, the speech-language pathologist's primary goal should be to A. ensure that the client develops skill in using every technical aspect of the aid B. ensure that the client's caregivers learn how to modify the aid's hardware and any applicable software to meet the client's communication needs C. train the client to use the aid as independently and interactively as possible in a variety of settings D. help the client develop the skills necessary for moving on to a more sophisticated device

C. train the client to use the aid as independently and interactively as possible in a variety of settings

Damage to the arcuate fasciculus may result in ________.

Conduction Aphasia

The anterior cerebral artery supplies blood to the ___.

Corpus callosum & basal ganglia

Place the examples of assessment tasks with the type of attention that is primarily being evaluated. A. Listening to a list of spoken words for a target word B. Focusing on hearing a person speaking while the television is on C. Mentally solving a complex math problem D. Sorting playing cards by color, then by number, and then by color again 1. Alternating Attention 2. Sustained Attention 3. Selective Attention 4. Working Memory

D(1), B(2), A(3), C(4)

An SLP will be working with a new mother to evaluate a term infant's feeding and swallowing skills to determine the infant's readiness for oral feeding. The infant has been diagnosed with a unilateral, complete cleft lip without cleft palate. The mother asks the SLP about what caused the infant's cleft lip. The SLP explains that while we know that both genetic and environmental factors are likely involved with this congenital birth abnormality, there is no consensus in research about a single cause of cleft lip. The SLP completes a comprehensive oral sensorimotor and behavioral observation examination prior to initiating an oral-feeding trial. The following is documented in the SLP's assessment notes: Right-sided complete cleft lip; otherwise unremarkable oral peripheral mechanism examination; medical chart notes no associated neurological difficulties or diagnosed syndromes; primitive reflexes present (e.g.for example, rooting); normal observation of posture, positioning, tone, and motor activity; infant presents awake, alert, and calm; baseline vital signs at rest are normal and no changes in respiratory rate, heart rate, or oxygen saturation noted with nonnutritive sucking; no respiratory stridor noted; mild external support needed to increase lip closure at introduction of pacifier nipple, and mother with strong desire to breastfeed. The mother expressed some anxiety and concern surrounding feeding an infant with a cleft lip. Based on information from the oral sensorimotor and behavioral observation assessments, which of the following plans for continuing with an oral-feeding trial is most appropriate with the infant? A. An oral-feeding trial should not be attempted until the infant's cleft lip is repaired. B. An oral-feeding trial should not be attempted until a VFSS or FEES instrumental examination is completed. C. An oral-feeding trial should be postponed until the infant's airway is examined by a physician. D. An oral-feeding trial by the SLP can proceed during this initial feeding and swallowing assessment.

D. An oral-feeding trial by the SLP can proceed during this initial feeding and swallowing assessment.

Alan, a 62-year-old, right-handed African American male, sustained a traumatic brain injury mostly affecting his right hemisphere and bi-lateral frontal lobes. Alan was hospitalized for 24 days before being discharged to an inpatient brain injury rehabilitation program. His spouse attends his initial evaluation sessions. The SLP reports impairments in memory, poor awareness of deficits, and some changes in his communication skills. The SLP already has current information about Alan's performance on a standardized language battery suggesting minimal impairments. Therefore, the SLP completes an evaluation of Alan's functional communication skills. The SLP's assessment involves testing his functional use of humor, facial expressions, nonverbal communication strategies, and understanding functional written materials. The SLP hopes to use this information to determine the impact of Alan's impairments on his daily life. The SLP determines that Alan would benefit from an external memory aid, specifically a memory notebook. The SLP wants to be sure that Alan can learn to use the aid but is concerned that because of his memory impairment, he will struggle to retain the basic information about how to use the aid. The SLP discourages Alan from guessing and intervenes with support before Alan can make a mistake when using the device. Alan's performance profile reveals which impairment typical to right hemisphere dysfunction? A. Aprosodia B. Unilateral spatial neglect C. Hyperresponsiveness D. Anosognosia

D. Anosognosia

Which of the following should an SLP recommend to best help a patient who has advanced to late stages of dementia of the Alzheimer's type? A. Group treatment to improve the patient's conversational intelligibility B. Individual treatment to improve the patient's recall of salient vocabulary words C. Individual treatment to improve the patient's comprehension during social discourse D. Assistance from caregivers to improve the patient's communication skills

D. Assistance from caregivers to improve the patient's communication skills

A 42-year-old client with upper-and lower-extremity weakness and a diagnosis of amyotrophic lateral sclerosis is referred for a speech-language evaluation. The evaluation reveals a progressive severe dysarthria that is characterized by imprecise articulation secondary to bilateral facial and lingual weakness, atrophy, and fasciculations; mild-to-moderate hypernasality and weak pressure consonants with associated nasal emission during speech; and strained, harsh, groaning voice quality with occasional inhalatory stridor. Speech intelligibility is poor. Which of the following will most effectively improve this client's ability to communicate? A. Teflon injection into one or both vocal cords B. Palatal-lift prosthesis C. Amplification device D. Augmentative communication system

D. Augmentative communication system

Jay is 3 years and 8 months old and was diagnosed with stuttering following a speech-language assessment. The diagnosis is based in part on the presence of part-word repetitions on 15% of syllables in conversational speech during the initial evaluation. His mother says that the onset of stuttered speech occurred eight months before the assessment and that the child occasionally is frustrated with his fluency difficulties. Which of the following is the most appropriate recommendation for the SLP to make? A. Monitoring fluency performance until age 5 years and 0 months old, but commencing fluency therapy immediately if Jay's stuttering frequency exceeds 20% of syllables before then. B. Monitoring fluency performance until age 5 years and 0 months old, and commencing fluency therapy immediately if Jay's stuttering has not resolved by then. C. Beginning a treatment program in which Jay learns how to regulate his articulation rate during daily conversational speech. D. Beginning a treatment program in which Jay's parents learn how to respond to his stuttering-related frustration and how to label and comment on his fluent and stuttered utterances.

D. Beginning a treatment program in which Jay's parents learn how to respond to his stuttering-related frustration and how to label and comment on his fluent and stuttered utterances.

Cranial nerve IX is a motor nerve, sensory nerve, or both? A. Motor B. Sensory C. Neither D. Both

D. Both

When completing an oral mechanism examination on a patient presenting with stroke-like symptoms, which of the following findings would be most likely to predict aspiration during an instrumental swallow evaluation? A. Inability to swallow on command B. Facial droop C. Absent gag reflex D. Breathy phonation

D. Breathy phonation

Which of the following neuroimaging studies specifies which artery or arteries is/are occluded in a patient with a stroke? A. Electroencephalography B. Magnetoencephalography C. Evoked-response potentials D. CT or MR angiography

D. CT or MR angiography

According to research, which of the following structural factors has the most adverse effect on articulation? A. Short labial frenulum B. Micrognathia C. Unilateral facial palsy D. Class III malocclusion

D. Class III malocclusion

A 3-year-old child was seen at the speech-and-language clinic for a speech assessment to address teacher and parent concerns about speech intelligibility. During the initial interview and observation, the SLP noted that the child exhibited inconsistent errors of vowel and consonant production during repetitive speech tasks, inappropriate prosody, and prolongations of speech sounds. The SLP chooses an assessment to determine the presence of childhood apraxia of speech (CAS). Which of the following procedures is most appropriate for this type of assessment? A. Analyzing place, manner, and voicing of all consonant sounds at the word level B. Evaluating fluency of speech during structured and unstructured conversational activities C. Testing stimulability of later-developing speech sounds in multiple phonetic contexts D. Determining speech production in a variety of syllables-to-sentence combinations

D. Determining speech production in a variety of syllables-to-sentence combinations

Which of the following is the most important acoustic cue that distinguishes between an unreleased final /p/ and an unreleased final /b/, as in "cap" versus "cab"? A. Locus frequency of burst B. Voice onset time C. Vocal fundamental frequency D. Duration of the preceding vowel

D. Duration of the preceding vowel

An SLP conducts a videofluoroscopic swallowing study (VFSS) for a patient referred for suspected dysphagia secondary to stroke. The SLP finds that the patient has dysphagia characterized by delayed onset of the pharyngeal response, limited lingual retraction, limited oropharyngeal propulsion, and the need to swallow two or more times to clear boluses. Some aspiration of hypopharyngeal residue occurs after the swallow from the pyriform sinuses. During the VFSS, several compensatory maneuvers (chin-down posture) were attempted to test their efficacy. The SLP is developing a treatment plan to eliminate aspiration and to improve lingual strength, retraction, and oropharyngeal propulsion through a combined strategy using compensatory and restorative maneuvers, including a progressive resistive exercise program of isometric tongue-press exercises using an instrument that measures the pressure of tongue contact with the hard palate (IOPI or SwallowStrong). The SLP measures the patient's baseline maximum tongue-press pressure generation with the instrument and begins treatment. Which of the following interventions could be deployed while the patient is swallowing to increase intrabolus pressure? A. Tongue tether maneuver B. Mendelsohn maneuver C. Breath-holding with Valsalva maneuver D. Effortful swallow maneuver

D. Effortful swallow maneuver

Mr. Charles, age 78, has had Alzheimer's disease for the past nine years. A recent speech and language evaluation at his nursing home indicated severe deficits in verbal reasoning, memory, word finding, discourse, pragmatics, phonology, semantics, and syntax. Which of the following should the speech-language pathologist do next? A. Recommend individual treatment to improve his receptive and expressive language skills, beginning with the areas where he is least deficient B. Recommend individual treatment to improve his receptive and expressive language skills, beginning with the areas where he is most deficient C. Initiate group treatment with other adults with language impairments to improve spontaneous conversational speech and pragmatic skills D. Examine his living conditions and, if necessary, educate the nursing staff concerning ways to help him communicate more effectively

D. Examine his living conditions and, if necessary, educate the nursing staff concerning ways to help him communicate more effectively

During a trip to the grocery store, a three-year-old boy with delayed language points to an apple on the floor and says to his mother, "Apple." The boy's mother says, "Yes, that's a big, shiny apple." Based on the mother's response, which of the following language stimulation techniques is she using? A. Imitation B. Parallel talk C. Think aloud D. Expansion

D. Expansion

Which of the following is a type of perturbation that can be measured to determine the amount of noise in the voice? A. Changes in the frequency range between F1 and F2 over time B. Changes in the frequency range between F2 and F3 over time C. F3 cycle-to-cycle variations in sound energy over time D. F0 cycle-to-cycle variations in sound energy over time

D. F0 cycle-to-cycle variations in sound energy over time

Infants with cleft lip and palate are susceptible to middle ear disease because which of the following muscles is commonly impaired? A. The superior constrictor muscle B. The levator veli palatini muscle C. The palatopharyngeus muscle D. The tensor veli palatini muscle

D. The tensor veli palatini muscle

If a child's language exhibits the phonological process of gliding, the child might say [wɛd] for "red." When asked, "Do you mean wed?" the child may respond, "No! [wɛd]!" Such a response demonstrates which of the following? A. Phonological development lags behind semantic development. B. Semantic development lags behind phonological development. C. Linguistic competence lags behind linguistic performance. D. Linguistic performance lags behind linguistic competence.

D. Linguistic performance lags behind linguistic competence.

Based on a phonemic approach of operant conditioning, which treatment activity can be used to expand an individuals underlying knowledge of sound system and features? A. Minimal Pairs B. Multiple Oppositions C. Phonetic Placements D. Maximal oppositions

D. Maximal oppositions

Which intrinsic muscle of the larynx is the only abductor muscle of the vocal folds? A. Lateral Cricoarytenoid B. Transverse Arytenoid C. Oblique Arytenoid D. Posterior Cricoarytenoid E. None of the above

D. Posterior Cricoarytenoid

A 75-year-old patient is referred from a laryngologist with a diagnosis of presbylaryngis/presbyphonia. The patient complains of reduced vocal loudness and increased vocal effort while speaking. Which of the following treatments is most appropriate for the patient? A. Using sigh phonation with soft glottal attacks B. Generating vowel sounds with inhalatory phonation C. Completing voice rest for one week then gradually reintroducing voice use D. Producing vowel sounds and phrases using high vocal intensity phonation

D. Producing vowel sounds and phrases using high vocal intensity phonation

Which of the following instruments is often used to document stages of recovery after traumatic brain injury (TBI) ? A. Glasgow Coma Scale B. Scale of Executive Function C. Sequenced Inventory of Communication Development D. Rancho Los Amigos Scales of Cognitive Function

D. Rancho Los Amigos Scales of Cognitive Function

An SLP is reviewing research literature for an evidence-based communication intervention for a young child with autism spectrum disorder (ASD). Which of the following research designs provides the strongest proof of efficacy for evidence-based communication intervention? A. Case studies describing a successful treatment for a child with ASD B. Single case designs that rigorously test an intervention for a child with ASD C. A group design with children receiving two different ASD treatments simultaneously D. Randomized controlled trials of a treatment including children with ASD

D. Randomized controlled trials of a treatment including children with ASD

In the treatment of voice disorders, the chewing technique is used to do which of the following? A. Improve control of loudness B. Increase pitch range during voice production C. Increase air supply during voice production D. Reduce tension in the laryngeal area

D. Reduce tension in the laryngeal area

A patient presents to a speech-language pathologist with voice changes. After completing a patient assessment, the SLP notes that the patient's vocal quality and function indicate the possibility of a superior laryngeal nerve injury. Which of the following presentations is most consistent with this impression? A. Inspiratory stridor B. Hypernasal resonance C. Complete aphonia D. Reduced pitch range

D. Reduced pitch range

An SLP has a consultation with a self-referred adult who has a fluency disorder. The client had been enrolled in treatment programs with the clinician three times before and had reached from 75 to 90 percent fluency before dropping out of treatment for various reasons. Thirty percent of the client's syllables are spoken disfluently. The client also exhibits signs of depression and anxiety. Which of the following is the most appropriate action for the SLP to take? A. Encouraging the client to re-enroll for remedial services B. Encouraging the client to take responsibility for maintaining fluency by using techniques learned in the previous treatment sessions C. Recommending that the client schedule a neurological evaluation D. Referring the client for psychological counseling

D. Referring the client for psychological counseling

Which of the following is typical of spondee words? A. They are phonetically balanced two-syllable words. B. They are phonetically balanced monosyllabic words. C. They represent twenty of the most common English words. D. They are two-syllable words produced with equal stress.

D. They are two-syllable words produced with equal stress.

Ankyloglossia may be an organic articulation or phonological disorder. Ankyloglossia is a medical term for what? A. Cleft palate B. Tongue spots C. Enlarged tongue D. Tongue tie

D. Tongue tie

Disfluent speech, intact repetition skills, and impaired writing are characteristics of which type of aphasia? A. Broca's aphasia B. Wernicke's aphasia C. Transcortical Sensory Aphasia D. Transcortical Motor Aphasia E. None of the above F. A & B

D. Transcortical Motor Aphasia

A 24-year-old male self-refers for a fluency evaluation. His presenting complaint is stuttering. During conversation at the initial assessment, he speaks intelligibly at a typical rate and produces no overt stutter-like disfluencies. He reports that he often expects to stutter while conversing, but that he usually can prevent or conceal the occurrence of the expected fluency disruptions either by substituting a word or by inserting a pause or "um" before the word upon which he expects to be disfluent. He states that these strategies are useful and that he would like to be able to "talk without thinking about talking." He reports that he attended speech therapy from elementary school through high school and that it helped him reduce disfluency significantly during therapy activities, but his disfluency frequency did not change much during activities outside of therapy. He fears that coworkers will react negatively to hearing him stutter. Consequently, he talks as little as possible at work. Which of the following approaches is the most appropriate initial therapy focus for the patient? A. Improving the naturalness of the speech B. Implementing use of the cancellation technique in conjunction with conversational disfluencies C. Improving the ability to pause at appropriate linguistic boundaries when speaking D. Reinforcing the patient's attempts to produce words without the use of fillers, pauses, or word substitutions

D. Reinforcing the patient's attempts to produce words without the use of fillers, pauses, or word substitutions

According to the Diagnostic and Statistical Manual of Mental Disorders, Fifth Edition (DSM-5™) criteria for autism spectrum disorder (ASD) and social communication disorder (SCD), which of the following characteristics best helps an SLP provide a differential diagnosis for ASD instead of SCD? A. Deficits in communication appropriate for a social context B. Deficits that result in functional limitations in social participation C. Difficulty following rules for storytelling D. Repetitive and restrictive patterns of behavior

D. Repetitive and restrictive patterns of behavior

Which of the following is used to improve the performance of struggling students who receive scientifically based instruction in a general education classroom? A. Discrepancy formula model B. Early intervention C. Multitier intervention model D. Response to Intervention

D. Response to Intervention

Of the following, which is generally the most appropriate treatment goal for clients who have had a laryngectomy? A. Acceptance of the alaryngeal status B. Production of an esophageal voice C. Use of a voice prosthesis D. Restoration of oral communication

D. Restoration of oral communication

A 24 month old who was screened for autism spectrum disorder (ASD) attends a twice-weekly early intervention program to address developmental delays. Which of the following is the best way to determine the communication goals for the child? A. Aligning with developmental norms for communication behaviors typical of 24 month olds B. Waiting to address communication goals only after behavioral issues have been remediated C. Aligning with goals of other children in the program to heighten intensity of intervention efforts D. Setting goals that address communication challenges within daily activities and routines

D. Setting goals that address communication challenges within daily activities and routines

Ms. Helene, a 60-year-old woman with a suspected neurological disorder, is referred for speech-language evaluation. She achieves a score of 35/50 on a measure of confrontation object-naming ability. This score is below norms established for individuals of her age and educational level. Based on these results alone, which of the following statements can most reliably be made about Ms. Helene's disorder? A. She has anomic aphasia, which might or might not be associated with other language deficits. B. She has aphasia, but the type of aphasia cannot be specified on the basis of this test score alone. C. She does not have aphasia but probably does have a memory disturbance. D. She has difficulty with naming, but the precise nature of the deficit cannot be determined on the basis of this test score alone.

D. She has difficulty with naming, but the precise nature of the deficit cannot be determined on the basis of this test score alone.

Which sampling procedure is first divided into attribute categories before randomly choosing participants? A. Simple random sampling B. Consecutive sampling C. Systematic sampling D. Stratified random sampling E. None of the above

D. Stratified random sampling

Which of the following nerves provides efferent innervation to the stylopharyngeus muscle and contributes toward the elevation of the pharynx and the larynx? A. The trigeminal B. The hypoglossal C. The vagus D. The glossopharyngeal

D. The glossopharyngeal

A 5-year-old child's evaluation reveals a developmental speech delay secondary to an intellectual disability. Which of the following statements about etiology is most likely true? A. A genetic etiology can be assumed and treatment can be withheld. B. A genetic etiology can be assumed and compensatory strategies should be taught. C. Prenatal trauma can be assumed and a plan to counsel the parents is necessary. D. The information provided does not allow an assumption to be made on etiology.

D. The information provided does not allow an assumption to be made on etiology.

A 12-year-old native speaker of Spanish who has been studying English as a second language for three years is most likely to do which of the following when speaking English in casual conversation with teachers at school? A. Use the auxiliary "have" in place of "be" in progressive tenses B. Use incorrect word order within prepositional phrases C. Use conjunctions in place of prepositions D. Use multiple negation improperly

D. Use multiple negation improperly

Which of the following recommendations to a parent of an infant with cleft lip and palate should an SLP make to best optimize feeding for adequate nutrition and appropriate growth? A. Providing a bottle rather than breast-feeding the infant B. Positioning the infant in a supine position during feeding times C. Allowing the infant to take as much time as needed to feed D. Using a bottle with a modified nipple during feedings

D. Using a bottle with a modified nipple during feedings

An SLP is working with a sixth-grade student who is having difficulty comprehending written text. Which of the following strategies would be most effective for the SLP to use to target this difficulty? A. Creating a log in which unfamiliar sight words are written down for later practice B. Developing the student's ability to chunk multisyllabic words into smaller units C. Selecting words from a text that can be used as both spelling and sight words D. Using semantic networks in which ideas are displayed in connected clusters

D. Using semantic networks in which ideas are displayed in connected clusters

Which of the following plans is most appropriate when assessing a patient with a history of traumatic brain injury (TBI) to characterize cognitive aspects of communication that affect functional abilities? A. Administering an aphasia battery to form a comprehensive assessment of the patient's linguistic abilities B. Conducting an assessment to evaluate for the presence and severity of dysarthria C. Observing a conversation between the patient and a familiar individual for difficulties in functional communication D. Using standardized tests as needed and supplementing with discourse samples and dynamic assessment

D. Using standardized tests as needed and supplementing with discourse samples and dynamic assessment

Which of the following is an organic structural voice disorder? A. muscle tension dysphonia B. Puberphonia C. Vocal fatigue D. Vocal polyps E. None of the above

D. Vocal polyps

Which of the following actions will most effectively control the problem of overreferral in school screening programs that use impedance/immittance measurements? A. Obtaining the measurements in a professional sound-insulated room B. Including 500 Hz in the audiometric screening procedure C. Retesting immediately those who did not pass the first screening D. Waiting three to five weeks to retest those who did not pass the first screening

D. Waiting three to five weeks to retest those who did not pass the first screening

A child repeatedly inserts an inappropriate sound in certain environments; for example, [fpɪʃ] for [fɪʃ] . Which of the following would likely be most helpful for the child as a target for treatment? A. Bisyllabic words for which a minimal-contrast pair can be easily identified B. Words containing phonemes that have distinctive features in common with the sound the child inserts inappropriately C. Repeated practice with the combinations of phonemes that the child finds particularly easy to produce D. Words that contrast the child's error pattern with the target pattern in the word

D. Words that contrast the child's error pattern with the target pattern in the word

Primary motor innervation to the larynx and velum is provided by which cranial nerve? A. V B. VII C. XI D. X

D. X

Which of the following criteria must be met prior to using and implementing an AAC device with your patients? A. Mild score intellectual testing B. Adequate use of limbs C. Have a few words D. Zero exclusion criteria

D. Zero exclusion criteria

For a test of expressive morphology and syntax for speakers of African American Vernacular English (AAVE), the test item that would be considered LEAST biased against such speakers would be one requiring: A. use of the auxiliary verb "be" in the present progressive tense B. use of the past-tense ending "-ed" C. use of sentences with multiple negation D. agreement of personal pronouns with their antecedents in gender and number

D. agreement of personal pronouns with their antecedents in gender and number

Michael is a 32-month-old boy who has been receiving early intervention services over the past ten months for delayed speech and expressive-language development. Although his birth was reportedly unremarkable, Michael does have a history of recurrent otitis media with effusion. His parents described him as having been a "well-behaved and quiet baby." When Michael began receiving services, he communicated mainly through gestures and crude vocalizations. An open resting mouth position with slight tongue protrusion was sometimes noted. However, his receptive-language skills were found to be age appropriate and he showed no oral motor deficits during feeding. Michael's expressive-language skills have shown some progress since he began working with the speech-language pathologist, but he remains poorly intelligible. Michael's imitation of tongue, lip, and jaw movements is characterized by inconsistent groping and errors of sequencing not observed in his spontaneous oral movements. Michael has an age-appropriate vocabulary and produces utterances of up to five words. Articulation errors, especially metathesis of phones and syllables, increase as his utterance length increases. Michael's intelligibility is greatest at the single-word level. Automatic speech and highly familiar utterances are much more intelligible than his imitated productions. As part of ongoing assessment, the SLP's most appropriate action is to have Michael's parents consult with his primary-care provider for referral to A. a special educator B. a neurologist C. a psychologist D. an otolaryngologist

D. an otolaryngologist

Intervention from an SLP for a nursing-home resident who is in a late stage of progressive dementia will most effectively focus on: A. conversational intelligibility B. recall of salient vocabulary words C. comprehension of social discourse D. assisting in communication routines

D. assisting in communication routines

Which of the following phonological processing errors is atypical? A. fronting B. final consonant deletion C. gliding D. backing E. A, B, and C F. None of the above

D. backing

An SLP engages in interprofessional practice in a large health care center and works with a neurologic rehabilitation team to treat a patient who is status post left-hemisphere stroke. The SLP's primary focus on the team is to: A. lead the interdisciplinary team to improve patient care B. reduce costs and the number of professionals working at the site C. obtain licenses in multiple areas of practice to engage in a range of services D. improve the patient experience and outcome of care at the site

D. improve the patient experience and outcome of care at the site

Michael is a 32-month-old boy who has been receiving early intervention services over the past ten months for delayed speech and expressive-language development. Although his birth was reportedly unremarkable, Michael does have a history of recurrent otitis media with effusion. His parents described him as having been a "well-behaved and quiet baby." When Michael began receiving services, he communicated mainly through gestures and crude vocalizations. An open resting mouth position with slight tongue protrusion was sometimes noted. However, his receptive-language skills were found to be age appropriate and he showed no oral motor deficits during feeding. Michael's expressive-language skills have shown some progress since he began working with the speech-language pathologist, but he remains poorly intelligible. Michael's imitation of tongue, lip, and jaw movements is characterized by inconsistent groping and errors of sequencing not observed in his spontaneous oral movements. Michael has an age-appropriate vocabulary and produces utterances of up to five words. Articulation errors, especially metathesis of phones and syllables, increase as his utterance length increases. Michael's intelligibility is greatest at the single-word level. Automatic speech and highly familiar utterances are much more intelligible than his imitated productions. Based on Michael's case history, the SLP would most appropriately provide activities to A. stabilize muscle tone to improve gross motor support of speech movement B. improve receptive-language ability C. strengthen tongue, lip, and jaw muscles D. increase the accuracy of CV, VC, and CVC syllable sequences

D. increase the accuracy of CV, VC, and CVC syllable sequences

Elisions and transpositions are referred to as phoneme A. additions B. blending C. deletions D. manipulations

D. manipulations

Following anoxic encephalopathy, clients are likely to experience the most significant long-term impairments in the area of A. prosody B. resonance C. aphonia D. memory

D. memory

Research regarding the use of intensive phonemic-awareness treatment for children who have difficulty learning to read has demonstrated that the treatment A. is effective only for children from 4 to 8 years old B. is effective mainly with children who have remediated all phonological process errors C. is effective only when combined with a supplemental literacy program D. might have no direct relationship to improvement in reading abilities

D. might have no direct relationship to improvement in reading abilities

a 78-year old female is reporting loss of her taste. Findings from her VFSE reveal age related changes. Which of the following treatment would you recommend? A. Modified diet B. Oral motor exercises C. Pharyngeal strength exercises D. no speech therapy indicated

D. no speech therapy indicated

Two months after undergoing surgery to improve velopharyngeal function, a client continues to exhibit nasal airflow only on the production of /s/ and /sh/, and also exhibits glottal stops for several pressure consonants. The most appropriate next action for the SLP to take is to A. request consideration of prosthetic management B. request a nasoendoscopic study C. inform the surgeon that the client is not making satisfactory progress D. provide speech treatment to correct compensatory articulation errors

D. provide speech treatment to correct compensatory articulation errors

For a patient with communication and swallowing disorders secondary to acquired immunodeficiency syndrome (AIDS), a speech-language pathologist will most appropriately: A. serve only as a consultant to others who are directly involved in the patient's care B. provide treatment on a monthly basis C. provide treatment only as prescribed by the patient's physician D. provide treatment in consultation with the patient's primary-care physician or medical team

D. provide treatment in consultation with the patient's primary-care physician or medical team

The major objective of auditory training in the treatment of a client with a hearing loss is to A. improve the client's awareness of position and movements of the speech mechanism B. improve the client's kinesthetic and auditory awareness C. increase the client's kinesthetic and proprioceptive discrimination D. teach the client to make discriminations among speech sound

D. teach the client to make discriminations among speech sound

In terms of communication impairment, an adult with Alzheimer's-type dementia tends to differ from an adult with aphasia associated with a CVA in that: A. repetition abilities are typically more seriously impaired in the adult with dementia B. the adult with dementia generally experiences greater dysfluency C. the adult with dementia is generally more aware of any disruptions in communication D. the capacity of the adult with aphasia to make appropriate comments about recent events is generally better

D. the capacity of the adult with aphasia to make appropriate comments about recent events is generally better

Ms. Lopez's articulation errors consist of the following: f/v, ʃ/ʒ, and s/z. On the basis of these errors, the SLP should begin remediation that focuses on: A. manner B. place C. fricatives D. voicing

D. voicing

Which of the following professionals must be included in every IEP team? A. Parents B. Principal C. School Psych D. School District Rep E. A, C, and D F. All of the above

E. A, C, and D

Which of the following is a muscle that originates and extends within the tongue? A. Hypoglossus muscle B. Genioglosses muscle C. Inferior longitudinal muscle D. Superior longitudinal muscle E. C & D

E. C & D

What Law Replaced the "No Child Left Behind" Act?

ESSA - "Every Student Succeeds Act"

What is the role of the stylopharyngeus muscle?

Elevates and opens the pharynx

By 3 months old, typically developing infants consistently do which of the following? A. Follow 1-step directions B. Cry for basic needs C. Look when name is called D. Quiet in response to noise E. B & C F. B & D

F. B & D

What type of dysarthria is characterized by imprecise articulation, monotonous pitch and loudness, and harsh vocal quality?

Flaccid Dysarthria

When a person is producing a voiced and voiceless /th/, the muscle that is most involved is the ___.

Genioglossus

A natural frequency is a frequency: I. with which a source of sound vibrates naturally II. that is affected by the mass and stiffness of the vibrating body III. that is the center frequency of a formant IV. that is the first harmonic V. that is the lowest frequency of a periodic wave

I, II

When a person is adducting and abducting the vocal folds, which of the following muscles are the most involved in that process? I. Thyroarytenoid II. Digastrics III. Lateral Cricoarytenoid IV. Transverse Arytenoids V. Sternothyroid

I, III, IV

What muscles serve to elevate the soft palate?

Levator veli palatini

The structure at the inferior portion of the tongue that connects the tongue with the mandible is called the

Lingual frenum

_____ states that vocal folds vibrate because of the forces, air pressure, and elasticity of the vocal folds.

Myoelastic-Aerodynamic theory

What muscle is circled?

Orbicularis Oris

The cranial nerves are composed of 12 pairs of nerves and belong to the ___ nervous system. A. Central B. Autonomic C. Somatic D. Peripheral

Peripheral

primary motor cortex in the frontal lobe is located on the ___.

Precentral gyrus

the CNS's primary mechanism of attention, alertness, and consciousness, which is also related to sleep-wake cycles is the ___.

Reticular Activating System

Medicaid and Medicare emerged from what government program?

The Social Security Act

Broad phonemic transcription involves:

The use of IPA symbols to transcribe phonemes by enclosing them within slash marks (e.g., /f/)

Most pharyngeal muscles are innervated by cranial nerves:

X & XI

Based on the IDEA, early intervention is provided from ___ to ___ y/o. a. Birth-4 y/o b. Birth-3 y/o c. 1 month-5 y/o d. 3 months-3 y

b. Birth-3 y/o

Lack of fusion of the lip and alveolus during embryological development will cause what type of craniofacial malformation? a. Complete secondary palate cleft b. Cleft of the primary palate c. Cleft of the secondary palate d. Isolated cleft of the lip only

b. Cleft of the primary palate

Downgrading a patients diet to puree textures is an example of what type of dysphagia treatment technique? a. Sensory b. Compensatory c. Maneuver d. Muscle strengthening

b. Compensatory

Jimmy is 3 years old and is substituting affricates for fricatives. [E.g., "ship" for "chip"] Which phonological process is Jimmy demonstrating? a. Backing b. Deaffrication c. Stopping d. Depalatalization

b. Deaffrication

Holding the larynx in an elevated position while swallowing is an example of which swallowing maneuver? a. Supraglottic swallow b. Mendelson c. Super supraglottic swallow d. Effortful swallow

b. Mendelson

Tommy is 7 months old and recently began babbling. He produces chains such as "mama" and "dada." What type of babbling is Tommy demonstrating? a. Variegated babbling b. Reduplicated babbling c. Cooing d. None of the above

b. Reduplicated babbling

Barbara, an 82 year old female, presented to her primary MD with the symptoms listed below over the course of the past year. What medical diagnosis does Barbara's symptoms most closely align with? (1) unsteady frozen gait, (2) decline in short term memory, (3) tremors, (4) trouble swallowing, and a (5) low volume when speaking a. Anoxia b. Huntington's Disease c. Parkinson's Disease

c. Parkinson's Disease

The cerebral hemispheres are connected by ___.

commissural fibers

Language is often classified into three components: form, content, and use. Which of the following is an example of language form? a. Syntax b. Pragmatics c. Morphology d. A and C e. All of the above

d. A and C

Jen suffered a cerebellar stroke and her husband describes her speech as "drunk and inebriated." What type of speech disorder does Jen most likely have? a. Spastic dysarthria b. Hyperkinetic dysarthria c. Flaccid Dysarthria d. Ataxic dysarthria

d. Ataxic dysarthria

Tongue fasciculations, slow and slurred diadochokinetic rates, and a breathy quality are all symptoms of which dysarthria type? a. Spastic b. Ataxic c. Hyperkinetic d. Flaccid

d. Flaccid

What is the first grammatical morpheme to be used expressively by a child?

progressive -ing

The term coarticulation refers to

the influence of one phoneme upon another in production and perception, wherein two different articulators move simultaneously to produce two different speech sounds


संबंधित स्टडी सेट्स

DECA Entrepreneurship Questions 40-60

View Set

Mkt 301: Test 2 (Chapter 6,7,11,12,&14)

View Set

What is a Computer? - Matching Activity

View Set

Intermediate Accounting Chapter 1 and 2

View Set

Research for Senior High School: Inquiries, Investigations, and Immersion

View Set

Ch 6 - Bacterial Metabolism pt 1_ Microbiology

View Set